tabe pert fcat reading and writing practice mmacvcerreiacks

141
TABE PERT FCAT Reading and Writing Practice 1. Which word is not spelled correctly? A: engage B: judicious C: authority D: batalion 2. Which word is not spelled correctly? A: commence B: commitee C: commerce D: commingle 3. Which word is not spelled correctly? A: issue B: asbestos C: cronic D: stodgy For the following four questions, read the sentence and indicate which part is incorrect. 4. 'I don't want too and you can't make me,' he said. A: Correct as is B: too C: me,' D: 'I don't 5. Many people would immediately stop smoking if they knew only all the dangers, says the report. A: Correct as is B: immediately stop C: dangers, says D: knew only 6. Lester, a giant for his age; was respected by all the other children. A: Correct as is B: Lester, a C: a giant for D: age; was 7. Asking for directions when you are lost is the properly course of action. A: Correct as is B: you are lost C: for directions D: properly Read the student essay and answer the three questions that follow. How to Make a Cheese Sandwich First you take two pieces of bread. Then you put mustard or mayonnaise on them whichever you like. Then you can add the cheese. I like chedder cheese but you can use another kind. Some people like to put their cheese sandwiches in the toaster oven for a little while. I love to eat my cheese sandwich with a bowl of tomato soup. 8. Which would be the best main introductory sentence for this essay? A: The ingredients for a cheese sandwich are mustard, bread , and cheese. B: Cheese sandwiches are delicious and easy to make. C: I love cheese sandwiches. D: Cheese sandwiches are extremely nutritious. 9. What grammatical advice would you give the student about the sentence beginning 'I like____'? A: The sentence is perfect as it is. B: This sentence needs a comma after 'cheese.' C: The sentence needs a colon after 'cheese.' D: The sentence makes no sense. 10. Which word in the essay is misspelled? A: tomato B: mayonnaise C: chedder D: sandwich Answer Key 1. D. The correct spelling is battalion. 2. B. The correct spelling is committee. 3. C. The correct spelling is chronic. 4. B. The form should be to. 5. D. The words knew and only should be switched. 6. D. A comma should be used here rather than a semi-colon 7. D. The adjectival form rather than the adverbial form should be used here.

Upload: steve-mccrea-facilitator

Post on 04-Apr-2015

1.641 views

Category:

Documents


4 download

DESCRIPTION

MmAcVcErReIaCKS

TRANSCRIPT

Page 1: TABE PERT FCAT Reading and Writing Practice    MmAcVcErReIaCKS

TABE PERT FCAT Reading and Writing Practice

1. Which word is not spelled correctly? A: engage B: judicious C: authority D: batalion

2. Which word is not spelled correctly? A: commence B: commitee C: commerce D: commingle

3. Which word is not spelled correctly? A: issue B: asbestos C: cronic D: stodgy

For the following four questions, read the sentence and indicate which part is incorrect.

4. 'I don't want too and you can't make me,' he said. A: Correct as is B: too C: me,' D: 'I don't

5. Many people would immediately stop smoking if they knew only all the dangers, says the report. A: Correct as is B: immediately stop C: dangers, says D: knew only

6. Lester, a giant for his age; was respected by all the other children. A: Correct as is B: Lester, a C: a giant for D: age; was

7. Asking for directions when you are lost is the properly course of action. A: Correct as is B: you are lost C: for directions D: properly

Read the student essay and answer the three questions that follow. How to Make a Cheese Sandwich

First you take two pieces of bread. Then you put mustard or mayonnaise on them whichever you like. Then you can add the cheese. I like chedder cheese but you can use another kind. Some people like to put their cheese sandwiches in the toaster oven for a little while. I love to eat my cheese sandwich with a bowl of tomato soup.

8. Which would be the best main introductory sentence for this essay? A: The ingredients for a cheese sandwich are mustard, bread , and cheese. B: Cheese sandwiches are delicious and easy to make. C: I love cheese sandwiches. D: Cheese sandwiches are extremely nutritious.

9. What grammatical advice would you give the student about the sentence beginning 'I like____'? A: The sentence is perfect as it is. B: This sentence needs a comma after 'cheese.' C: The sentence needs a colon after 'cheese.' D: The sentence makes no sense.

10. Which word in the essay is misspelled? A: tomato B: mayonnaise C: chedder D: sandwich

Answer Key

1. D. The correct spelling is battalion.

2. B. The correct spelling is committee.

3. C. The correct spelling is chronic.

4. B. The form should be to.

5. D. The words knew and only should be switched.

6. D. A comma should be used here rather than a semi-colon

7. D. The adjectival form rather than the adverbial form should be used here.

8. B. This sentence best summarizes the essay.

9. B. A comma would be an effective way of clarifying this sentence.

10. C. The correct spelling is cheddar.

==============

Reading Practice Questions

Page 2: TABE PERT FCAT Reading and Writing Practice    MmAcVcErReIaCKS

Read the passage and answer the five questions that follow. Philosophers have explained space. They have not explained time. It is the inexplicable raw material of everything. With it, all is possible; without it, nothing. The supply of time is truly a daily miracle, an affair genuinely astonishing when one examines it. You wake up in the morning, and lo! your purse is magically filled with twenty-four hours of the unmanufactured tissue of the universe of your life! It is yours. It is the most precious of possessions. A highly singular commodity, showered upon you in a manner as singular as the commodity itself!

For remark! No one can take it from you. It is unstealable. And no one receives either more or less than you receive.

Talk about an ideal democracy! In the realm of time there is no aristocracy of wealth, and no aristocracy of intellect. Genius is never rewarded by even an extra hour a day. And there is no punishment. Waste your infinitely precious commodity as much as you will, and the supply will never be withheld from you. No mysterious power will say:--"This man is a fool, if not a knave. He does not deserve time; he shall be cut off at the meter." It is more certain than consols [Roman coins], and payment of income is not affected by Sundays. Moreover, you cannot draw on the future. Impossible to get into debt! You can only waste the passing moment. You cannot waste to- morrow; it is kept for you. You cannot waste the next hour; it is kept for you.

1. What is the 'unmanufactured tissue of the universe of your life'? A: time B: flesh C: paper D: blood

2. In the final sentence of the first paragraph, what does the author mean by the word 'singular'? A: not plural B: unmarried C: perfect D: unique

3. Why does the author consider the 'realm of time' to be an 'ideal democracy'? A: Because every person gets one vote. B: Because everyone gets exactly what they want. C: Because you can borrow time as much as you want. D: Because everyone gets the exact same amount every day.

4. What does the author mean by the phrase 'draw on the future'? A: sketch B: borrow C: loan D: pull in a cart

5. The main idea of this passage is: A: every person gets the same amount of time B: it is possible to borrow time C: some people get more time than others D: philosophers have explained space

The following passage is being read by students in your class. Answer the questions that follow. It was seven o'clock of a very warm evening in the Seeonee hills when Father Wolf woke up from his day's rest, scratched himself, yawned, and spread out his paws one after the other to get rid of the sleepy feeling in the tips. Mother Wolf lay with her big gray nose dropped across her four tumbling, squealing cubs, and the moon shone into the mouth of the cave where they all lived. ``Augrh!'' said Father Wolf, ``it is time to hunt again''; and he was going to spring downhill when a little shadow with a bushy tail crossed the threshold and whined: ``Good luck go with you, O Chief of the Wolves; and good luck and strong white teeth go with the noble children, that they may never forget the hungry in this world.''

6. If you want your students to picture the scene described in the passage, which of the following activities would be most appropriate? A: Recording wolf sounds. B: Reading a nonfiction article about wolves. C: Dressing up in wolf costumes. D: Drawing pictures of the wolves in their cave.

7. What is one thing that students can tell about the wolves' habits from this passage? A: They are friendly to other animals. B: They love to eat vegetables. C: They sleep during the day. D: They only hunt by a full moon.

8. If you want your students to describe Father Wolf's attitude towards hunting, which of the following activities would be most appropriate? A: Write a short story with Father Wolf as the main character. B: Make a list of all the things Father Wolf does and says. C: Dress up like Father Wolf. D: Ask the students to imagine how they would feel if they were a wolf.

9. If you ask your students to find the words that describe the cubs, which would be the correct answer? A: scratched and yawned B: strong and white C: bushy and whined D: tumbling and squealing

10. If you ask your students who the passing visitor might be, which would be the most accurate answer? A: a donkey B: a squirrel C: a wolf D: a lizard

Answer Key

1. A. The author asserts that every life is made up of time.

2. D. The author goes on to describe the ways in which time is unique.

3. D. The author elaborates on how no person can buy, earn, or borrow extra time.

4. B. This phrase means to borrow something before it is due.

5. A. The author marvels that every person, no matter their position, must have the same relationship with time.

6. D. The passage is filled with visual details that lend themselves to drawing.

7. C. The passage alludes to their 'day's rest.'

8. B. Students can get a better idea of a character's personality by isolating his or her words and deeds.

9. D. The words tumbling and squealing are used to describe the cubs.

10. B. The visitor's bushy tail indicates that he might be a squirrel.

Page 3: TABE PERT FCAT Reading and Writing Practice    MmAcVcErReIaCKS

Comma Practice Questions

1. For the Thanksgiving reunion, relatives were sitting in the dining room, on the porch, and in the carport.

A. Thanksgiving, reunion B. Were, sitting C. Porch and D. No error

2. Lydia seems to be a kind, considerate girl.

A. Seems, to B. Considerate, girl C. Kind considerate D. No error

3. This fishing pole Nathan, has seen better days.

A. Pole, Nathan, B. Has, seen C. Nathan, D. No error

4. My cousin has moved to 56 Central Street Narragansett, Rhode Island 02882.

A. Has moved, B. Central Street, C. 56, Central D. No error

5. The badger, a shy animal sometimes makes friends with a coyote.

A. Sometimes, makes B. Friends, with C. A shy animal, D. No error

6. After the death of Blackbeard, the famous pirate, piracy disappeared from the coast of the American colonies.

A. The famous pirate B. After the death, C. Coast, of D. No error

7. “Silent Night” was written by two men from the village of Oberndorf Austria.

A. men, from B. Silent Night, C. Oberndorf, Austria D. No error

8. On November 19, 1929 Admiral Richard E. Byrd flew the Floyd Bennett to the base of the Queen Maud Mountains.

A. Base, of B. The, Queen C. 1929, D. no error

9. Oh I forgot to bring the cookies.

Page 4: TABE PERT FCAT Reading and Writing Practice    MmAcVcErReIaCKS

A. Oh, B. I, forgot C. To, bring D. No error

10. “The boy in the kayak,” whispered Sue “is the new football captain."

A. Boy, in the B. New, football C. Whispered Sue, D. No error

Answer Key 1. D 2. D 3. A 4. B 5. C 6. D 7. C 8. C 9. A 10. C 

1. Everyone in the bank-including the manager and the tellers, ran to the door when the fire alarm rang.

A. tellers, ran B. tellers:ran C. tellers, had run D. tellers-ran E. tellers' ran”

2. To no ones surprise, Joe didn't have his homework ready.

A. no ones surprise B. noones surprise C. no-ones surprise D. no ones' surprise E. no one's surprise

3. If he would have read “The White Birds,” he might have liked William Butler Yeats's poetry.

A. would have read B. could have read C. would of read D. could of read E. had read

4. After the hurricane, uprooted trees were laying all over the ground.

A. were laying B. lying C. were lying D. were laid E. was laid

5. Ralph Waldo Emerson (1803-1882), the great Transcendentalist philosopher, wrote in his essay “Self-Reliance” of the need for an individual to develop his capacities.

A. essay “Self–Reliance” B. essay, “Self-Reliance” C. essay: Self-Reliance D. essay, Self-Reliance E. essay; “Self-Reliance”

6. The recently built children's amusement park has been called “ a boon to the community “ by its supportersand “an eyesore” by its harshest critics.

A. and “an eyesore” by its harshest B. and, “ an eyesore,” by its harshest C. and, an eyesore; by its harshest D. and-an eyesore- by its' harshest E. and-“an eyesore”- by its' harshest

7. I always have trouble remembering the meaning of these two common verbs, affect (to change” or “to influence”) and effect (“to cause” or “to accomplish ) . “

A. “ to accomplish ). “ B. “ to accomplish” ). C. “to accomplish). D. To accomplish. E. ( “ to accomplish. “ )

Page 5: TABE PERT FCAT Reading and Writing Practice    MmAcVcErReIaCKS

8. My class just finished reading- “ The Fall of the House of Usher “, a short story by Edgar Allen Poe.

A. reading- “ The Fall of the House of Usher”, B. reading, The Fall of the House of Usher, C. reading “The Fall of the House of Usher, “ D. reading, The Fall of the house of Usher, “ E. reading: The Fall of the House of Usher-

9. After it was repaired it ran perfect again.

A. ran perfect B. ran perfectly C. could run perfect D. could of run perfect E. would run perfectly

10. "Are there two e's in beetle," asked Margo?

A. there two e's in beetle," asked Margo? B. their two e's in beetle?" asked Margo. C. there two e's in beetle," asked Margo? D. there two e's in beetle?" asked Margo. E. there two e's in beetle, asked Margo?

11. The circus audience received a well-deserved round of applause for the perfectly timed acrobatic stunt.

A. audience received a well-deserved B. audience gave a well deserved C. audience did receive a well deserved D. audience gave a well-deserved E. audience did get a well-deserved

12. Looking directly at me, my Mother said, “ These are your options: the choice is yours.”

A. Mother said, “ These are your options: the choice is B. Mother said- these are your options, the choice is C. Mother had said, These are your options; the choice is D. Mother had said, “These are your options; the choice is E. Mother said, “These are your options; the choice is

13. Porcupine is from Latin porcus, “pig,” and spina, “spine.”

A. porcus, “pig,” and spina, “spine.” B. Porcus-pig and spina, “spine.” C. Porcus-pig, and Spina, “spine.” D. Porcus-Pig-,Spina-spine. E. Porcus, “pig,” and spina “spine”.

14. Seeing the dolphins, some sharks, a killer whale, and a Moray eel made the visit to the marine park worthwhile.

A. a killer whale, and a Moray eel made the visit B. a killer whale, and a moray eel made the visit C. a killer whale and a moray eel makes the visit D. a killer whale and a Moray eel makes the visit E. a killer whale and a moray eel made the visit

15. Still, the fact that a planet exists outside our solar system encourages hope that other solar systems exist, and in them, perhaps, a planet that does support life.

A. that a planet exists outside our solar system encourages hope that other solar systems exist, and B. that a Planet exists out side our solar system encourages hope that other solar systems exist and C. could be that a planet exists outside our solar system encourages hope that other solar systems exist, and D. that a planet exist outside our solar systems encourage hope that other solar systems exist, and E. that a planet does exists out side our solar system encourages hope that other solar systems exist, and

16. Mail-order shopping can be convenient and timesaving with appropriate precautions, it is safe as well.

A. can be convenient and timesaving B. can be convenient and timesaving; C. should be convenient and time saving; D. could be convenient and time saving; E. can be convenient and time-saving;

17. Among the many fields of science, no matter what turns you on, there are several fields of study.

A. science, no matter what turns you on, B. Science, no matter what turns you on, C. Science, no matter which you chose, D. Science, no matter which of these you chose- E. science, no matter which you choose,

Page 6: TABE PERT FCAT Reading and Writing Practice    MmAcVcErReIaCKS

18. The fact that boxing is known to cause head injuries and brain damage should lead us to inform the public and push for a ban on boxing.

A. should lead us to inform B. could lead us to inform C. should of led us to inform D. will lead us to inform E. should have led us to inform,

19. The first part of the test was on chemistry, the second on mathematics, and the third on english.

A. on mathematics, and the third on english. B. on mathematics; and the third on English. C. on Mathematics; and the third on English. D. on mathematics, and the third on English. E. on mathematics: and the third on English.

20. The Diary of Anne Frank  showed a young girl's courage  during two years of hiding.

A. showed a young girl's courage B. shows a young girl's courage C. did show a young girls courage D. has shown a young girl's courage E. showed a young girl's courage

21. In August my parents will be married for twenty-five years.

A. will be married for twenty-five years. B. shall have been married for twenty-five years. C. will have been married for twenty-five years. D. will be married for twenty five years. E. will have married for twenty-five years.

Answer Key 1. D 2. E 3. E 4. C 5. A 6. A 7. B 8. C 9. B 10. D 11. D 12. E 13. A 14. B 15. A 16. E 17. E 18. A 19. D 20. B 21. C 

1. The word boycott derives from the name of Charles C. Boycott, an English land agent in Ireland that was ostracized for refusing to reduce rent.

A. that was ostracized for refusing B. who was ostracized for refusing C. which was ostracized for refusing D. that had been ostracized for refusing E. who had been ostracized for refusing

2. As a result of his method for early music education, Shinichi Suzuki has been known as one of the world's great violin teachers.

A. has been known as one B. had been known as one C. is seen as one D. is being seen as one E. has been one

3. Last night the weather forecaster announced that this is the most rainy season the area has had in the past decade.

A. this is the most rainy season the B. this has been the most rainy season the C. this was the most rainy season the D. this is noted as the most rainy season the E. this is the rainiest season the

4. Although Mandy is younger than her sister, Mandy is the tallest of the   two.

Page 7: TABE PERT FCAT Reading and Writing Practice    MmAcVcErReIaCKS

A. is the tallest of the B. is the taller of the C. has been the taller of the D. is the most tall of the E. is the more taller of the

5. When Katherine Hepburn's play came to town, all the tickets had sold out far in advance.

A. had sold out far B. have sold out far C. were sold out far D. had been sold out far E. had been sold out for

6. The origins of most sports is unknown.

A. sports is unknown B. sports have been unknown C. sports are unknown D. sports has been unknown E. sports are now unknown

7. Neither of the Smith brothers expect to be drafted by a major league team this year.

A. expect to be drafted B. expects to be drafted C. has expected to be drafted D. is expecting to be drafted E. was expecting to be drafted

8. Has any of the witnesses been sworn in yet?

A. Has any of the B. Is any of the C. Will any of the D. Are any of the E. Have any of the

9. The Lusitania  sunk   on May 7, 1915.

A. sunk B. did sink C. was sunk D. did sank E. sank

10. Whos in the office now?

A. Whos in B. Whose in C. Who is in D. Who's in E. Whose' in

11. There are now many kinds of dictionaries, such as a dictionary of synonyms and antonyms, a biographical dictionary, and a geographical dictionary with pronunciations given.

A. with pronunciations given B. that has pronunciations given C. with pronunciations' given D. that have pronunciations given E. that do have pronunciations given

12. Towering seven hundred feet above the valley floor, Mount Rushmore National Memorial was an impressive site.

A. was an impressive site B. is a impressive sight C. is an impressive sight D. was an impressive sight E. is an impressive site

13. San Francisco lays southwest of Sacramento.

A. lays southwest B. has laid southwest C. is lying southwest D. lain southwest E. lies southwest

Page 8: TABE PERT FCAT Reading and Writing Practice    MmAcVcErReIaCKS

14. Did they know that Labor Day always came on the first Monday in September?

A. came on B. comes on C. has come on D. had come on E. has came on

15. Eating, drinking, and to stay up late at night were among her pleasures.

A. to stay up late B. to remain up late C. staying up late D. she liked staying up late E. trying to stay up late

16. Each night when night came and the temperature fell, my parents lit the fire in the bedroom.

A. and the temperature fell, B. and that the temperature did fall C. and that the temperature fell D. and because the temperature fell E. and when the temperature fell

17. Francis promised to bring the Papago basket that she bought in   Arizona.

A. bought in B. had bought in C. has bought in D. did buy in E. purchased in

18. He has lain   his racquetball glove on the beach.

A. has lain B. has laid C. have lain D. have laid E. is lying

19. I would have lent you my notes if you would have asked   me.

A. would have asked me B. could of asked C. could ask D. had asked E. had of asked

20. Many scientists are still hoping to have found life on another planet.

A. to have found B. to find C. two find D. to have been found E. too have found

21. Because she had an astounding memory, Sue has never forgotten an important equation.

A. had an B. could have had C. has D. did have E. has had

Answer Key 1. B 2. C 3. E 4. B 5. D 6. C 7. B 8. E 9. E 10. D 11. A 12. C 13. E 14. B 15. C 16. E 17. B 

Page 9: TABE PERT FCAT Reading and Writing Practice    MmAcVcErReIaCKS

18. B 19. D 20. B 21. C 

1. David was known for belching; and telling inappropriate jokes in public.

A. Capitalization B. Punctuation C. Spelling D. Grammar

2. Graduation from High School is considered by many a momentous occasion.

A. Capitalization B. Punctuation C. Spelling D. Grammar

3. Nurses plays a vital role in the healthcare profession.

A. Capitalization B. Punctuation C. Spelling D. Grammar

4. After having his tonsels removed, the child was listless for a few days.

A. Capitalization B. Punctuation C. Spelling D. Grammar

5. The park was serine at twilight.

A. Capitalization B. Punctuation C. Spelling D. Grammar

6. Was the patient's mind lucid during the evaluation.

A. Capitalization B. Punctuation C. Spelling D. Grammar

7. The bachalor never married. Most people thought it was because of misogyny.

A. Capitalization B. Punctuation C. Spelling D. Grammar

8. The intricacy of the mathematical equation, drove the student crazy trying to solve it.

A. Capitalization B. Punctuation C. Spelling D. Grammar

9. The hybrid tomatoes is immune to most common diseases.

A. Capitalization B. Punctuation C. Spelling D. Grammar

10. The professor was humiliated when his students reported him to the Dean for verbal abuse.

A. Capitalization B. Punctuation C. Spelling D. Grammar

11. The con artist hoodwinked the old lady when he sold her fradulent insurance.

Page 10: TABE PERT FCAT Reading and Writing Practice    MmAcVcErReIaCKS

A. Capitalization B. Punctuation C. Spelling D. Grammar

12. The movie star was accused of a misdemeanor, when she stole 15 dollars worth of merchandise from the store.

A. Capitalization B. Punctuation C. Spelling D. Grammar

13. The congregation sang a comtemporary hymn.

A. Capitalization B. Punctuation C. Spelling D. Grammar

14. The wound were necrotic when examined.

A. Capitalization B. Punctuation C. Spelling D. Grammar

15. The defendint exhibited a peevish appearance.

A. Capitalization B. Punctuation C. Spelling D. Grammar

16. The band director was scheduled to play the piccolo on tuesday.

A. Capitalization B. Punctuation C. Spelling D. Grammar

17. The renter was remiss; about the rent.

A. Capitalization B. Punctuation C. Spelling D. Grammar

18. The old man was know for sapient knowledge.

A. Capitalization B. Punctuation C. Spelling D. Grammar

19. The inventor create several specious ideas to solve the problem.

A. Capitalization B. Punctuation C. Spelling D. Grammar

20. The teacher identified the troublemakers, in her classroom.

A. Capitalization B. Punctuation C. Spelling D. Grammar

Answer Key 1. B 2. A 3. D 4. C 5. C 6. B 7. C 8. B 9. D 10. A 

Page 11: TABE PERT FCAT Reading and Writing Practice    MmAcVcErReIaCKS

11. C 12. B 13. C 14. D 15. C 16. A 17. B 18. D 19. D 20. B 

Author's Purpose Practice Questions

Read the passage below and answer question 1.

ASTHMA

About 17 million children and adults in the United States suffer from asthma, a condition that makes it hard to breathe. Today it is a problem that is treatable with modern medicine. In days gone by, there were many different superstitions about how to cure asthma. Some people thought that eating crickets with a little wine would help. Eating raw cat's meat might be the cure. Another idea was to gather some spiders' webs, roll them into a ball, and then swallow them. People also thought that if you ate a diet of only boiled carrots for two weeks, your asthma might go away. This carrot diet may actually have done some good for asthma patients, since vitamin A in carrots is good for the lungs.

1. The main purpose of the passage is to:

a. Describe herbal remediesb. Explain some of the measures for treating asthma from long agoc. Define superstitionsd. Extol the virtues of modern medicinee. Explain why asthma came about

Read the passage below and answer question 2.

BLACK HISTORY MONTH

Black History Month is unnecessary. In a place and time in which we overwhelmingly elected an African-American president, we can and should move to a postracial approach to education. As Detroit Free Press columnist Rochelle Riley wrote in a February 1 column calling for an end to Black History Month, "I propose that, for the first time in American history, this country has reached a point where we can stop celebrating separately, stop learning separately, stop being American separately."

In addition to being unnecessary, the idea that African-American history should be focused on in a given month suggests that it belongs in that month alone. Instead it is important to incorporate African-American history into what is taught every day as American history. It needs to be recreated as part of mainstream thought and not as an optional, often irrelevant, side note. We should focus efforts on pushing schools to diversify and broaden their curricula.

There are a number of other reasons to abolish it. First of all, it has become a shallow commercial ritual that does not even succeed in its (limited and misguided) goal of focusing for one month on a sophisticated, intelligent appraisal of the contributions and experiences of African-Americans throughout history. Second, there is a paternalistic flavor to the mandated bestowing of a month in which to study African-American history that is overcome if we instead assert the need for a comprehensive curriculum. Third, the idea of Black History Month suggests that the knowledge imparted in that month is for African-Americans only, rather than for all people.

2. The author's primary purpose in Passage 2 is to:

a. Argue that Black History Month should not be so commercialb. Argue that Black History Month should be abolishedc. Argue that Black History Month should be maintained

Page 12: TABE PERT FCAT Reading and Writing Practice    MmAcVcErReIaCKS

d. Suggest that African-American history should be taught in two months rather than just onee. Argue that African-American history is not part of mainstream curriculum

Read the passage below and answer question 3.

PARK WILDLIFE

Sequoia and Kings Canyon National Parks support a wide diversity of animal species, reflecting the range in elevation, climate, and habitat variety there. Over 260 native vertebrate species are in the parks; numerous additional species may be present but have not been confirmed. Of the native vertebrates, five species are extirpated (here meaning extinct), and over 150 are rare or uncommon. There have been some studies of invertebrates in the area, but there is not enough information to know how many species occur specifically in the parks. Many of the parks' caves contain invertebrates, some of which exist only in one cave and are known nowhere else in the world. Plant life in the foothills, where summers are hot and dry and winters are mild, is largely chaparral on the lower slopes, with blue oak and California buckeye in the valleys and on higher slopes. A number of animals live in this area year-round; some breed here, while others winter here. Local species include the gray fox, bobcat, striped and spotted skunks, black bear, wood rat, pocket gopher, white-footed mouse, California quail, scrub jay, lesser goldfinch, wrentit, acorn woodpecker, gopher snake, California king snake, striped racer, western whiptail lizard, and the California newt.

3. What was the author's purpose in writing this passage?

a. To entertain the readerb. To bore the readerc. To persuade the readerd. To inform the readere. To humor the reader

Read the passage below and answer question 4.

CALIFORNIA GRAPES

Grapes are one of the oldest cultivated fruits. Hieroglyphics show that Egyptians were involved in grape and wine production. Also, the early Romans were known to have developed many grape varieties.

Grapes have been grown in California for more than 200 years. The tradition of viticulture (growing grapes) began in 1769 when Spanish friars established missions throughout California. Then the boom in grapes planted for eating arose in the early 1800s. William Wolfskill, founder of California's citrus industry, planted the first table grape vineyard in 1839 near Los Angeles.

By the 1850s, the United States had officially acquired California from Mexico, and 80,000 gold prospectors had moved to the region. A few of these had the foresight to realize that there was money in grapes as well as in gold.

Today California wine, table grapes, and raisins are all important agricultural commodities, with approximately 700,000 acres planted in vineyards. About 85% of California's table grape production is in the southern San Joaquin Valley region, with the Coachella Valley region accounting for most of the remaining production.

4. The author most likely wrote this passage to:

a. Entertain the readerb. Persuade the reader c. Humor the readerd. Inform the readere. Sway the reader

Read the passage below and answer question 5.

VISUAL PERCEPTION

Page 13: TABE PERT FCAT Reading and Writing Practice    MmAcVcErReIaCKS

It is tempting to think that your eyes are simply mirrors that reflect whatever is in front of them. Researchers, however, have shown that your brain is constantly working to create the impression of a continuous, uninterrupted world.

For instance, in the last 10 minutes, you have blinked your eyes around 200 times. You have probably not been aware of any of these interruptions in your visual world. Something you probably have not seen in a long time without the aid of a mirror is your nose. It is always right there, down in the bottom corner of your vision, but your brain filters it out so that you are not aware of your nose unless you purposefully look at it.

Nor are you aware of the artery that runs right down the middle of your retina. It creates a large blind spot in your visual field, but you never notice the hole it leaves. To see this blind spot, try the following: Cover your left eye with your hand. With your right eye, look at the O on the left. As you move your head closer to the O, the X will disappear as it enters the blind spot caused by your optical nerve.

O X

Your brain works hard to make the world look continuous!

5. What is the main purpose of this passage?

a. To persuade the reader to pay close attention to blind spotsb. To explain the way visual perception worksc. To persuade the reader to consult an optometrist if the O and X disappeard. To prove that vision is a passive process

Read the passage below and answer question 6.

OPPOSITIONAL DEFIANT DISORDER

On a bad day, have you ever been irritable? Have you ever used a harsh tone or even been verbally disrespectful to your parents or teachers? Everyone has a short temper from time to time, but current statistics indicate that between 16% and 20% of a school's population suffer from a psychological condition known as oppositional defiant disorder, or ODD.

ODD symptoms include difficulty complying with adult requests, excessive arguments with adults, temper tantrums, difficulty accepting responsibility for actions, low frustration tolerance, and behaviors intended to annoy or upset adults. Parents of children with ODD often feel as though their whole relationship is based on conflict after conflict.

Unfortunately, ODD can be caused by a number of factors. Some students affected by ODD suffer abuse, neglect, and severe or unpredictable discipline at home. Others have parents with mood disorders or have experienced family violence. Various types of therapy are helpful in treating ODD, and some drugs can treat particular symptoms. However, no single cure exists.

The best advice from professionals is directed toward parents. Therapists encourage parents to avoid situations that usually end in power struggles, to try not to feed into oppositional behavior by reacting emotionally, to praise positive behaviors, and to discourage negative behaviors with timeouts instead of harsh discipline

6. The author's purpose in writing this passage is to:

a. Express frustration about ODDb. Prove that parents are the cause of ODDc. Inform the reader about this complex conditiond. Persuade the reader to keep students with ODD out of public school

Read the passage below and answer question 7.

EARLY POLITICAL PARTIES

The United States has always been a pluralistic society, meaning it has embraced many points of view and many groups with different identities from its beginning. That is not to say that these groups have always seen eye to eye. The first political parties developed in the United States as a result of conflicting visions of the American identity. Many politicians believed that wealthy merchants and lawyers

Page 14: TABE PERT FCAT Reading and Writing Practice    MmAcVcErReIaCKS

represented the country's true identity, but many others saw it in the farmers and workers who formed the country's economic base.

The event that brought this disagreement to the surface was the creation of the Bank of the United States in 1791. The bank set out to rid the country of the debts it had accumulated during the American Revolution. Until then, each state was responsible for its own debts. The Bank of the United States, however, wanted to assume these debts and pay them off itself. While many people considered this offer to be a good financial deal for the states, many states were uncomfortable with the arrangement because they saw it as a power play by the federal government. If a central bank had control over the finances of individual states, the people who owned the bank would profit from the states in the future. This concern was the basis of the disagreement: Who should have more power: the individual states or the central government?

The Democratic-Republican Party developed to protest the bank, but it came to represent a vision of America with power spread among states. The Federalist Party was established in defense of the bank, but its ultimate vision was of a strong central government that could help steer the United States toward a more competitive position in the world economy. These different points of view-central government versus separate states-would not be resolved easily. These same disagreements fueled the tension that erupted into the Civil War over half a century later.

7. What is the author's purpose in writing this passage?

a. To persuade the reader to accept the Federalist Party's point of viewb. To explain the disagreements between early American political partiesc. To explain the importance of a strong central governmentd. To criticize the founders of the Bank of the United States

Read the passage below and answer question 8.

PERIMENOPAUSE

For most women, the onset of menopause, the period of life when they no longer experience menstrual periods, is gradual. Perimenopause is the time leading up to menopause, an interval when menopause-related changes begin to be noticed. This is the lengthy period during which the ovaries progressively reduce production of the hormones estrogen and progesterone, and a woman's ability to become pregnant is lost. Perimenopause is a normal event in the process of aging, and indicates that the reproductive years are coming to an end.

Most women experience the first signs of perimenopause around the age of forty. The first signs likely to be noticed are menopause-related symptoms, such as changes in periods. Menopause itself is said to occur on the day a period has not been experienced for 12 months in a row. Although it is sometimes referred to as "premenopause," perimenopause is also often considered to include the year after menopause occurs.

During the onset of menopause, the body's output of several key hormones fluctuates and begins to shut down. Because hormones influence a wide spectrum of body functions, women may experience a variety of symptoms during this process. The symptoms of perimenopause are both physical and emotional and often begin years before a woman's period actually ceases. The cessation of ovulation known as menopause isn't a process that happens overnight; over a protracted period of several years, a woman's body undergoes a progression of changes.

Symptoms

Many of the symptoms a woman may experience during menopause itself have their onset during the time of perimenopause. Most symptoms can be managed through a combination of diet and exercise. Most women are not immediately aware of the changes their bodies are going through as a result of these hormonal fluctuations and will become aware of changes in the schedule of their periods as the first indication that menopause is at hand. Symptoms experienced during perimenopause include headaches, depression and anxiety, hot flashes and night sweats, insomnia and fatigue, urinary problems and vaginal dryness, weight gain, heart palpitations, and loss of bone mass. Women who have experienced excessive premenstrual syndrome (PMS) symptoms in the past are also likely to experience severe symptoms during perimenopause.

Onset and Duration

Perimenopause usually takes place between the ages of 45 and 55. In some women, the onset may occur much earlier. The duration of the change is quite variable from woman to woman and may last for as long as fifteen years, although ten years is typical. The timing and progression of the changes varies between individuals, depending on a number of factors including genetics and family history. A woman whose mother or grandmother experienced perimenopause early is likely to do so as well. There is also evidence linking an early onset of perimenopause to factors such as smoking, early childhood cancer, hysterectomy, and women who have not borne children.

Treatment

The symptoms of perimenopause are a normal manifestation of the aging process and do not ordinarily require treatment. Some physicians may choose to monitor the levels of certain hormones-particularly thyroid function-if the symptoms are severe. Under normal circumstances, this is unnecessary. However, if bleeding is extremely heavy, lasts longer than a week, or occurs between periods, a medical professional should be consulted. Symptoms such as these may indicate a more serious underlying gynecological problem.

8. The purpose of this article is to:

a. Describe the treatment of perimenopauseb. Enumerate the symptoms that women may experience when going through perimenopause

Page 15: TABE PERT FCAT Reading and Writing Practice    MmAcVcErReIaCKS

c. Give an overall description of perimenopaused. Compare perimenopause to menopause itself

Read the passage below and answer questions 9-10.

DISCUSSION OF NATIVE SPEAKER, BY CHANG-RAE LEE

Chang-Rae Lee's award-winning debut novel Native Speaker is about Henry Park, a Korean-American individual who struggles to find his place as an immigrant in a suburb of New York City. This novel addresses the notion that as the individuals who know us best, our family, peers, and lovers are the individuals who direct our lives and end up defining us. Henry Park is confronted with this reality in the very beginning of the novel, which opens:

The day my wife left she gave me a list of who I was.

Upon separating from his wife, Park struggles with racial and ethnic identity issues due to his loneliness. Through Parks' work as an undercover operative for a private intelligence agency, the author presents the theme of espionage as metaphor for the internal divide that Park experiences as an immigrant. This dual reality creates two worlds for Park and increases his sense of uncertainty with regard to his place in society. While he constantly feels like an outsider looking in, he also feels like he belongs to neither world.

Chang-Rae Lee is also a first-generation Korean American immigrant. He immigrated to America at the early age of three. Themes of identity, race, and cultural alienation pervade his works. His interests in these themes no doubt stem from his firsthand experience as a kid growing up in a Korean household while going to an American school. Lee is also author of A Gesture Life and Aloft. The protagonists are similar in that they deal with labels placed on them based on race, color, and language. Consequently, all of these characters struggle to belong in America.

Lee's novels address differences within a nation's mix of race, religion, and history, and the necessity of assimilation between cultures. In his works and through his characters, Lee shows us both the difficulties and the subtleties of the immigrant experience in America. He urges us to consider the role of borders, as well as why the idea of opening up one's borders is so frightening. In an ever-changing world in which cultures are becoming increasingly intermingled, the meaning of identity must be constantly redefined, especially when the security of belonging to a place is becoming more elusive. As our world grows smaller with increasing technological advances, these themes in Lee's novels become even more pertinent.

9. Which of the following best describes the purpose of this passage?

a. To criticizeb. To analyze c. To entertaind. To inform

10. Why does the author of the passage quote the first line of the novel Native Speaker?

a. To illustrate one of the themes in the novel b. To show how the book is semi-autobiographicalc. It is the main idea of the novel.d. To create interest in the novel

Answers and Explanations

1. B: The purpose of the passage is to describe different measures that people took for asthma long ago, before the advent of modern medicine. Answer choice A, herbal remedies, is incorrect because the majority of the "medicine" described in the passage is not herbal. The passage does not, as in answer choice C, define superstitions. Nor does it praise modern medicine, as answer choice D suggests.

2. B: The entire passage makes the argument that Black History Month should be abolished, offering various reasons why this is the best course of action, as in answer choice B. Each of the other answer choices offers a method of changing or maintaining the existing celebration of Black History Month, rather than abolishing it. They are, therefore, incorrect.

3. D: Since the author structures the passage using main idea and detail and gives many facts, you can determine the purpose of this passage is to inform the reader, as in answer choice D. Even if you did not find this passage to be entertaining, as in answer choice A, it is unlikely that the author would take the time to write a piece with the intent to bore the readers, so B can be easily eliminated. Nor is there any attempt within the passage to make an argument for any particular position and, thus, persuade the reader of a certain viewpoint; therefore, option C also does not apply.

4. D: Since the author structures the passage using chronological order and gives many facts and details, you can quickly determine that the purpose of this passage is to inform the reader, as in answer choice D. Even if you did not find this passage to be entertaining, as in answer choice A, it is unlikely that the author would take the time to write a piece with the intent to bore the readers, so B can be easily eliminated. Nor is there any attempt within the passage to make an argument for any particular position and, thus, persuade the reader of a certain viewpoint; therefore, option C also does not apply.

5. B: The passage explains the way that visual perception works. Choice B is, therefore, the best answer. The author does not attempt to persuade the reader or prove a particular viewpoint; therefore options A, C, and D are incorrect.

6. C: This passage explores numerous facets of ODD and is meant to inform the reader about this psychological condition. Choice C is the best choice. Although parental frustration is noted, it is not the primary focus; therefore, option A is incorrect. Likewise, despite the fact that parental behavior as a contributor is mentioned, parents are not overtly blamed for ODD. Answer choice B is, therefore, not the correct choice. Option D does note that a relatively high percentage of students have ODD, but this passage nowhere implies that they should be kept out of school because of it, so D should also be eliminated.

Page 16: TABE PERT FCAT Reading and Writing Practice    MmAcVcErReIaCKS

7. B: This passage does not choose one point of view on the issue, so only choice B is in keeping with the passage's purpose, which is to explain the disagreements between the earliest political parties in the US. All other answer choices would need to reflect the author's preference for a particular position in order to be valid options.

8. C: While the article does describe some treatments, as in option A, it points out that these are seldom necessary. And although it does enumerate symptoms, as with B, this is only a portion of the overall purpose of the article, which is broader than a listing of symptoms. The passage also does not delve into a lengthy comparison of perimenopause and menopause, eliminating option D. This article clearly encompasses a general description of the condition, making C the correct choice.

9. B: The passage neither criticizes (A) nor entertains (C), so these two options may be quickly removed from the list of viable choices. It does provide information, as in option D; however, the writer goes beyond straightforward presentation of facts into analysis of the details and underlying meaning. It explores the "why." This piece was written to analyze the works by Chang-Rae Lee and the themes presented in his most famous novels. Answer choice B most clearly expresses this purpose.

10. A: The author of this passage uses the first line of the novel to provide an example of one of the themes of the novel. By showing a direct example, the writer is illustrating the theme-option A-and goes on to discuss that very point in the passage. Although the piece does express certain parallels between Lee's experience as a Korean-American and the character's, it's nowhere suggested that the novel is semi-autobiographical, rendering B incorrect. The main idea of the novel (C) and any suggestion that the reader of the passage should also read the novel (D) are nowhere stated, so these two answer choices are also incorrect. 

============

Reading Comprehension Practice Questions

1. Questions 1-7.

In the sixteenth century, an age of great marine and terrestrial exploration, Ferdinand Magellan led the first expedition to sail around the world. As a young Portuguese noble, he served the king of Portugal, but he became involved in the quagmire of political intrigue at court and lost the king's favor. After he was dismissed from service to the king of Portugal, he offered to serve the future Emperor Charles V of Spain.

A papal decree of 1493 had assigned all land in the New World west of 50 degrees W longitude to Spain and all the land east of that line to Portugal. Magellan offered to prove that the East Indies fell under Spanish authority. On September 20, 1519, Magellan set sail from Spain with five ships. More than a year later, one of these ships was exploring the topography of South America in search of a water route across the continent. This ship sank, but the remaining four ships searched along the southern peninsula of South America. Finally they found the passage they sought near a latitude of 50 degrees S. Magellan named this passage the Strait of All Saints, but today we know it as the Strait of Magellan.

One ship deserted while in this passage and returned to Spain, so fewer sailors were privileged to gaze at that first panorama of the Pacific Ocean. Those who remained crossed the meridian we now call the International Date Line in the early spring of 1521 after ninety-eight days on the Pacific Ocean. During those long days at sea, many of Magellan's men died of starvation and disease.

Later Magellan became involved in an insular conflict in the Philippines and was killed in a tribal battle. Only one ship and seventeen sailors under the command of the Basque navigator Elcano survived to complete the westward journey to Spain and thus prove once and for all that the world is round, with no precipice at the edge.

The sixteenth century was an age of great ___exploration.

A. cosmic B. land C. mental D. common man E. none of the above

2. Magellan lost the favor of the king of Portugal when he became involved in a political ___.

A. entanglement B. discussion C. negotiation D. problems E. none of the above

3. The Pope divided New World lands between Spain and Portugal according to their location on one side or the other of an imaginary geographical line 50 degrees west of Greenwich that extends in a ___ direction.

A. north and south B. crosswise C. easterly D. south east E. north and west

4. One of Magellan's ships explored the ___ of South America for a passage across the continent.

A. coastline B. mountain range C. physical features D. islands E. none of the above

5. Four of the ships sought a passage along a southern ___.

Page 17: TABE PERT FCAT Reading and Writing Practice    MmAcVcErReIaCKS

A. coast B. inland C. body of land with water on three sides D. border E. answer not available

6. The passage was found near 50 degrees S of ___.

A. Greenwich B. The equator C. Spain D. Portugal E. Madrid

7. In the spring of 1521, the ships crossed the ___ now called the International Date Line.

A. imaginary circle passing through the poles B. Imaginary line parallel to the equator C. area D. land mass E. answer not found in article

8. Questions 8-14 Marie Curie was one of the most accomplished scientists in history. Together with her husband, Pierre, she discovered radium, an element widely used for treating cancer, and studied uranium and other radioactive substances. Pierre and Marie's amicable collaboration later helped to unlock the secrets of the atom.

Marie was born in 1867 in Warsaw, Poland, where her father was a professor of physics. At the early age, she displayed a brilliant mind and a blithe personality. Her great exuberance for learning prompted her to continue with her studies after high school. She became disgruntled, however, when she learned that the university in Warsaw was closed to women. Determined to receive a higher education, she defiantly left Poland and in 1891 entered the Sorbonne, a French university, where she earned her master's degree and doctorate in physics.

Marie was fortunate to have studied at the Sorbonne with some of the greatest scientists of her day, one of whom was Pierre Curie. Marie and Pierre were married in 1895 and spent many productive years working together in the physics laboratory. A short time after they discovered radium, Pierre was killed by a horse-drawn wagon in 1906. Marie was stunned by this horrible misfortune and endured heartbreaking anguish. Despondently she recalled their close relationship and the joy that they had shared in scientific research. The fact that she had two young daughters to raise by herself greatly increased her distress.

Curie's feeling of desolation finally began to fade when she was asked to succeed her husband as a physics professor at the Sorbonne. She was the first woman to be given a professorship at the world-famous university. In 1911 she received the Nobel Prize in chemistry for isolating radium. Although Marie Curie eventually suffered a fatal illness from her long exposure to radium, she never became disillusioned about her work. Regardless of the consequences, she had dedicated herself to science and to revealing the mysteries of the physical world.

The Curies' ____ collaboration helped to unlock the secrets of the atom.

A. friendly B. competitive C. courteous D. industrious E. chemistry

9. Marie had a bright mind and a __personality.

A. strong B. lighthearted C. humorous D. strange E. envious

10. When she learned that she could not attend the university in Warsaw, she felt___.

A. hopeless B. annoyed C. depressed D. worried E. none of the above

11. Marie ___ by leaving Poland and traveling to France to enter the Sorbonne.

A. challenged authority B. showed intelligence C. behaved D. was distressed E. answer not available in article

12. _____she remembered their joy together.

A. Dejectedly B. Worried C. Tearfully D. Happily E. Sorrowfully

Page 18: TABE PERT FCAT Reading and Writing Practice    MmAcVcErReIaCKS

13. Her ____ began to fade when she returned to the Sorbonne to succeed her husband.

A. misfortune B. anger C. wretchedness D. disappointment E. ambition

14. Even though she became fatally ill from working with radium, Marie Curie was never ____.

A. troubled B. worried C. disappointed D. sorrowful E. disturbed

15. Questions 15-19.

Mount Vesuvius, a volcano located between the ancient Italian cities of Pompeii and Herculaneum, has received much attention because of its frequent and destructive eruptions. The most famous of these eruptions occurred in A. D. 79.

The volcano had been inactive for centuries. There was little warning of the coming eruption, although one account unearthed by archaeologists says that a hard rain and a strong wind had disturbed the celestial calm during the preceding night. Early the next morning, the volcano poured a huge river of molten rock down upon Herculaneum, completely burying the city and filling in the harbor with coagulated lava. Meanwhile, on the other side of the mountain, cinders, stone and ash rained down on Pompeii. Sparks from the burning ash ignited the combustible rooftops quickly. Large portions of the city were destroyed in the conflagration. Fire, however, was not the only cause of destruction. Poisonous sulphuric gases saturated the air. These heavy gases were not buoyant in the atmosphere and therefore sank toward the earth and suffocated people.

Over the years, excavations of Pompeii and Herculaneum have revealed a great deal about the behavior of the volcano. By analyzing data, much as a zoologist dissects a specimen animal, scientist have concluded that the eruption changed large portions of the area's geography. For instance, it turned the Sarno River from its course and raised the level of the beach along the Bay of Naples. Meteorologists studying these events have also concluded that Vesuvius caused a huge tidal wave that affected the world's climate. In addition to making these investigations, archaeologists have been able to study the skeletons of victims by using distilled water to wash away the volcanic ash. By strengthening the brittle bones with acrylic paint, scientists have been able to examine the skeletons and draw conclusions about the diet and habits of the residents. Finally, the excavations at both Pompeii and Herculaneum have yielded many examples of classical art, such as jewelry made of bronze, which is an alloy of copper and tin.

The eruption of Mount Vesuvius and its tragic consequences have provided us with a wealth of data about the effects that volcanoes can have on the surrounding area. Today volcanologists can locate and predict eruptions, saving lives and preventing the destruction of cities and cultures.

Herculaneum and its harbor were buried under ___lava.

A. liquid B. solid C. flowing D. gas E. answer not available

16. The poisonous gases were not ___ in the air.

A. able to float B. visible C. able to evaporate D. invisible E. able to condense

17. Scientists analyzed data about Vesuvius in the same way that a zoologist ___ a specimen.

A. describes in detail B. studies by cutting apart C. photographs D. chart E. answer not available

18. ____have concluded that the volcanic eruption caused a tidal wave.

A. Scientist who study oceans B. Scientist who study atmospheric conditions C. Scientist who study ash D. Scientist who study animal behavior E. Answer not available in article

19. Scientist have used ___water to wash away volcanic ash from the skeletons of victims.

A. bottled B. volcanic C. purified D. sea E. fountain

Page 19: TABE PERT FCAT Reading and Writing Practice    MmAcVcErReIaCKS

20. Questions 20-24.

Conflict had existed between Spain and England since the 1570s. England wanted a share of the wealth that Spain had been taking from the lands it had claimed in the Americas.

Elizabeth I, Queen of England, encouraged her staunch admiral of the navy, Sir Francis Drake, to raid Spanish ships and towns. Though these raids were on a small scale, Drake achieved dramatic success, adding gold and silver to England's treasury and diminishing Spain's omnipotence.

Religious differences also caused conflict between the two countries. Whereas Spain was Roman Catholic, most of England had become Protestant. King Philip II of Spain wanted to claim the throne and make England a Catholic country again. To satisfy his ambition and also to retaliate against England's theft of his gold and silver, King Philip began to build his fleet of warships, the Armada, in January 1586.

Philip intended his fleet to be indestructible. In addition to building new warships, he marshaled one hundred and thirty sailing vessels of all types and recruited more than nineteen thousand robust soldiers and eight thousand sailors. Although some of his ships lacked guns and others lacked ammunition, Philip was convinced that his Armada could withstand any battle with England.

The martial Armada set sail from Lisbon, Portugal, on May 9,1588, but bad weather forced it back to port. The voyage resumed on July 22 after the weather became more stable.

The Spanish fleet met the smaller, faster, and more maneuverable English ships in battle off the coast of Plymouth, England, first on July 31 and again on August 2. The two battles left Spain vulnerable, having lost several ships and with its ammunition depleted. On August 7, while the Armada lay at anchor on the French side of the Strait of Dover, England sent eight burning ships into the midst of the Spanish fleet to set it on fire. Blocked on one side, the Spanish ships could only drift away, their crews in panic and disorder. Before the Armada could regroup, the English attacked again on August 8.

Although the Spaniards made a valiant effort to fight back, the fleet suffered extensive damage. During the eight hours of battle, the Armada drifted perilously close to the rocky coastline. At the moment when it seemed that the Spanish ships would be driven onto the English shore, the wind shifted, and the Armada drifted out into the North Sea. The Spaniards recognized the superiority of the English fleet and returned home, defeated.

Sir Francis Drake added wealth to the treasury and diminished Spain's ____.

A. unlimited power B. unrestricted growth C. territory D. treaties E. answer not available in article

21. Philip recruited many ___soldiers and sailors.

A. warlike B. strong C. accomplished D. timid E. non experienced

22. The ____ Armada set sail on May 9, 1588.

A. complete B. warlike C. independent D. isolated E. answer not available

23. The two battles left the Spanish fleet ____.

A. open to change B. triumphant C. open to attack D. defeated E. discouraged

24. The Armada was ___ on one side.

A. closed off B. damaged C. alone D. circled E. answer not available in this article

25. Questions 25-29.

The victory of the small Greek democracy of Athens over the mighty Persian empire in 490 B. C. is one of the most famous events in history. Darius, king of the Persian empire, was furious because Athens had interceded for the other Greek city-states in revolt against Persian domination. In anger the king sent an enormous army to defeat Athens. He thought it would take drastic steps to pacify the rebellious part of the empire. Persia was ruled by one man.

In Athens, however, all citizens helped to rule. Ennobled by this participation, Athenians were prepared to die for their city-state. Perhaps this was the secret of the remarkable victory at Marathon, which freed them from Persian rule. On their way to Marathon, the Persians tried to fool some Greek city-states by claiming to have come in peace. The frightened citizens of Delos refused to believe this. Not wanting to abet the conquest of Greece, they fled from their city and did not return until the Persians had left. They were wise, for the Persians next conquered the city of Etria and captured its people.

Page 20: TABE PERT FCAT Reading and Writing Practice    MmAcVcErReIaCKS

Tiny Athens stood alone against Persia. The Athenian people went to their sanctuaries. There they prayed for deliverance. They asked their gods to expedite their victory. The Athenians refurbished their weapons and moved to the plain of Marathon, where their little band would meet the Persians. At the last moment, soldiers from Plataea reinforced the Athenian troops.

The Athenian army attacked, and Greek citizens fought bravely. The power of the mighty Persians was offset by the love that the Athenians had for their city. Athenians defeated the Persians in archery and hand combat. Greek soldiers seized Persian ships and burned them, and the Persians fled in terror. Herodotus, a famous historian, reports that 6400 Persians died, compared with only 192 Athenians.

Athens had ____the other Greek city-states against the Persians.

A. refused help to B. intervened on behalf of C. wanted to fight D. given orders for all to fight E. defeated

26. Darius took drastic steps to ___ the rebellious Athenians.

A. weaken B. destroy C. calm D. placate E. answer not available

27. Their participation___to the Athenians.

A. gave comfort B. gave honor C. gave strength D. gave fear E. gave hope

28. The people of Delos did not want to ___ the conquest of Greece.

A. end B. encourage C. think about D. daydream about E. answer not available

29. The Athenians were ___by some soldiers who arrived from Plataea.

A. welcomed B. strengthened C. held D. captured E. answer not available

30. Questions 30-32.

The Trojan War is one of the most famous wars in history. It is well known for the ten-year duration, for the heroism of a number of legendary characters, and for the Trojan horse. What may not be familiar, however, is the story of how the war began. According to Greek myth, the strife between the Trojans and the Greeks started at the wedding of Peleus, King of Thessaly, and Thetis, a sea nymph. All of the gods and goddesses had been invited to the wedding celebration in Troy except Eris, goddesses of discord. She had been omitted from the guest list because her presence always embroiled mortals and immortals alike in conflict.

To take revenge on those who had slighted her, Eris decided to cause a skirmish. Into the middle of the banquet hall, she threw a golden apple marked “for the most beautiful.” All of the goddesses began to haggle over who should possess it. The gods and goddesses reached a stalemate when the choice was narrowed to Hera, Athena, and Aphrodite. Someone was needed to settle the controversy by picking a winner. The job eventually fell to Paris, son of King Priam of Troy, who was said to be a good judge of beauty.

Paris did not have an easy job. Each goddess, eager to win the golden apple, tried aggressively to bribe him. 

“I'll grant you vast kingdoms to rule, “ promised Hera. “Vast kingdoms are nothing in comparison with my gift,” contradicted Athena. “Choose me and I'll see that you win victory and fame in war.” Aphrodite outdid her adversaries, however. She won the golden apple by offering Helen, Zeus' daughter and the most beautiful mortal, to Paris. Paris, anxious to claim Helen, set off for Sparta in Greece.

Although Paris learned that Helen was married, he accepted the hospitality of her husband, King Menelasu of Sparta, anyway. Therefore, Menelaus was outraged for a number of reasons when Paris departed, taking Helen and much of the king's wealth back to Troy. Menelaus collected his loyal forces and set sail for Troy to begin the war to reclaim Helen.

Eris was known for ___both mortals and immortals.

A. scheming against B. involving in conflict C. feeling hostile toward D. ignoring E. comforting

Page 21: TABE PERT FCAT Reading and Writing Practice    MmAcVcErReIaCKS

31. Each goddess tried ___to bribe Paris.

A. boldly B. effectively C. secretly D. carefully E. answer not stated

32. Athena ___ Hera, promising Paris victory and fame in war.

A. denied the statement of B. defeated C. agreed with D. restated the statement E. questioned the statement

33. Questions 33-37.

One of the most intriguing stories of the Russian Revolution concerns the identity of Anastasia, the youngest daughter of Czar Nicholas II. During his reign over Russia, the Czar had planned to revoke many of the harsh laws established by previous czars. Some workers and peasants, however, clamored for more rapid social reform. In 1918 a group of these people, known as Bolsheviks, overthrew the government. On July 17 or 18, they murdered the Czar and what was thought to be his entire family.

Although witnesses vouched that all the members of the Czar's family had been executed, there were rumors suggesting that Anastasia had survived. Over the years, a number of women claimed to be Grand Duchess Anastasia. Perhaps the best –known claimant was Anastasia Tschaikovsky, who was also known as Anna Anderson.

In 1920, eighteen months after the Czar's execution, this terrified young woman was rescued from drowning in a Berlin river. She spent two years in a hospital, where she attempted to reclaim her health and shattered mind. The doctors and nurses thought that she resembled Anastasia and questioned heer about her background. She disclaimed any connection with the Czar's family.

Eight years later, though, she claimed that she was Anastasia. She said that she had been rescued by two Russian soldiers after the Czar and the rest of her family had been killed. Two brothers named Tschaikovsky had carried her into Romania. She had married one of the brothers, who had taken her to Berlin and left her there, penniless and without a vocation. Unable to invoke the aid of her mother's family in Germany, she had tried to drown herself.

During the next few years, scores of the Czar's relatives, ex-servants, and acquaintances interviewed her. Many of these people said that her looks and mannerisms were evocative of the Anastasia that they had known. Her grandmother and other relatives denied that she was the real Anastasia, however. Tried of being accused of fraud, Anastasia immigrated to the United States in 1928 and took the name Anna Anderson. She still wished to prove that she was Anastasia, though, and returned to Germany in 1933 to bring suit against her mother's family. There she declaimed to the court, asserting that she was indeed Anastasia and deserved her inheritance.

In 1957, the court decided that it could neither confirm nor deny Anastasia's identity. Although we will probably never know whether this woman was the Grand Duchess Anastasia, her search to establish her identity has been the subject of numerous books, plays, and movies.

Some Russian peasants and workers___for social reform.

A. longed B. cried out C. begged D. hoped E. thought much

34. Witnesses ___ that all members of the Czar's family had been executed.

A. gave assurance B. thought C. hoped D. convinced some E. answer not stated

35. Tschaikovsky ____any connection with the Czar's family.

A. denied B. stopped C. noted D. justified E. answer not stated

36. She was unable to ___the aid of her relative.

A. locate B. speak about C. call upon D. identify E. know

37. In court she ___ maintaining that she was Anastasia and deserved her inheritance.

A. finally appeared B. spoke forcefully 

Page 22: TABE PERT FCAT Reading and Writing Practice    MmAcVcErReIaCKS

C. testified D. gave evidence E. answer not stated

38. Questions 38-39.

King Louis XVI and Queen Marie Antoinette ruled France from 1774 to 1789, a time when the country was fighting bankruptcy. The royal couple did not let France's insecure financial situation limit their immoderate spending, however. Even though the minister of finance repeatedly warned the king and queen against wasting money, they continued to spend great fortunes on their personal pleasure. This lavish spending greatly enraged the people of France. They felt that the royal couple bought its luxurious lifestyle at the poor people's expense.

Marie Antoinette, the beautiful but exceedingly impractical queen, seemed uncaring about her subjects; misery. While French citizens begged for lower taxes, the queen embellished her palace with extravagant works of art. She also surrounded herself with artists, writers, and musicians, who encouraged the queen to spend money even more profusely.

While the queen's favorites glutted themselves on huge feasts at the royal table, many people in France were starving. The French government taxed the citizens outrageously. These high taxes paid for the entertainments the queen and her court so enjoyed. When the minister of finance tried to stop these royal spendthrifts, the queen replaced him. The intense hatred that the people felt for Louis XVI and Marie Antoinette kept building until it led to the French Revolution. During this time of struggle and violence (1789-1799), thousands of aristocrats, as well as the king and queen themselves, lost their lives at the guillotine. Perhaps if Louis XVI and Marie Antoinette had reined in their extravagant spending, the events that rocked France would not have occurred.

The people surrounding the queen encouraged her to spend money ____.

A. wisely B. abundantly C. carefully D. foolishly E. joyfully

39. The minister of finance tried to curb these royal ___.

A. aristocrats B. money wasters C. enemies D. individuals E. spenders

40. Questions 40-45.

Many great inventions are greeted with ridicule and disbelief. The invention of the airplane was no exception. Although many people who heard about the first powered flight on December 17,1903, were excited and impressed, others reacted with peals of laughter. The idea of flying an aircraft was repulsive to some people. Such people called Wilbur and Orville Wright, the inventors of the first flying machine, impulsive fools. Negative reactions, however, did not stop the Wrights. Impelled by their desire to succeed, they continued their experiments in aviation.

Orville and Wilbur Wright had always had a compelling interest in aeronautics and mechanics. As young boys they earned money by making and selling kites and mechanical toys. Later, they designed a newspaper-folding machine, built a printing press, and operated a bicycle-repair shop. In 1896, when they read about the death of Otto Lilienthal, the brother's interest in flight grew into a compulsion.

Lilienthal, a pioneer in hang-gliding, had controlled his gliders by shifting his body in the desired direction. This idea was repellent to the Wright brothers, however, and they searched for more efficient methods to control the balance of airborne vehicles. In 1900 and 1901, the Wrights tested numerous gliders and developed control techniques. The brothers' inability to obtain enough lift power for the gliders almost led them to abandon their efforts.

After further study, the Wright brothers concluded that the published tables of air pressure on curved surfaces must be wrong. They set up a wind tunnel and began a series of experiments with model wings. Because of their efforts, the old tables were repealed in time and replaced by the first reliable figures for air pressure on curved surfaces. This work, in turn, made it possible for them to design a machine that would fly. In 1903 the Wrights built their first airplane, which cost less than one thousand dollars. They even designed and built their own source of propulsion- a lightweight gasoline engine. When they started the engine on December 17, the airplane pulsated wildly before taking off. The plane managed to stay aloft for twelve seconds, however, and it flew one hundred twenty feet.

By 1905 the Wrights had perfected the first airplane that could turn, circle, and remain airborne for half an hour at a time. Others had flown in balloons or in hang gliders, but the Wright brothers were the first to build a full-size machine that could fly under its own power. As the contributors of one of the most outstanding engineering achievements in history, the Wright brothers are accurately called the fathers of aviation.

The idea of flying an aircraft was ___to some people.

A. boring B. distasteful C. exciting D. needless E. answer not available

41. People thought that the Wright brothers had ____.

A. acted without thinking B. been negatively influenced C. been too cautious D. had not given enough thought E. acted in a negative way

42. The Wright's interest in flight grew into a ____.

Page 23: TABE PERT FCAT Reading and Writing Practice    MmAcVcErReIaCKS

A. financial empire B. plan C. need to act D. foolish thought E. answer not in article

43. Lilenthal's idea about controlling airborne vehicles was ___the Wrights.

A. proven wrong by B. opposite to the ideas of C. disliked by D. accepted by E. opposed by

44. The old tables were __ and replaced by the first reliable figures for air pressure on curved surfaces.

A. destroyed B. canceled C. multiplied D. discarded E. not used

45. The Wrights designed and built their own source of ____.

A. force for moving forward B. force for turning around C. turning D. force to going backward E. none of the above

Answer Key 1. B 2. A 3. A 4. C 5. C 6. B 7. A 8. A 9. B 10. B 11. A 12. A 13. C 14. C 15. B 16. A 17. B 18. B 19. C 20. A 21. B 22. B 23. C 24. A 25. B 26. C 27. B 28. B 29. B 30. B 31. A 32. A 33. B 34. A 35. A 36. C 37. B 38. B 39. B 40. B 41. A 42. C 43. C 44. B 45. A 

=========== Find the Main Idea

Page 24: TABE PERT FCAT Reading and Writing Practice    MmAcVcErReIaCKS

1. Americans have always been interested in their Presidents' wives. Many First Ladies have been remembered because of the ways they have influenced their husbands. Other First Ladies have made the history books on their own.

At least two First Ladies, Bess Truman and Lady Bird Johnson, made it their business to send signals during their husbands' speeches. When Lady Bird Johnson thought her husband was talking too long, she wrote a note and sent it up to the platform. It read, “It's time to stop!” And he did. Once Bess Truman didn't like what her husband was saying on television, so she phoned him and said,” If you can't talk more politely than that in public, you come right home.”

Abigail Fillmore and Eliza Johnson actually taught their husbands, Millard Fillmore and Andrew Johnson, the thirteenth and seventeenth Presidents. A schoolteacher, Abigail eventually married her pupil, Millard. When Eliza Johnson married Andrew, he could not read or write, so she taught him herself.

It was First Lady Helen Taft's idea to plant the famous cherry trees in Washington, D. C. Each spring these blossoming trees attract thousands of visitors to the nation's capital. Mrs. Taft also influenced the male members of her family and the White House staff in a strange way: she convinced them to shave off their beards!

Shortly after President Wilson suffered a stroke, Edith Wilson unofficially took over most of the duties of the Presidency until the end of her husband's term. Earlier, during World War I, Mrs. Wilson had had sheep brought onto the White House lawn to eat the grass. The sheep not only kept the lawn mowed but provided wool for an auction sponsored by the First Lady. Almost $100,000 was raised for the Red Cross.

Dolly Madison saw to it that a magnificent painting of George Washington was not destroyed during the War of 1812. As the British marched toward Washington, D. C., she remained behind to rescue the painting, even after the guards had left. The painting is the only object from the original White House that was not burned.

One of the most famous First Ladies was Eleanor Roosevelt, the wife of President Franklin D. Roosevelt. She was active in political and social causes throughout her husband's tenure in office. After his death, she became famous for her humanitarian work in the United Nations. She made life better for thousands of needy people around the world.

What is the main idea of this passage?

A. The Humanitarian work of the First Ladies is critical in American government. B. Dolly Madison was the most influential president's wife. C. Eleanor Roosevelt transformed the First Lady image. D. The First Ladies are important in American culture. E. The First Ladies are key supporters of the Presidents.

2. Of the many kinds of vegetables grown all over the world, which remains the favorite of young and old alike? Why, the potato, of course.

Perhaps you know them as “taters,” “spuds,” or “Kennebees,” or as “chips,” “Idahoes,” or even “shoestrings.” No matter, a potato by any other name is still a potato- the world's most widely grown vegetable. As a matter of fact, if you are an average potato eater, you will put away at least a hundred pounds of them each year.

That's only a tiny portion of the amount grown every year, however. Worldwide, the annual potato harvest is over six billion bags- each bag containing a hundred pounds of spuds, some of them as large as four pounds each. Here in the United States, farmers fill about four hundred million bags a year. That may seem like a lot of “taters,” but it leaves us a distant third among world potato growers. Polish farmers dig up just over 800 million bags a year, while the Russians lead the world with nearly 1.5 billion bags.

The first potatoes were grown by the Incas of South America, more than four hundred years ago. Their descendants in Ecuador and Chile continue to grow the vegetable as high as fourteen thousand feet up in the Andes Mountains. ( That's higher than any other food will grow.) Early Spanish and English explorers shipped potatoes to Europe, and they found their way to North America in the early 1600s.

People eat potatoes in many ways-baked, mashed, and roasted, to name just three. However, in the United States most potatoes are devoured in the form of French fries. One fast-food chain alone sells more than $1 billion worth of fries each year. No wonder, then, that the company pays particular attention to the way its fries are prepared.

Before any fry makes it to the people who eat at these popular restaurants, it must pass many separate tests. Fail any one and the spud is rejected. To start with, only russet Burbank potatoes are used. These Idaho potatoes have less water content than other kinds, which can have as much as eighty percent water. Once cut into “shoestrings” shapes, the potatoes are partly fried in a secret blend of oils, sprayed with liquid sugar to brown them, steam dried at high heat, then flash frozen for shipment to individual restaurants.

Before shipping, though, every shoestring is measured. Forty percent of a batch must be between two and three inches long. Another forty percent has to be over three inches. What about the twenty percent that are left in the batch? Well, a few short fries in a bag are okay, it seems.

So, now that you realize the enormous size and value of the potato crop, you can understand why most people agree that this part of the food industry is no “small potatoes.”

What is the main idea of this passage?

A. Potatoes from Ireland started the Potato Revolution. B. The average American eats 50 lbs of potatoes a year. C. French fries are made from potatoes. D. Potatoes are a key vegetable in America. E. The various terms for potatoes have a long history.

3. What does the word patent mean to you? Does it strike you as being something rather remote from your interests? If it does, stop and think a moment about some of the commonplace things that you use every day, objects that you take for granted as part of the world around you. The telephone, radio, television, the automobile, and the thousand and one other things (even the humble safety pin) that enrich our lives today once existed only as ideas in the minds of men. If it had not been possible to patent their ideas and thus protect them against copying by others, these inventions might never have been fully developed to serve mankind.

If there were no patent protection there would be little incentive to invent and innovate, for once the details of an invention became known, hordes of imitators who did not share the inventor's risks and expenses might well flood the market with their copies of his product and reap much of the benefit of his efforts. The technological progress that has made America great would wither rapidly under conditions such as these.

The fundamental principles in the U. S. patent structure came from England. During the glorious reign of Queen Elizabeth I in England, the expanding technology was furthered by the granting of exclusive manufacturing and selling privileges to citizens who had invented new processes or tools- a step that did much to encourage creativity.

Page 25: TABE PERT FCAT Reading and Writing Practice    MmAcVcErReIaCKS

Later, when critics argued that giving monopoly rights to one person infringed on the rights of others, an important principle was added to the patent structure: The Lord Chief Justice of England stated that society had everything to gain and nothing to lose by granting exclusive privileges to an inventor, because a patent for an invention was granted for something new that society never had before.

Another basic principle was brought into law because certain influential people in England had managed to obtain monopoly control over such age-old products as salt, and had begun charging as much as the traffic would bear. The public outcry became so great that the government was forced to decree that monopoly rights could be awarded only to those who created or introduced something really unique. These principles are the mainstays of our modern patent system in the United States.

In colonial times patent law was left up to the separate states. The inconsistency, confusion, and unfairness that resulted clearly indicated the need for a uniform patent law, and the men who drew up the Constitution incorporated one. George Washington signed the first patent law on April 10,1790, and less than four months later the first patent was issued to a man named Samuel Hopkins for a chemical process, an improved method of making potash for use in soapmaking.

In 1936 the Patent Office was established as a separate bureau. From the staff of eight that it maintained during its first year of operation it has grown into an organization of over 2500 people handling more than 1600 patent applications and granting over 1000 every week.

The Patent Office in Washington, D. C., is the world's largest library of scientific and technical data, and this treasure trove of information is open for public inspection. In addition to more than 3 million U. S. patents, it houses more than 7 million foreign patents and thousands of volumes of technical literature. Abraham Lincoln patented a device to lift steam vessels over river shoals, Mark Twain developed a self-pasting scrapbook, and millionaire Cornelius Vanderbilt invented a shoe-shine kit.

A patent may be granted for any new and useful process, machine, article of manufacture, or composition of matter ( a chemical compound or combinations of chemical compounds), or any distinct and new variety; of plant, including certain mutants and hybrids.

The patent system has also helped to boost the wages of the American worker to an unprecedented level; he can produce more and earn more with the computer, adding machines, drill press or lathe. Patented inventions also help keep prices down by increasing manufacturing efficiency and by stimulating the competition that is the foundation of our free enterprise system.

The decades of history have disclosed little need for modification of the patent structure. Our patent laws, like the Constitution from which they grew, have stood the test of time well. They encouraged the creative processes, brought untold benefits to society as a whole, and enabled American technology to outstrip that of the rest of the civilized world.

What is the main idea of this passage?

A. The patent system encourages free enterprise. B. The Constitution protects the patent system. C. The patent system in England has been influential in American patent development. D. Patents are important tools for inventors. E. Patented inventions protect the inventor, free enterprise, and the creative process.

4. Most people think it's fine to be “busy as a beaver.” Little do they know. Beavers may work hard, but often they don't get much done.

Beavers are supposed to be great tree cutters. It is true that a beaver can gnaw through a tree very quickly. (A six-inch birch takes about ten minutes.) But then what? Often the beaver does not make use of the tree. One expert says that beavers waste one out of every five trees they cut.

For one thing, they do not choose their trees wisely. One bunch of beavers cut down a cottonwood tree more than one hundred feet tall. Then they found that they could not move it.

In thick woods a tree sometimes won't fall down. It gets stuck in the other trees. Of course, doesn't think to cut down the trees that are in the way. So a good tree goes to waste.

Some people think that beavers can make a tree fall the way they want it to. Not true. (In fact, a beaver sometimes gets pinned under a falling tree.) When beavers cut a tree near a stream, it usually falls into the water. But they do not plan it that way. The fact is that most trees lean toward the water to start with.

Now what about dam building? Most beaver dams are wonders of engineering. The best ones are strongly built of trees, stones, and mud. They are wide at the bottom and narrow at the top.

Beavers think nothing of building a dam more than two hundred feet long. One dam, in Montana, was more than two thousand feet long. The largest one ever seen was in New Hampshire. It stretched four thousand feet. It made a lake large enough to hold forty beaver homes.

So beavers do build good dams. But they don't always build them in the right places. They just don't plan. They will build a dam across the widest part of the stream. They don't try to find a place where the stream is narrow. So a lot of their hard work is wasted.

Beavers should learn that it's not enough to be busy. You have to know what you're doing, too. For example, there was one Oregon beaver that really was a worker. It decided to fix a leak in a man-made dam. After five days of work it gave up. The leak it was trying to block was the lock that boats go through.

What is the main idea of this passage?

A. Beavers may be hard working animals, but they don't always choose the most efficient mechanisms. B. Beavers are excellent dam builders. C. New Hampshire was the site of the largest beaver dam. D. Beavers are well developed tree cutters. E. Beavers are poor surveyors of aquatic environments in some cases.

5. The raisin business in America was born by accident. It happened in 1873 in the San Joaquin Valley of California. Many farmers raised grapes in this valley. That year, just before the grape harvest, there was a heat wave. It was one of the worst heat waves ever known. It was so hot the grapes dried on the vines. When they were picked, California had its first raisin crop.

Page 26: TABE PERT FCAT Reading and Writing Practice    MmAcVcErReIaCKS

People were surprised to find how good raisins were. Everybody wanted more. So the San Joaquin farmers went into the raisin business. Today, of course, they do not let the grapes dry on the vines. They treat them with much more care.

In late August the grapes start to ripen. They are tested often for sweetness. The growers wait until the sugar content is twenty-one percent. Then they know the grapes are ripe enough to be picked.

Skilled workers come to the vineyards. They pick the bunches of grapes by hand. The workers fill their flat pans with grapes. They gently empty the pans onto squares of paper. These squares lie between the long rows of vines. They sit in the sun.

Here the grapes stay while the sun does its work. It may take two weeks or longer. The grapes are first dried on one side. When they have reached the right color, they are turned to dry on the other side. The grapes are dried until only fifteen percent of the moisture is left. Then they have turned into raisins.

The raisins are rolled up in the paper on which they have dried. Trucks take them from the fields. They are poured into big boxes called sweatboxes. Each box holds one hundred and sixty pounds of raisins. Here, any raisins that are a bit too dry take moisture from those that have a bit too much. After a while they are all just moist enough.

The big boxes are trucked next to the packaging plant. They are emptied onto a conveyor belt that shakes the raisins gently. This knocks them from their stems. A blast of air whisks the stems away. The water bath is next. Then the plump brown raisins have a last inspection. They are again checked for moisture and sugar. Then they go on a belt to packing machines. Here they are poured into packages, which are automatically weighed and sealed. The raisins are now ready for market.

What is the main idea of this passage?

A. The creation of raisins in America was an accident. B. The process of raisin development requires multiple steps. C. Raisins on the grocery store shelf undergo a brief fermentation process. D. Raisins are cleaned thoroughly at the packing plant. E. California has been the leader in American raisin development.

6. In 1976, Sichan Siv was crawling through the jungle, trying to escape from Cambodia. By 1989, however, Siv was working in the White House, in Washington D. C., as an advisor to the President of the United States. How did this strange journey come about?

Like millions of Cambodians, Siv was a victim of a bloody civil war. One of the sides in this war was the Cambodian government. The other was a group called the Khmer Rouge. When the Khmer Rouge won the war, the situation in Cambodia got worse. Many people were killed, while others were forced into hard labor. Sometimes entire families were wiped out.

Siv came from a large family that lived in the capital of Cambodia. After finishing high school, Siv worked for a while with a Cambodian airline company. Later, he taught English. After that, he took a job with CARE, an American group that was helping victims of the war.

Siv had hope to leave Cambodia before the Khmer Rouge took over the country. Unfortunately, he was delayed. As a result, he and his family were taken from their homes and forced to labor in rice fields. After a while, Siv managed to escape. He rode an old bicycle for miles, trying to reach Thailand where he would be free and safe. For three weeks he slept on the ground and tried to hide from the soldiers who were looking for him. Caught at last, he was afraid he would be killed. Instead, he was put into a labor camp, where he worked eighteen hours each day without rest. After several months, he escaped again; this time he made it. The journey, however, was a terrifying one. After three days of staggering on foot through mile after mile of thick bamboo, Siv finally made his way to Thailand.

Because he had worked for an American charity group, Siv quickly found work in a refugee camp. Soon he was on his way to the states. He arrived in June of 1976 and got a job-first picking apples and then cooking in a fast-food restaurant. Siv, however, wanted more than this; he wanted to work with people who, like himself, had suffered the hardship of leaving their own countries behind. Siv decided that the best way to prepare for this kind of work was to go to college. He wrote letters to many colleges and universities. They were impressed with his school records from Cambodia, and they were impressed with his bravery. Finally, in 1980, he was able to study at Columbia University in New York City. After finishing his studies at Columbia, Siv took a job with the United Nations. He married an American woman and became a citizen. After several more years, he felt that he was very much a part of his new country.

In 1988, Siv was offered a job in the White House working for President Reagan's closest advisors. It was a difficult job, and he often had to work long hours. However the long hard work was worth it, because Siv got the opportunity to help refugees in his work.

What is the main idea of this passage?

A. Persistence and courage are global ideas. B. Siv covered a large area during his life. C. Siv persevered to become an American citizen D. Siv overcame numerous challenges to come to American and help others. E. Siv persevered to become an American citizen.

7. When you want to hang the American flag over the middle of a street, suspend it vertically with the blue field, called the union, to the north and east-west street. When the flag is displayed with another banner from crossed staffs, the American flag is on the right. Place the staff of the American flag in front of the other staff. Raise the flag quickly and lower it slowly and respectfully. When flying the flag at half-mast, hoist it to the top of the pole for a moment before lowering it to mid-pole. When flying the American flag with banners from states or cities, raise the nation's banner first and lower it last. Never allow the flag to touch the ground.

What is the main idea of this passage?

A. The American flag is the symbol of American freedom. B. The American flag has fifty stars. C. Placing the American flag inappropriately will draw government intervention. D. American flag should be flown differently in certain situations. ","The flag should be lowered quickly and respectfully.

8. What if someone told you about a kind of grass that grows as tall as the tallest trees? A grass that can be made as strong as steel? A grass from which houses, furniture, boats, and hundreds of other useful things can be made? A grass that you would even enjoy eating? Would you believe that person? You should, for that grass is bamboo, the “wood” of 1,001 uses.

Page 27: TABE PERT FCAT Reading and Writing Practice    MmAcVcErReIaCKS

Bamboo may look like wood, but it is part of the family of plants that includes wheat, oats, and barley. It is a kind of grass. This grass is not just a material for making useful products. Young bamboo is eaten, often mixed with other vegetables, in many Asian foods.

Bamboo grows in many parts of the world. In the United States it grows in an area from Virginia west to Indiana and south to Florida, Louisiana, and Texas. Most bamboo, however, is found in warm, wet climates, especially in Asia and on the islands of the South Pacific Ocean.

In most Asian countries, bamboo is nearly as important as rice. Many Asians live in bamboo houses. They sit on bamboo chairs and sleep on bamboo mats. They fence their land with bamboo and use the wood for cages for chickens and pigs.

Bamboo is used to build large buildings as well as homes. When it is glued in layers, it becomes as strong as steel. On some islands in the South Pacific, bamboo is even used for water pipes. This extraordinary material has many other uses. It is used to make musical instruments, such as flutes and recorders. Paper made from bamboo has been highly prized by artists for thousands of years.

Bamboo is light and strong, and it bends without breaking. It is cheap, floats on water, almost never wears out, and is easy to grow. Nothing else on earth grows quite so fast as bamboo. At times you can even see it grow! Botanists have recorded growths of more than three feet in just twenty-four hours! Bamboo is hollow and has a strong root system that almost never stops growing and spreading. In fact, only after it flowers, an event that may happen only once every thirty years, will bamboo die.

There are more than a thousand kinds of bamboo. The smallest is only three inches tall and one-tenth of an inch across. The largest reaches more than two hundred feet in height and seven inches in diameter. No wonder, then, that the lives of nearly half the people on earth would change enormously if there were no longer any bamboo. No wonder, too, that to many people bamboo is a symbol of happiness and good fortune.

What is the main idea of this passage?

A. Bamboo has at least 2,000 uses. B. Bamboo grows at an amazing rate and is found primarily in Asia. C. Bamboo is an amazing grass that can be used in multiple ways. D. There are at least a 1,000 types of bamboo. E. Bamboo could be considered a flower in some cases.

9. Every year since 1986, some of the world's most daring runners have gathered in the desert of Morocco. They are there to take part in one of the most difficult races in the world. The Marathon of the Sands, as it is called, covers over 125 miles of desert and mountain wilderness. The runners complete the course in fewer than seven days, and they run with their food, clothing, and sleeping bags on their backs.

The Marathon of the Sands was founded in 1986 by Patrick Bauer. His idea was to give the runners, who come from all over the world, a special kind of adventure. Most of the runners in this race have found that they form deep friendships with the other runners during their days and nights in the desert. Facing terrible heat and complete exhaustion, they learn much about themselves and each other.

For most of the runners, though, the challenge of the race is the main reason for coming. On the first day, for example, they run fifteen miles across a desert of sand, rocks, and thorny bushes. Few runners finish the day without blistered and raw feet. They also suffer from a lack of water. (They are allowed less than nine quarts of water during each day of the race.) Most of all, they are exhausted when they arrive at the campsite for the night.

The second day, the runners are up at 6:00 A. M. Within a few hours, it is 100 degrees F, but the runners do not hesitate. They must cover eighteen miles that day. That night, they rest. They must be ready for the next day's run.

On the third day, the runners must climb giant sand dunes- the first they have faced. Dust and sand mix with the runners' sweat. Soon their faces are caked with mud. After fifteen miles of these conditions, the runners finally reach their next camp.

The race continues like this for four more days. The fourth and fifth days are the worst. On the fourth day, the runners pass through a level stretch and a beautiful, tree-filled oasis, but then, on this and on the next day, they cross more than twenty-one miles of rocks and sand dunes. The temperature soars to 125 degrees F, and many runners cannot make it. Helicopters rush fallen runners to medical help. Runners who make it to the end of the fifth day know that the worst is over.

On the sixth day, heat and rocks punish the racers terribly. In the Valley of Dra, the wind picks up and, as the desert heat is thrust against them with great force, they grow more and more exhausted.

The seventh day is the last, with only twelve miles to be covered. The dusty, tired, blistered runners set out at daybreak. Near the finish line, children race along with the runners, for everybody has caught the excitement. The ones who have run the whole marathon know they have accomplished what most people could not even dream of. “During the hard moments,” says one contestant who has raced here twice, “I'd think, ‘Why am I here?' Then I'd realize I was there to find my limits.”

What is the main idea of this passage?

A. The Marathon of the Sands race tests the limits of human endurance. B. The runners run at their own pace. C. The race causes the strong to stumble and the weak to not finish. D. The seventh day is the hardest day of the race. E. Every runner runs the race to find their human limits.

10. High in the Andes Mountains in Peru stands the ancient city of Machu Picchu. No one knows why this great city was built, nor is it likely that we will ever know. Nevertheless, the deserted city of Machu Picchu is important for what it reveals about the ancient Inca people of South America.

The Incas once ruled a great empire that covered a large part of the South American continent. The empire was more than five hundred years old when the first Spanish explorers, looking for gold, went to that continent in the sixteenth century.

The Incas were an advanced people. They were skillful engineers who paved their roads and built sturdy bridges. They plowed the land in such a way that rains would not wash away valuable soil. They dug ditches to carry water into dry areas for farming.

Page 28: TABE PERT FCAT Reading and Writing Practice    MmAcVcErReIaCKS

Even though they did not know about the wheel, the Incas were able to move huge stone blocks- some as heavy as ten tons- up the sides of mountains to build walls. The blocks were fitted so tightly, without cement of any kind, that it would be impossible to slip a knife blade between them! The walls have stood firm through great storms and earthquakes that have destroyed many modern buildings.

The Incas were great artists, too. Today, Incan dishes and other kinds of pottery are prized for their wonderful designs. Since both gold and silver were in great supply, the Incas created splendid objects from these precious metals.

While it is true that the Incas had no written language, they kept their accounts by using a system of knotted strings of various lengths and colors. The sizes of the knots and the distances between them represented numbers.

At its height, the Incan empire included as many as thirty million people. The emperor ruled them with an iron hand. He told his subjects where to live, what to plant, how long they should work-even whom they could marry. Since he owned everything, the emperor gave what he wished when he wished- and in the amount he wished -to his people.

In 1533 Spanish explorers led by Francisco Pizarro murdered the emperor of the Incas. Earlier, the heir to the Incan empire had also been killed. The Incas, who had always been entirely dependent on their emperor, now had no recognized leader. The Spaniards easily conquered the empire and plundered its riches.

Have the Incas disappeared from South America? Not at all. In Peru alone, once the center of that great empire, eighty percent of the twenty million people are descendants of the Inca people. Evidence of the Incan empire can be found in many other places in South America as well. You can even visit Machu Picchu. The remains of this ancient city still stand high in the mountains of Peru, an awesome tribute to this once powerful empire.

What is the main idea of this passage?

A. The Incas once inhabited the ancient city of Machu Picchu. B. Peru was the primary country of the Incas. C. The Incan empire can be found in ancient cities and was plundered by the Spanish. D. Spanish conquerors destroyed the Incan empire in the thirteenth century. E. Machu Picchu was the capital of the Incan empire.

Answer Key 1. D 2. D 3. E 4. A 5. B 6. D 7. D 8. C 9. A 10. C 

=============

==========

Advanced Reading for Understanding

1. In 1892 the Sierra Club was formed. In 1908 an area of coastal redwood trees north of San Francisco was established as Muir Woods National Monument. In the Sierra Nevada mountains, a walking trail from Yosemite Valley to Mount Whitney was dedicated in 1938. It is called John Muir Trail.

John Muir was born in 1838 in Scotland. His family name means “moor,” which is a meadow full of flowers and animals. John loved nature from the time he was small. He also liked to climb rocky cliffs and walls.

When John was eleven, his family moved to the United States and settled in Wisconsin. John was good with tools and soon became an inventor. He first invented a model of a sawmill. Later he invented an alarm clock that would cause the sleeping person to be tipped out of bed when the timer sounded.

Muir left home at an early age. He took a thousand-mile walk south to the Gulf of Mexico in 1867and 1868. Then he sailed for San Francisco. The city was too noisy and crowded for Muir, so he headed inland for the Sierra Nevadas.

When Muir discovered the Yosemite Valley in the Sierra Nevadas, it was as if he had come home. He loved the mountains, the wildlife, and the trees. He climbed the mountains and even climbed trees during thunderstorms in order to get closer to the wind. He put forth the theory in the late 1860's that the Yosemite Valley had been formed through the action of glaciers. People ridiculed him. Not until 1930 was Muir's theory proven correct.

Muir began to write articles about the Yosemite Valley to tell readers about its beauty. His writing also warned people that Yosemite was in danger from timber mining and sheep ranching interests. In 1901 Theodore Roosevelt became president of the United States. He was interested in conservation. Muir took the president through Yosemite, and Roosevelt helped get legislation passed to create Yosemite National Park in 1906. Although Muir won many conservation battles, he lost a major one. He fought to save the Hetch Valley, which people wanted to dam in order to provide water for San Francisco. In the late 1913 a bill was signed to dam the valley. Muir died in 1914. Some people say losing the fight to protect the valley killed Muir.

Page 29: TABE PERT FCAT Reading and Writing Practice    MmAcVcErReIaCKS

What happened first?

A. The Muir family moved to the United States. B. Muir Woods was created. C. John Muir learned to climb rocky cliffs. D. John Muir walked to the Gulf of Mexico E. Muir visited along the east coast.

2. When did Muir invent a unique form of alarm clock?

A. while the family still lived in Scotland B. after he sailed to San Francisco C. after he traveled in Yosemite D. while the Muir family lived in Wisconsin E. after he took the long walk

3. What did John Muir do soon after he arrived in San Francisco?

A. He ran outside during an earthquake. B. He put forth a theory about how Yosemite was formed. C. He headed inland for the Sierra Nevadas. D. He began to write articles about the Sierra Nevadas. E. He wrote short stories for the local newspaper.

4. When did John Muir meet Theodore Roosevelt?

A. between 1901 and 1906 B. between 1838 and 1868 C. between 1906 and 1914 D. between 1868 and 1901 E. between 1906-1907

5. What happened last?

A. John Muir died. B. John Muir Trail was dedicated. C. Muir's glacial theory was proven. D. The Sierra Club was formed. E. John's family visited him.

6. When using a metal file, always remember to bear down on the forward stroke only. On the return stroke, lift the file clear of the surface to avoid dulling the instrument's teeth. Only when working on very soft metals is it advisable to drag the file's teeth slightly on the return stroke. This helps clear out metal pieces from between the teeth.

It is best to bear down just hard enough to keep the file cutting at all times. Too little pressure uses only the tips of the teeth; too much pressure can chip the teeth. Move the file in straight lines across the surface. Use a vice to grip the work so that your hands are free to hold the file. Protect your hands by equipping the file with a handle. Buy a wooden handle and install it by inserting the pointed end of the file into the handle hole.

These directions show you how to-

A. work with a hammer B. use a file C. polish a file D. oil a vise E. repair shop tools

7. When using a file-

A. always bear down on the return stroke B. move it in a circle C. remove the handle D. press down on the forward stroke E. wear protective gloves

8. When working on soft metals, you can-

A. remove the handle B. clear metal pieces from the teeth C. bear down very hard on the return stroke D. file in circles E. strengthen them with added wood

9. Protect your hands by-

A. dulling the teeth B. dragging the teeth on the backstroke C. using a vise D. installing a handle E. wearing safety gloves

Page 30: TABE PERT FCAT Reading and Writing Practice    MmAcVcErReIaCKS

10. “Old woman,” grumbled the burly white man who had just heard Sojourner Truth speak, “do you think your talk about slavery does any good? I don't care any more for your talk than I do for the bite of a flea.”

The tall, imposing black woman turned her piercing eyes on him. “Perhaps not,” she answered, “but I'll keep you scratching.”

The little incident of the 1840s sums up all that Sojourner Truth was: utterly dedicated to spreading her message, afraid of no one, forceful and witty in speech. Yet forty years earlier, who could have suspected that a spindly slave girl growing up in a damp cellar in upstate New York would become one of the most remarkable women in American history? Her name then was Isabella (many slaves had no last names), and by the time she was fourteen she had seen both parents die of cold and hunger. She herself had been sold several times. By 1827, when New York freed its slaves, she had married and borne five children.

The first hint of Isabella's fighting spirit came soon afterwards, when her youngest son was illegally seized and sold. She marched to the courthouse and badgered officials until her son was returned to her.

In 1843, inspired by religion, she changed her name to Sojourner(meaning “one who stays briefly”) Truth, and, with only pennies in her purse, set out to preach against slavery. From New England to Minnesota she trekked, gaining a reputation for her plain but powerful and moving words. Incredibly, despite being black and female (only white males were expected to be public speakers), she drew thousands to town halls, tents, and churches to hear her powerful, deep-voiced pleas on equality for blacks-and for women. Often she had to face threatening hoodlums. Once she stood before armed bullies and sang a hymn to them. Awed by her courage and her commanding presence, they sheepishly retreated.

During the Civil War she cared for homeless ex-slaves in Washington. President Lincoln invited her to the White House to bestow praise on her. Later, she petitioned Congress to help former slaves get land in the West. Even in her old age, she forced the city of Washington to integrate its trolley cars so that black and white could ride together.

Shortly before her death at eighty-six, she was asked what kept her going. “I think of the great things,” replied Sojourner.

The imposing black woman promised to keep the white man-

A. searching B. crying C. hollering D. scratching E. fleeing

11. This incident occurred in the-

A. 1760s B. 1900s C. 1840s D. 1920s E. 1700s

12. Sojourner Truth was raised in a damp cellar in-

A. New York B. Georgia C. New Jersey D. Idaho E. Maryland

13. Isabella lost both parents by the time she was-

A. twenty-seven B. two C. seven D. fourteen E. nineteen

14. When New York freed its slaves, Isabella had-

A. problems B. no children C. five children D. an education E. three children

15. Her change in name was inspired by-

A. a fighting spirit B. religion C. her freedom D. officials E. friends

16. She traveled from New England to-

A. Canada B. California C. Minnesota 

Page 31: TABE PERT FCAT Reading and Writing Practice    MmAcVcErReIaCKS

D. Alaska E. Virginia

17. She forced the city of Washington to-

A. integrate its trolleys B. give land grants C. care for ex-slaves D. provide food for ex-slaves E. clean its trolleys

18. She preached against-

A. smoking B. slavery C. alcohol D. hoodlums E. women having no rights

19. Sojourner Truth died at-

A. 48 B. 72 C. 63 D. 86 E. 88

20. The Galapagos Islands are in the Pacific Ocean, off the western coast of South America. They are a rocky, lonely spot, but they are also one of the most unusual places in the world. One reason is that they are the home of some of the last giant tortoises left on earth.

Weighing hundreds of pounds, these tortoises, or land turtles, wander slowly around the rocks and sand of the islands. Strangely, each of these islands has its own particular kinds of tortoises. There are seven different kinds of tortoises on the eight islands, each kind being slightly different from the other.

Hundreds of years ago, thousands of tortoises wandered around these islands. However, all that changed when people started landing there. When people first arrived in 1535, their ships had no refrigerators. This meant that fresh food was always a problem for the sailors on board. The giant tortoises provided a solution to this problem.

Ships would anchor off the islands, and crews would row ashore and seize as many tortoises as they could. Once the animals were aboard the ship, the sailors would roll the tortoises onto their backs. The tortoises were completely helpless once on their backs, so they could only lie there until used for soups and stews. Almost 100,000 tortoises were carried off in this way.

The tortoises faced other problems, too. Soon after the first ships, settlers arrived bringing pigs, goats, donkeys, dogs and cats. All of these animals ruined life for the tortoises. Donkey and goats ate all the plants that the tortoises usually fed on, while the pigs. Dogs and cats consumed thousands of baby tortoises each year. Within a few years, it was hard to find any tortoise eggs-or even any baby tortoises.

By the early 1900s, people began to worry that the last of the tortoises would soon die out. No one, however, seemed to care enough to do anything about the problem. More and more tortoises disappeared, even though sailors no longer needed them for food. For another fifty years, this situation continued. Finally, in the 1950s, scientist decided that something must be done.

The first part of their plan was to get rid of as many cats, dogs and other animals as they could. Next, they tried to make sure that more baby tortoises would be born. To do this, they started looking for wild tortoise eggs. They gathered the eggs and put them in safe containers. When the eggs hatched, the scientists raised the tortoises in special pens. Both the eggs and tortoises were numbered so that the scientists knew exactly which kinds of tortoises they had-and which island they came from. Once the tortoises were old enough and big enough to take care of themselves, the scientists took them back to their islands and set them loose. This slow, hard work continues today, and, thanks to it, the number of tortoises is now increasing every year. Perhaps these wonderful animals will not disappear after all.

What happened first?

A. Sailors took tortoises aboard ships. B. The tortoise meat was used for soups and stews. C. Tortoises were put onto their backs. D. Settlers brought other animals to the islands. E. Pigs had been all the sailors had to eat.

21. What happened soon after people brought animals to the islands?

A. Tortoise eggs were kept in safe containers. B. Scientists took away as many animals as they could. C. The animals ate the tortoises' food and eggs. D. The tortoises fought with the other animals. E. The tortoises continued to wander freely.

22. When did people start to do something to save the tortoises?

A. in the 1500s B. in the 1950s C. in the early 1900s D. in the 1960s E. in the 1400s

Page 32: TABE PERT FCAT Reading and Writing Practice    MmAcVcErReIaCKS

23. What happens right after the tortoise eggs hatch?

A. The scientists take the tortoises back to their islands. B. The scientists get rid of cats, dogs, and other animals. C. The sailors use the tortoises for food. D. The scientist raised the tortoises in special pens. E. The scientist encouraged the villagers to help.

24. What happened last?

A. The tortoises began to disappear. B. The number of tortoises began to grow. C. Scientists took away other animals. D. Tortoises were taken back to their home islands. E. The number of tortoises began to decrease.

25. The first person in the group starts off by naming anything that is geographical. It could be a city, state, country, river, lake, or any proper geographical term. For example, the person might say,”Boston.” The second person has ten seconds to think of how the word ends and come up with another geographical term starting with that letter. The second participant might say, “Norway,” since the geographical term has to start with “N.” The third person would have to choose a word beginning with “ Y.” If a player fails to think of a correct answer within the time limit, that player is out of the game. The last person to survive is the champion.

This game may help you with-

A. history B. music C. geography D. sports E. current events

26. The person trying to answer needs-

A. no time limit B. to know geography only C. to ignore the last letters of words D. to know something about spelling and geography E. to be a good speller

27. Before you choose your own word, think about how-

A. the last word starts B. the last word ends C. smart you are D. long the last word is E. the spelling of the first word

28. The answer must be-

A. in New York B. within the United States C. proper geographical terms D. in the same region E. along a coast line

29. Charles A. Lindbergh is remembered as the first person to make a nonstop solo flight across the Atlantic, in 1927. This feat, when Lindbergh was only twenty-five years old, assured him a lifetime of fame and public attention.

Charles Augustus Lindbergh was more interested in flying airplanes than he was in studying. He dropped out of the University of Wisconsin after two years to earn a living performing daredevil airplane stunts at country fairs. Two years later, he joined the United States Army so that he could go to the Army Air Service flight-training school. After completing his training, he was hired to fly mail between St. Louis and Chicago. Then came the historic flight across the Atlantic. In 1919, a New York City hotel owner offered a prize of $25,000 to the first pilot to fly nonstop from New York to Paris. Nine St. Louis business leaders helped pay for the plane Lindbergh designed especially for the flight. Lindbergh tested the plane by flying it from San Diego to New York, with an overnight stop in St. Louis. The flight took only 20 hours and 21 minutes, a transcontinental record.

Nine days later, on May 20,1927, Lindbergh took off from Long Island, New York, at 7:52 A. M. He landed at Paris on May 21 at 10:21 P. M. He had flown more than 3,600 miles in less than thirty four hours. His flight made news around the world. He was given awards and parades everywhere he went. He was presented with the U. S. Congressional Medal of Honor and the first Distinguished Flying Cross. For a long time, Lindbergh toured the world as a U. S. goodwill ambassador. He met his future wife, Anne Morrow, in Mexico, where her father was the United States ambassador.

During the 1930s, Charles and Anne Lindbergh worked for various airline companies, charting new commercial air routes. In 1931, for a major airline, they charted a new route from the east coast of the United States to the Orient. The shortest, most efficient route was a great curve across Canada, over Alaska, and down to China and Japan. Most pilots familiar with the Arctic did not believe that such a route was possible. The Lindberghs took on the task of proving that it was. They arranged for fuel and supplies to be set out along the route. On July 29, they took off from Long Island in a specially equipped small seaplane. They flew by day and each night landed on a lake or a river and camped. Near Nome, Alaska, they had their first serious emergency. Out of daylight and nearly out of fuel, they were forced down in a small ocean inlet. In the next morning's light, they discovered they had landed on barely three feet of water. On September 19, after two more emergency landings and numerous close calls, they landed in China with the maps for a safe airline passenger route.

Even while actively engaged as a pioneering flier, Lindbergh was also working as an engineer. In 1935, he and Dr. Alexis Carrel were given a patent for an artificial heart. During World War I in the 1940s, Lindbergh served as a civilian technical advisor in aviation. Although he was a civilian, he flew over fifty combat missions in the Pacific. In the 1950s, Lindbergh helped design the famous 747 jet airliner. In the late 1960s, he spoke widely on conservation issues. He died August 1974, having lived through aviation history from the time of the first powered flight to the first steps on the moon and having influenced a big part of that history himself.

Page 33: TABE PERT FCAT Reading and Writing Practice    MmAcVcErReIaCKS

What did Lindbergh do before he crossed the Atlantic?

A. He charted a route to China. B. He graduated from flight-training school. C. He married Anne Morrow. D. He acted as a technical advisor during World War II. E. He was responsible for the fuel supply for planes.

30. What happened immediately after Lindbergh crossed the Atlantic?

A. He flew the mail between St. Louis and Chicago. B. He left college. C. He attended the Army flight-training school. D. He was given the Congressional Medal of Honor. E. He married Anne Morrow.

31. When did Charles meet Anne Morrow?

A. before he took off from Long Island B. after he worked for an airline C. before he was forced down in an ocean inlet D. after he received the first Distinguished Flying Cross E. when visiting his parents

32. When did the Lindberghs map an air route to China?

A. before they worked for an airline B. before Charles worked with Dr. Carrel C. after World War II D. while designing the 747 E. when he was thirty

33. What event happened last?

A. Lindbergh patented an artificial heart. B. The Lindberghs mapped a route to the Orient. C. Lindbergh helped design the 747 airline. D. Lindbergh flew fifty combat missions. E. Charles finally was given an honorary degree from college.

34. Always read the meter dials from the right to the left. This procedure is much easier, especially if any of the dial hands are near the zero mark. If the meter has two dials, and one is smaller than the other, it is not imperative to read the smaller dial since it only registers a small amount. Read the dial at the right first. As the dial turns clockwise, always record the figure the pointer has just passed. Read the next dial to the left and record the figure it has just passed. Continue recording the figures on the dials from right to left. When finished, mark off the number of units recorded. Dials on water and gas meters usually indicate the amount each dial records.

These instructions show you how to –

A. read a meter B. turn the dials of a meter C. install a gas meter D. repair a water meter E. be prepared for outside employment

35. Always read the meter dials-

A. from top to bottom B. from right to left C. from left to right D. from the small to the large dial E. from the large dial to the small dial

36. As you read the first dial, record the figures

A. on the smaller dial B. the pointer is approaching C. the pointer has just passed D. at the top E. at the bottom

37. When you have finished reading the meter, mark off-

A. the number of units recorded B. the figures on the small dial C. the total figures D. all the zero marks E. the last reading of the month

Page 34: TABE PERT FCAT Reading and Writing Practice    MmAcVcErReIaCKS

38. The village of Vestmannaeyjar, in the far northern country of Iceland, is as bright and clean and up-to-date as any American or Canadian suburb. It is located on the island of Heimaey, just off the mainland. One January night in 1973, however, householders were shocked from their sleep. In some backyards red-hot liquid was spurting from the ground. Flaming “skyrockets” shot up and over the houses. The island's volcano, Helgafell, silent for seven thousand years, was violently erupting!

Luckily, the island's fishing fleet was in port, and within twenty-four hours almost everyone was ferried to the mainland. But then the agony of the island began in earnest. As in a nightmare, fountains of burning lava spurted three hundred feet high. Black, baseball-size cinders rained down. An evil-smelling, eye-burning, throat-searing cloud of smoke and gas erupted into the air, and a river of lava flowed down the mountain. The constant shriek of escaping steam was punctuated by ear-splitting explosions.

As time went on, the once pleasant village of Vestmannaeyjar took on a weird aspect. Its street lamps still burning against the long Arctic night, the town lay under a thick blanket of cinders. All that could be seen above the ten-foot black drifts were the tips of street signs. Some houses had collapsed under the weight of cinders; others had burst into flames as the heat ignited their oil storage tanks. Lighting the whole lurid scene, fire continued to shoot from the mouth of the looming volcano.

The eruption continued for six months. Scientists and reporters arrived from around the world to observe the awesome natural event. But the town did not die that easily. In July, when the eruption ceased, the people of Heimaey Island returned to assess the chances of rebuilding their homes and lives. They found tons of ash covering the ground. The Icelanders are a tough people, however, accustomed to the strange and violent nature of their Arctic land. They dug out their homes. They even used the cinders to build new roads and airport runways. Now the new homes of Heimaey are warmed from water pipes heated by molten lava.

The village is located on the island of-

A. Vestmannaeyjar B. Hebrides C. Heimaey D. Helgafell E. Heimma

39. The color of the hot liquid was-

A. orange B. black C. yellow D. red E. gray

40. This liquid was coming from the –

A. mountains B. ground C. sea D. sky E. ocean

41. The island's volcano had been inactive for-

A. seventy years B. seven thousand years C. seven thousand months D. seven hundred years E. seventy decades

42. Black cinders fell that were the size of__

A. baseballs B. pebbles C. golf balls D. footballs E. hail-stones

43. Despite the eruption-

A. buses kept running B. the radio kept broadcasting C. the police kept working D. street lamps kept burning E. the television kept broadcasting

44. This volcanic eruption lasted for six ___.

A. weeks B. hours C. months D. days E. years

Answer Key 1. C 2. D 3. C 4. A 5. B 6. B 

Page 35: TABE PERT FCAT Reading and Writing Practice    MmAcVcErReIaCKS

7. D 8. B 9. D 10. D 11. C 12. A 13. D 14. C 15. B 16. C 17. A 18. B 19. D 20. A 21. C 22. B 23. D 24. B 25. C 26. D 27. B 28. C 29. B 30. D 31. D 32. B 33. C 34. A 35. B 36. C 37. A 38. C 39. D 40. B 41. B 42. A 43. D 44. C 

===========

Sentence Correction Practice Questions

1. If the books have been cataloged last week, why haven't they been placed on the shelf?

A. have been cataloged B. would have been cataloged C. was cataloged D. were cataloged E. had been cataloged

2. Jessica Mitford wrote The American Way of Death, a best-selling book, that led eventually to an official investigation of the funeral industry.

A. that led eventually B. that had led eventually C. that eventually led D. which led eventually E. who eventually led

3. Sabotage came from the French saboter,  which means“to  clatter with wooden shoes (sabots).”

A. which means “ to B. which means, “ to C. that means “to D. that means- “to E. that means, “to

4. In studying an assignment it is wise to read it over quickly at first, than see the major points, and finally outline the material.

A. first, than B. first: then C. first-then D. first, then E. first-than

5. To judge the Tidy City contest, we picked an uninterested party.

A. picked an uninterested party. B. picked an interested party! C. picked a disinterested party. 

Page 36: TABE PERT FCAT Reading and Writing Practice    MmAcVcErReIaCKS

D. are in the process of picking an uninterested party. E. picked an disinterested party.

6. Linda decides they had better scram before the killers find them.

A. had better scram B. had better leave C. should hurry and scram D. could hurry and leave E. had better get out

7. I really dug the character of Brutus.

A. dug B. thought about C. thought of D. admired E. gazed at

8. Once upon a point a time, a small person named Little Red Riding Hood initated plans for the preparation, delivery and transportation of foodstuffs to her Grandmother.

A. and transportation of foodstuffs to her Grandmother. B. and transportation of food stuffs to her Grandmother. C. and transportation of food supplies to her Grandmother. D. and transportation of foodstuffs to her grandmother. E. and, transportation of food supplies to her grand mother.

9. The setting of a story effects the story's plot.

A. effects the story's plot B. effects the stories plot C. affect the story's plot D. affects the story's plot E. affects the story's plots

10. Arctic trees are scrubbiest than trees in milder climates.

A. scrubbiest than trees B. scrubbier then trees C. scrubbiest than are trees D. scrubbier than are trees E. scrubbier than trees

11. Quebec rises in a magnificent way above the St. Lawrence River.

A. rises in a magnificent way above B. rises in a magnificent way, way above C. rises magnificently above D. rises magnificently way above E. is raised in a magnificent way above

12. Someone gives the school gerbils every year.

A. Someone gives the school gerbils B. Some one gives the school gerbils C. Some one gives the School gerbils D. There is a person that gives the school gerbils E. An individual gave gerbils

13. During colonial days, a school room looked rather empty.

A. colonial days, a school room looked B. colonial days, a schoolroom looked C. colonial days; a schoolroom looked D. colonial days; a school room looked E. colonial days- a schoolroom looked

14. The helium- filled balloon rose in the air.

A. rose in the air. B. was rising in the air. C. was in the air. D. rose into the air. E. would rise in the air.

15. If I had the address, I would have delivered the package myself.

Page 37: TABE PERT FCAT Reading and Writing Practice    MmAcVcErReIaCKS

A. had the address, B. had the address; C. had the address- D. had had the address; E. had had the address,

16. Do you know that these gloves have lay on the bureau all week?

A. have lay on B. have laid on C. would lie on D. had laid on E. have lain on

17. If I would have known about the team tryouts, I would have signed up for them.

A. would have known B. would had known C. could of known D. had been told E. could have been told

18. If he would have revised his first draft, he would have received a better grade.

A. would have revised B. had revised C. could of revised D. had of revised E. would revise

19. Valarie claims that cats made the best pets.

A. made the best pets. B. could be the best pets. C. are the best pets. D. make of the best pets E. make the best pets.

20. By next month Ms. Jones will be Mayor of Tallahassee for two years.

A. will be Mayor of Tallahassee B. will have been Mayor of Tallahassee C. will be mayor of Tallahassee D. will have been mayor of Tallahassee E. could have been mayor of Tallahassee

Answer Key 1. D 2. D 3. A 4. D 5. C 6. B 7. D 8. D 9. D 10. E 11. C 12. A 13. B 14. D 15. E 16. E 17. A 18. B 19. E 20. D 

========

Sentence Corrections part 2

1. Hours of driving laid ahead of us.

A. laid B. have lain C. lay D. has lay E. lie

Page 38: TABE PERT FCAT Reading and Writing Practice    MmAcVcErReIaCKS

2. By the time we get to the picnic area, the rain will stop.

A. will stop B. shall stop C. will has stopped D. shall have stopped E. will have stopped

3. If Judy would not have missed the deadline, the yearbook delivery would have been on time.

A. would not have missed B. should have not missed C. wouldn't have missed D. had not missed E. would have not missed

4. We spent Sunday afternoon wandering aimless in the park.

A. wandering aimless B. wandering aimlessly C. wandering without purpose D. wandering in an aimless manner E. wandering almost aimlessly

5. Only after I went home did I remember my dental appointment.

A. went home B. had went home C. had gone home D. gone home E. should go home

6. The book lay open at page 77.

A. lay open B. laid open C. lied open D. lain open E. was laid open

7. By this time next year Johanna will begin classes at the University of Colorado.

A. will begin classes B. will have begun classes C. has began classes D. should begin classes E. should have begun classes

8. After comparing my air conditioner with the one on sale, I decided that mine was the most efficient.

A. was the most efficient. B. should be the most efficient. C. was the more efficient. D. was, by far the most efficient E. should be considered the most efficient.

9. I would have liked to have gone swimming yesterday.

A. to have gone swimming B. to go swimming C. to had gone swimming D. to go to swim E. to of gone swimming

10. I wish I read the chapter before I tried to answer the questions.

A. read the chapter B. would read the chapter C. should of read the chapter D. could have read the chapter E. had read the chapter

11. Nathanael West said that he'd never have written his satirical novel if he had not visited Hollywood.

A. have written his B. would have written his C. could of written his D. could have written his E. should of written his

Page 39: TABE PERT FCAT Reading and Writing Practice    MmAcVcErReIaCKS

12. The smell from the paper mill laid   over the town like a blanket.

A. laid B. has lain C. will lie D. lay E. has laid

13. When I was halfway down the stairs, I suddenly knew what I had wanted to have said.

A. to have said B. too say C. to have been said D. to had say E. to say

14. I would be more careful if I had been you.

A. had been B. could have been C. was D. were E. could have been

15. They read where the governor has appointed a special committee to improve the school calendar.

A. where B. how C. were D. of where E. wear

16. In study hall I sit besides Paul Smith, who is captain of the swim team and one of the best swimmers in the state.

A. sit besides B. sat beside C. have set beside D. sit beside E. have sit beside

17. This classic has been read with enjoyment for nearly two hundred years.

A. has been read B. will have been read C. shall have been read D. is being read E. was read

18. Many nineteenth-century biographers rely on their imagination, not on real facts.

A. rely on their imagination, B. relied on their imagination, C. have relied on their imagination D. could have relied on their imagination, E. could rely on their imaginations:

19. The private lives of politicians, generals, and other notables fascinates the reading public.

A. fascinates the reading B. have fascinated the reading C. will fascinate the reading D. fascinate the reading E. has fascinate the reading

20. That small man chose a seat near the door and carefully sat down.

A. sat B. will sit C. could of sat D. have sit down E. set down

21. Last summer I worked in the chemical laboratory at the Brass Company; most the work came into the lab for testing marked with thewords top priority.

A. words top priority. B. words - top priority. C. words: Top priority. D. words, “Top Priority.” E. Words “top priority.”

Page 40: TABE PERT FCAT Reading and Writing Practice    MmAcVcErReIaCKS

Answer Key 1. C 2. E 3. D 4. B 5. C 6. A 7. B 8. C 9. A 10. E 11. A 12. D 13. E 14. D 15. A 16. D 17. A 18. B 19. D 20. A 21. D 

=====

Science Readings for Practice

1. In 1949, a study of heart disease included 5,137 adults: 2,292 men and 2,845 women. All the individuals were 49 to 70 years of age, and none showed any signs of coronary artery disease. After 14-16 years of careful follow-up, it was found that: 

I.151 men and 37 women showed evidence of coronary artery disease that would account for a heart attack. II.102 men and 18 women died of coronary artery disease before they reached the age of 65. III.58 percent of the men and 39 percent of the women died within one hour of having a heart attack. IV.For both men and women, sudden death was more likely if they were under age 55 at the time of the attack.V.Not all heart attacks produce symptoms. A considerable number of men and women had a myocardial infarction (heart muscle damage) without knowing it. Of those who suffered such "silent coronaries"; 22 percent of the men and 35 percent of the women did not know muscle damage had occurred.

In the design of this study, the researchers made sure that

A. some individuals had coronary artery disease B. some individuals were in their thirties C. the number of men and women was equal D. all participants were adults

2. As explained in the study, a person who has a "silent coronary"

A. is unaware of any heart attack symptoms B. does not scream in pain while having a heart attack C. shows no evidence of heart muscle damage D. always dies of a myocardial infarction

3. A woman who is 53 years old has a heart attack. Compared with a man, she is more likely to

A. die of coronary artery disease before age 65 B. have a "silent coronary" C. die within an hour of the attack D. die as the result of a heart attack after age 65

4. Which conclusion is consistent with the results of the study?

A. Chances of death within one hour of a heart attack are greater for women than men. B. Chances of death from coronary artery disease are less for men than women. C. Evidence of coronary artery disease is equal among men and women. D. Sudden death is more likely for both men and women if they are under age 55 at the time of the attack.

5. Based on the results of this experiment, of the people who died of coronary artery disease before age 65

A. 18 percent were women B. 20 percent were women C. 85 percent were men D. 102 percent were men

6. A group of 1000 men and 1200 women between the ages of 50 and 65 are to be studied for coronary artery disease. Based on the original study, which of the following results could be predicted most reliably?

A. Eight (8) women will die of coronary heart disease after they reach age 65. B. Sixteen (16) women will show significant evidence of coronary artery disease. 

Page 41: TABE PERT FCAT Reading and Writing Practice    MmAcVcErReIaCKS

C. Forty-four (44) men will die of coronary artery disease after they reach age 65. D. Sixty-five (65) men will show no significant evidence of coronary artery disease.

7. A chemistry student placed a strip of blue litmus paper and a strip of pink litmus paper in a glass dish. Then she added a drop of dilute sulfuric acid to each strip of litmus paper. She observed that the blue litmus paper turned pink, but the pink litmuspaper did not change color. Next she placed a drop of sodium hydroxide (NaOH) on other strips of blue and pink litmus paper. This time, the pink litmus paper turned blue, but the blue litmus paper did not change. Finally, she put a drop of distilled water onstrips of blue and pink litmus paper. Neither strip changed color. She repeated the tests several times with the same results. The student concluded that acids turn blue litmus paper pink; bases, such as sodium hydroxide, turn pink litmus paper blue.As water did not affect either pink or blue litmus paper, she reasoned that water was not an acid or a base, but a neutral substance.

Keeping these results in mind, the student poured a little sodium hydroxide into a beaker containingpink and blue litmus paper. Then she added hydrochloric acid (HCl) drop by drop until the solutionbecame neutral. She determined that a new, neutral substance had formed in the beaker. The substance was table salt, or sodium chloride (NaCl), which isone of many salts formed from an acid and a base.

If a drop of an unknown substance turns blue litmus paper pink, but does not change pink litmus paper, the substance is a(n):

A. acid B. base C. water D. salt

8. In the presence of potassium hydroxide (KOH):

A. blue litmus paper turns pink B. pink litmus paper turns blue C. blue litmus paper becomes darker D. pink litmus paper does not change

9. When strips of blue and pink litmus paper are put in a beaker filled with a clear solution, neither litmus paper changes color. The solution:

A. must be water B. must be neutral C. may be an acid D. may be a base

10. In another experiment, the student added hydrochloric acid drop by drop to a solution of sodium hydroxide containing strips of originally blue and originally pink litmus paper. As she continued adding acid, the originally:

A. pink litmus paper remained pink B. blue litmus paper remained blue C. blue litmus paper turned from pink back to blue D. pink litmus paper turned from blue back to pink

11. Based on the results of this experiment, a salt would be formed when:

A. NaCl is combined with NaOH B. H2O is combined with HCl C. KOH is combined with HCl D. HCl is combined with H2OSO4

12. In setting up an aquarium, several factors must be considered before introducing fish. Which of the following factors could be tested using litmus paper?

A. salinity B. acidity C. chlorination D. temperature

13. The complex behavior of the poor-sighted, three-spined male stickleback fish has been studied extensively as a model of species behavior in courtship and mating. After a male has migrated to a suitable spot, he builds a spawning nest of sand and sediment. In courting, he performs a special "zigzag" dance. The female then follows the male to the nest where she spawns and he fertilizes the spawned eggs. Also, male sticklebacks have been shown to exhibit territorial behaviors. A biologist performed three experiments to learn more about the behavior of the stickleback.

Experiment 1

Tank 1 and Tank 2 are set up with identical conditions and one male stickleback is placed in each tank. Both fish build nests in their respective tanks. The male from Tank 1 is removed from his tank and is replaced with an egg-laden female; the male from Tank 2 is removed from his tank and is introduced into Tank 1. In Tank 1, the male does not perform the zigzag dance and no spawning occurs. The male retreats to a corner of the tank.

Experiment 2

A male stickleback in an aquarium builds his nest. A fat, round male is introduced into the environment. The original male performs the zigzag dance and attempts to lead the round male to the nest. The round male refuses and begins to flap his fins and swim in circles. The first male then begins to flap his fins, circle his nest, and occasionally prod the other fish to a far corner of the tank.

Experiment 3

A small, flat-shaped female is introduced into a tank where a male has built a nest. The male circles the female a few times, and then retreats to a corner of the tank.

Page 42: TABE PERT FCAT Reading and Writing Practice    MmAcVcErReIaCKS

The experimental data would support the hypothesis that the purpose of the male stickleback's mating dance is to:

A. keep away other male sticklebacks. B. fertilize the eggs. C. lure and entice the female to the nest D. establish territorial rights.

14. Based on observations from the above experiments, which factor initially stimulates the male to do the zigzag dance?

A. The physical environment. B. The shape of the fish. C. The number of fish in the tank. D. The sex of the fish.

15. Which experiment supports the hypothesis that the male exhibits territorial behavior?

A. 1 only. B. 2 only. C. 1 and 2 only. D. 1, 2, and 3.

16. To further investigate the territorial behavior of the stickleback, the biologist should vary which of the following factors in Experiment 2?

A. The temperature of the water. B. The fatness of the male fish. C. The sediment and sand in the tank. D. The size of the tank.

17. To clarify the results of Experiment 1, the biologist should set up which of the following test situations?

A. Maintain the positions of the male sticklebacks and add another egg-laden female to Tank 1. B. Place both male sticklebacks in Tank 2. C. Return the original male stickleback to Tank 1 and observe its behavior with the female fish. D. Repeat the experiment using a different species of fish.

18. A male stickleback has been established in an aquarium and has built a nest. If one egg-laden female and several flat-shaped male sticklebacks are placed in the tank, one would most likely observe:

A. all the males would perform the zigzag dance. B. all the males would circle the female. C. only the male that was originally in the tank would perform the zigzag dance. D. the female would retreat to a corner.

Answer Key 1. D 2. A 3. B 4. D 5. C 6. B 7. A 8. B 9. B 10. D 11. C 12. B 13. C 14. A 15. C 16. D 17. C 18. C 

WORD LIST

The following is a list of high frequency vocabulary words found on multiple exams, such as the GRE and SAT. Make a study schedule and memorize the following terms to excel on your test.

aberrant: Markedly different from an accepted norm.

aberration: Deviation from a right, customary, or prescribed course.

abet: To aid, promote, or encourage the commission of (an offense).

abeyance: A state of suspension or temporary inaction.

Page 43: TABE PERT FCAT Reading and Writing Practice    MmAcVcErReIaCKS

abjure: To recant, renounce, repudiate under oath.

ablution: A washing or cleansing, especially of the body.

abrogate: To abolish, repeal.

abscond: To depart suddenly and secretly, as for the purpose of escaping arrest.

abstemious: Characterized by self denial or abstinence, as in the use of drink, food.

abstruse: Dealing with matters difficult to be understood.

abut: To touch at the end or boundary line.

accede: To agree.

acquiesce: To comply; submit.

acrid: Harshly pungent or bitter.

acumen: Quickness of intellectual insight, or discernment; keenness of discrimination.

adage: An old saying.

adamant: Any substance of exceeding hardness or impenetrability.

admonition: Gentle reproof.

adumbrate: To represent beforehand in outline or by emblem.

affable: Easy to approach.

aggrandize: To cause to appear greatly.

aggravate: To make heavier, worse, or more burdensome.

agile: Able to move or act quickly, physically, or mentally.

agog: In eager desire.

alacrity: Cheerful willingness.

alcove: A covered recess connected with or at the side of a larger room.

alleviate: To make less burdensome or less hard to bear.

aloof: Not in sympathy with or desiring to associate with others.

amalgamate: To mix or blend together in a homogeneous body.

ambidextrous: Having the ability of using both hands with equal skill or ease.

ambiguous: Having a double meaning.

ameliorate: To relieve, as from pain or hardship

anathema: Anything forbidden, as by social usage.

animadversion: The utterance of criticism or censure.

animosity: Hatred.

antediluvian: Of or pertaining to the times, things, events before the great flood in the days of Noah.

antidote: Anything that will counteract or remove the effects of poison, disease, or the like.

aplomb: Confidence; coolness.

Page 44: TABE PERT FCAT Reading and Writing Practice    MmAcVcErReIaCKS

apocryphal : Of doubtful authority or authenticity.

apogee: The climax.

apostate: False.

apotheosis: Deification.

apparition: Ghost.

appease: To soothe by quieting anger or indignation.

apposite: Appropriate.

apprise: To give notice to; to inform.

approbation: Sanction.

arboreal: Of or pertaining to a tree or trees.

ardor: Intensity of passion or affection.

argot: A specialized vocabulary peculiar to a particular group.

arrant: Notoriously bad.

ascetic: Given to severe self-denial and practicing excessive abstinence and devotion.

ascribe: To assign as a quality or attribute.

asperity: Harshness or roughness of temper.

assiduous: Unceasing; persistent

assuage: To cause to be less harsh, violent, or severe, as excitement, appetite, pain, or disease.

astringent: Harsh in disposition or character.

astute: Keen in discernment.

atonement: Amends, reparation, or expiation made from wrong or injury.

audacious: Fearless.

augury: Omen

auspicious: Favorable omen

austere: Severely simple; unadorned.

autocrat: Any one who claims or wields unrestricted or undisputed authority or influence.

auxiliary: One who or that which aids or helps, especially when regarded as subsidiary or accessory.

avarice: Passion for getting and keeping riches.

aver: To avouch, justify or prove

aversion: A mental condition of fixed opposition to or dislike of some particular thing.

avow: To declare openly.

baleful: Malignant.

banal: Commonplace.

bask: To make warm by genial heat.

Page 45: TABE PERT FCAT Reading and Writing Practice    MmAcVcErReIaCKS

beatify: To make supremely happy.

bedaub: To smear over, as with something oily or sticky.

bellicose: Warlike.

belligerent: Manifesting a warlike spirit.

benefactor: A doer of kindly and charitable acts.

benevolence: Any act of kindness or well-doing.

benign: Good and kind of heart.

berate: To scold severely.

bewilder: To confuse the perceptions or judgment of.

blandishment: Flattery intended to persuade.

blatant: Noisily or offensively loud or clamorous.

blithe: Joyous.

boisterous: Unchecked merriment or animal spirits.

bolster: To support, as something wrong.

bombast: Inflated or extravagant language, especially on unimportant subjects.

boorish: Rude.

breach: The violation of official duty, lawful right, or a legal obligation.

brittle: Fragile.

broach: To mention, for the first time.

bumptious: Full of offensive and aggressive self-conceit.

buoyant: Having the power or tendency to float or keep afloat.

burnish: To make brilliant or shining.

cabal: A number of persons secretly united for effecting by intrigue some private purpose.

cacophony: A disagreeable, harsh, or discordant sound or combination of sounds or tones.

cajole: To impose on or dupe by flattering speech.

callow: Without experience of the world.

calumny: Slander.

candid: Straightforward.

cant: To talk in a singsong, preaching tone with affected solemnity.

capacious: Roomy.

capitulate: To surrender or stipulate terms.

captious: Hypercritical.

castigate: To punish.

cataract: Opacity of the lens of the eye resulting in complete or partial blindness.

Page 46: TABE PERT FCAT Reading and Writing Practice    MmAcVcErReIaCKS

caustic: Sarcastic and severe.

censure: To criticize severely; also, an expression of disapproval.

centurion: A captain of a company of one hundred infantry in the ancient Roman army.

chagrin: Keen vexation, annoyance, or mortification, as at one's failures or errors.

chary: Careful; wary; cautious.

chicanery: The use of trickery to deceive.

circumlocution: Indirect or roundabout expression.

coddle: To treat as a baby or an invalid.

coerce: To force.

coeval: Existing during the same period of time; also, a contemporary.

cogent: Appealing strongly to the reason or conscience.

cogitate: Consider carefully and deeply; ponder.

cognizant: Taking notice.

colloquial: Pertaining or peculiar to common speech as distinguished from literary.

collusion: A secret agreement for a wrongful purpose.

comestible: Fit to be eaten.

commemorate: To serve as a remembrance of.

complaisance: Politeness.

complement: To make complete.

comport: To conduct or behave (oneself).

compunction: Remorseful feeling.

conceit: Self-flattering opinion.

conciliatory: Tending to reconcile.

concord: Harmony.

concur: To agree.

condense: To abridge.

conflagration: A great fire, as of many buildings, a forest, or the like.

confluence: The place where streams meet.

congeal: To coagulate.

conjoin: To unite.

connoisseur: A critical judge of art, especially one with thorough knowledge and sound judgment of art.

console: To comfort.

conspicuous: Clearly visible.

consternation: Panic.

Page 47: TABE PERT FCAT Reading and Writing Practice    MmAcVcErReIaCKS

constrict: To bind.

consummate: To bring to completion.

contiguous: Touching or joining at the edge or boundary.

contrite: Broken in spirit because of a sense of sin.

contumacious: Rebellious.

copious: Plenteous.

cornucopia: The horn of plenty, symbolizing peace and prosperity.

corporeal: Of a material nature; physical.

correlate: To put in some relation of connection or correspondence.

corroboration: Confirmation.

counterfeit: Made to resemble something else.

countervail: To offset.

covert: Concealed, especially for an evil purpose.

cower: To crouch down tremblingly, as through fear or shame.

crass: Coarse or thick in nature or structure, as opposed to thin or fine.

credulous: Easily deceived.

cupidity: Avarice.

cursory: Rapid and superficial.

curtail: To cut off or cut short.

cynosure: That to which general interest or attention is directed.

dearth: Scarcity, as of something customary, essential,or desirable.

defer: To delay or put off to some other time.

deign: To deem worthy of notice or account.

deleterious: Hurtful, morally or physically.

delineate: To represent by sketch or diagram.

deluge: To overwhelm with a flood of water.

demagogue: An unprincipled politician.

denizen: Inhabitant.

denouement: That part of a play or story in which the mystery is cleared up.

deplete: To reduce or lessen, as by use, exhaustion, or waste.

deposition: Testimony legally taken on interrogatories and reduced to writing, for use as evidence in court.

deprave: To render bad, especially morally bad.

deprecate: To express disapproval or regret for, with hope for the opposite.

deride: To ridicule.

Page 48: TABE PERT FCAT Reading and Writing Practice    MmAcVcErReIaCKS

derision: Ridicule.

derivative: Coming or acquired from some origin.

descry: To discern.

desiccant: Any remedy which, when applied externally, dries up or absorbs moisture, as that of wounds.

desuetude: A state of disuse or inactivity.

desultory: Not connected with what precedes.

deter: To frighten away.

dexterity: Readiness, precision, efficiency, and ease in any physical activity or in any mechanical work.

diaphanous: Transparent.

diatribe: A bitter or malicious criticism.

didactic: Pertaining to teaching.

diffidence: Self-distrust.

diffident: Affected or possessed with self-distrust.

dilate: To enlarge in all directions.

dilatory: Tending to cause delay.

disallow: To withhold permission or sanction.

discomfit: To put to confusion.

disconcert: To disturb the composure of.

disconsolate : Hopelessly sad; also, saddening; cheerless.

discountenance: To look upon with disfavor.

discredit: To injure the reputation of.

discreet: Judicious.

disheveled: Disordered; disorderly; untidy.

dissemble: To hide by pretending something different.

disseminate: To sow or scatter abroad, as seed is sown.

dissent: Disagreement.

dissolution: A breaking up of a union of persons.

distraught: Bewildered.

divulge: To tell or make known, as something previously private or secret.

dogmatic: Making statements without argument or evidence.

dormant: Being in a state of or resembling sleep.

dubious: Doubtful.

duplicity: Double-dealing.

earthenware: Anything made of clay and baked in a kiln or dried in the sun.

Page 49: TABE PERT FCAT Reading and Writing Practice    MmAcVcErReIaCKS

ebullient: Showing enthusiasm or exhilaration of feeling.

edacious: Given to eating.

edible: Suitable to be eaten.

educe: To draw out.

effete: Exhausted, as having performed its functions.

efficacy: The power to produce an intended effect as shown in the production of it.

effrontery: Unblushing impudence.

effulgence: Splendor.

egregious: Extreme.

egress: Any place of exit.

elegy: A lyric poem lamenting the dead.

elicit: To educe or extract gradually or without violence.

elucidate: To bring out more clearly the facts concerning.

emaciate: To waste away in flesh.

embellish: To make beautiful or elegant by adding attractive or ornamental features.

embezzle: To misappropriate secretly.

emblazon: To set forth publicly or in glowing terms.

encomium: A formal or discriminating expression of praise.

encumbrance: A burdensome and troublesome load.

endemic: Peculiar to some specified country or people.

enervate: To render ineffective or inoperative.

engender: To produce.

engrave: To cut or carve in or upon some surface.

enigma: A riddle.

enmity: Hatred.

entangle: To involve in difficulties, confusion, or complications.

entreat: To ask for or request earnestly.

Epicurean: Indulging, ministering, or pertaining to daintiness of appetite.

epithet: Word used adjectivally to describe some quality or attribute of is objects, as in "Father Aeneas".

epitome: A simplified representation.

equable: Equal and uniform; also, serene.

equanimity: Evenness of mind or temper.

equanimity : Calmness; composure.

equilibrium: A state of balance.

Page 50: TABE PERT FCAT Reading and Writing Practice    MmAcVcErReIaCKS

equivocal: Ambiguous.

equivocate: To use words of double meaning.

eradicate: To destroy thoroughly.

errant: Roving or wandering, as in search of adventure or opportunity for gallant deeds.

erratic: Irregular.

erroneous: Incorrect.

erudite: Very-learned.

eschew: To keep clear of.

espy: To keep close watch.

eulogy: A spoken or written laudation of a person's life or character.

euphonious: Characterized by agreeableness of sound.

evanescent: Fleeting.

evince: To make manifest or evident.

evoke: To call or summon forth.

exacerbate: To make more sharp, severe, or virulent.

exculpate: To relieve of blame.

exhaustive: Thorough and complete in execution.

exigency: A critical period or condition.

exigency : State of requiring immediate action; also, an urgent situation; also, that which is required in a

exorbitant: Going beyond usual and proper limits.

expatiate: To speak or write at some length.

expedient: Contributing to personal advantage.

expiate: To make satisfaction or amends for.

explicate: To clear from involvement.

expostulate: To discuss.

expropriate: To deprive of possession; also, to transfer (another's property) to oneself.

extant: Still existing and known.

extempore: Without studied or special preparation.

extenuate: To diminish the gravity or importance of.

extinct: Being no longer in existence.

extinguish: To render extinct.

extirpate: To root out; to eradicate.

extol: To praise in the highest terms.

extort: To obtain by violence, threats, compulsion, or the subjection of another to some necessity.

Page 51: TABE PERT FCAT Reading and Writing Practice    MmAcVcErReIaCKS

extraneous: Having no essential relation to a subject.

exuberance: Rich supply.

facetious: Amusing.

facile: Not difficult to do.

factious: Turbulent.

fallacious: Illogical.

fatuous: Idiotic

fawn: A young deer.

feint: Any sham, pretense, or deceptive movement.

felon: A criminal or depraved person.

ferocity: Savageness.

fervid: Intense.

fervor: Ardor or intensity of feeling.

fidelity: Loyalty.

finesse: Subtle contrivance used to gain a point.

flamboyant: Characterized by extravagance and in general by want of good taste.

flippant: Having a light, pert, trifling disposition.

florid: Flushed with red.

flout: To treat with contempt.

foible: A personal weakness or failing.

foment: To nurse to life or activity; to encourage.

foppish: Characteristic of one who is unduly devoted to dress and the niceties of manners.

forbearance: Patient endurance or toleration of offenses.

forfeit: To lose possession of through failure to fulfill some obligation.

forgery: Counterfeiting.

forswear: To renounce upon oath.

fragile: Easily broken.

frantic: Frenzied.

frugal: Economical.

fugacious: Fleeting.

fulminate: To cause to explode.

fulsome: Offensive from excess of praise or commendation.

gainsay: To contradict; to deny.

gamut: The whole range or sequence.

Page 52: TABE PERT FCAT Reading and Writing Practice    MmAcVcErReIaCKS

garrulous: Given to constant trivial talking.

germane: Relevant.

gesticulate: To make gestures or motions, as in speaking, or in place of speech.

glimmer: A faint, wavering, unsteady light.

gossamer: Flimsy.

gourmand: A connoisseur in the delicacies of the table.

grandiloquent: Speaking in or characterized by a pompous or bombastic style.

gregarious: Sociable, outgoing

grievous: Creating affliction.

guile: Duplicity.

gullible: Credulous.

halcyon: Calm.

harangue: A tirade.

harbinger: One who or that which foreruns and announces the coming of any person or thing.

head: Adv. Precipitately, as in diving.

heinous: Odiously sinful.

heresy: An opinion or doctrine subversive of settled beliefs or accepted principles.

heterogeneous: Consisting of dissimilar elements or ingredients of different kinds.

hirsute: Having a hairy covering.

hoodwink: To deceive.

hospitable: Disposed to treat strangers or guests with generous kindness.

hypocrisy: Extreme insincerity.

iconoclast: An image-breaker.

idiosyncrasy: A mental quality or habit peculiar to an individual.

ignoble: Low in character or purpose.

ignominious: Shameful.

illicit: Unlawful.

imbroglio: A misunderstanding attended by ill feeling, perplexity, or strife.

imbue : To dye; to instill profoundly.

immaculate: Without spot or blemish.

imminent: Dangerous and close at hand.

immutable: Unchangeable.

impair: To cause to become less or worse.

impassive: Unmoved by or not exhibiting feeling.

Page 53: TABE PERT FCAT Reading and Writing Practice    MmAcVcErReIaCKS

impecunious: Having no money.

impede: To be an obstacle or to place obstacles in the way of.

imperative: Obligatory.

imperious: Insisting on obedience.

imperturbable: Calm.

impervious: Impenetrable.

impetuous: Impulsive.

impiety: Irreverence toward God.

implacable: Incapable of being pacified.

implicate: To show or prove to be involved in or concerned

implicit: Implied.

importunate: Urgent in character, request, or demand.

importune: To harass with persistent demands or entreaties.

impromptu: Anything done or said on the impulse of the moment.

improvident: Lacking foresight or thrift.

impugn: To assail with arguments, insinuations, or accusations.

impute: To attribute.

inadvertent: Accidental.

inane: Silly.

incessant: Unceasing.

inchoate: Incipient.

incipient: Initial.

incite: To rouse to a particular action.

incongruous: Unsuitable for the time, place, or occasion.

inculcate: To teach by frequent repetitions.

indelible: That can not be blotted out, effaced, destroyed, or removed.

indigence: Poverty.

indigenous: Native.

indistinct: Vague.

indolence: Laziness.

indolent: Habitually inactive or idle.

indomitable: Unconquerable.

indulgent: Yielding to the desires or humor of oneself or those under one's care.

ineffable: Unutterable.

Page 54: TABE PERT FCAT Reading and Writing Practice    MmAcVcErReIaCKS

ineluctable: Impossible to avoid.

inept: Not fit or suitable.

inexorable: Unrelenting.

infuse: To instill, introduce, or inculcate, as principles or qualities.

ingenuous: Candid, frank, or open in character or quality.

inimical: Adverse.

innocuous: Harmless.

inscrutable: Impenetrably mysterious or profound.

insensible: Imperceptible.

insinuate: To imply.

insipid: Tasteless.

insouciant: Nonchalant.

insurrection: The state of being in active resistance to authority.

interdict: Authoritative act of prohibition.

interim: Time between acts or periods.

intransigent: Not capable of being swayed or diverted from a course.

intrepid: Fearless and bold.

introspection: The act of observing and analyzing one's own thoughts and feelings.

inundate: To fill with an overflowing abundance.

inure: To harden or toughen by use, exercise, or exposure.

invalid: One who is disabled by illness or injury.

invective: An utterance intended to cast censure, or reproach.

inveigh: To utter vehement censure or invective.

inveterate: Habitual.

invidious: Showing or feeling envy.

invincible: Not to be conquered, subdued, or overcome.

iota: A small or insignificant mark or part.

irascible: Prone to anger.

irate: Moved to anger.

ire: Wrath.

irksome: Wearisome.

itinerant: Wandering.

itinerate: To wander from place to place.

jocular: Inclined to joke.

Page 55: TABE PERT FCAT Reading and Writing Practice    MmAcVcErReIaCKS

jovial: Merry.

judicious: Prudent.

junta: A council or assembly that deliberates in secret upon the affairs of government.

lachrymose: Given to shedding tears.

lackadaisical: Listless.

languid: Relaxed.

lascivious: Lustful.

lassitude: Lack of vitality or energy.

latent: Dormant.

laudable: Praiseworthy.

laudatory: Pertaining to, expressing, or containing praise.

legacy: A bequest.

levee: An embankment beside a river or stream or an arm of the sea, to prevent overflow.

levity: Frivolity.

lexicon: A dictionary.

libel: Defamation.

licentious: Wanton.

lien: A legal claim or hold on property, as security for a debt or charge.

listless: Inattentive.

lithe: Supple.

loquacious: Talkative.

lugubrious: Indicating sorrow, often ridiculously.

luminary: One of the heavenly bodies as a source of light.

lustrous: Shining.

malaise: A condition of uneasiness or ill-being.

malcontent: One who is dissatisfied with the existing state of affairs.

malevolence: Ill will.

malign: To speak evil of, especially to do so falsely and severely.

malleable: Pliant.

massacre: The unnecessary and indiscriminate killing of human beings.

maudlin: Foolishly and tearfully affectionate.

mawkish: Sickening or insipid.

mellifluous: Sweetly or smoothly flowing.

mendacious: Untrue.

Page 56: TABE PERT FCAT Reading and Writing Practice    MmAcVcErReIaCKS

mendicant: A beggar.

meretricious: Alluring by false or gaudy show.

mesmerize: To hypnotize.

meticulous: Over-cautious.

mettle: Courage.

mettlesome: Having courage or spirit.

microcosm: The world or universe on a small scale.

mien: The external appearance or manner of a person.

mischievous: Fond of tricks.

miscreant: A villain.

miser: A person given to saving and hoarding unduly.

misnomer: A name wrongly or mistakenly applied.

moderation: Temperance.

modicum: A small or token amount.

mollify: To soothe.

molt: To cast off, as hair, feathers, etc.

monomania: The unreasonable pursuit of one idea.

morbid: Caused by or denoting a diseased or unsound condition of body or mind.

mordant: Biting.

moribund: On the point of dying.

morose: Gloomy.

multifarious: Having great diversity or variety.

mundane: Worldly, as opposed to spiritual or celestial.

munificent: Extraordinarily generous.

myriad: A vast indefinite number.

nadir: The lowest point.

nefarious: Wicked in the extreme.

negligent: Apt to omit what ought to be done.

neophyte: Having the character of a beginner.

noisome: Very offensive, particularly to the sense of smell.

nostrum: Any scheme or recipe of a charlatan character.

noxious: Hurtful.

nugatory: Having no power or force.

obdurate: Impassive to feelings of humanity or pity.

Page 57: TABE PERT FCAT Reading and Writing Practice    MmAcVcErReIaCKS

obfuscate: To darken; to obscure.

oblique: Slanting; said of lines.

obsequious: Showing a servile readiness to fall in with the wishes or will of another.

obstreperous: Boisterous.

obtrude: To be pushed or to push oneself into undue prominence.

obtrusive: Tending to be pushed or to push oneself into undue prominence.

obviate: To clear away or provide for, as an objection or difficulty.

odious: Hateful.

odium: A feeling of extreme repugnance, or of dislike and disgust.

officious: Intermeddling with what is not one's concern.

ominous: Portentous.

onerous: Burdensome or oppressive.

onus: A burden or responsibility.

opprobrium: The state of being scornfully reproached or accused of evil.

ossify: To convert into bone.

ostentation: A display dictated by vanity and intended to invite applause or flattery.

ostracism: Exclusion from intercourse or favor, as in society or politics.

ostracize: To exclude from public or private favor.

palate: The roof of the mouth.

palatial: Magnificent.

palliate: To cause to appear less guilty.

palpable: Perceptible by feeling or touch.

panacea: A remedy or medicine proposed for or professing to cure all diseases.

panegyric: A formal and elaborate eulogy, written or spoken, of a person or of an act.

panoply: A full set of armor.

paragon: A model of excellence.

Pariah: A member of a degraded class; a social outcast.

paroxysm: A sudden outburst of any kind of activity.

parsimonious: Unduly sparing in the use or expenditure of money.

partisan: Characterized by or exhibiting undue or unreasoning devotion to a party.

pathos: The quality in any form of representation that rouses emotion or sympathy.

paucity: Fewness.

peccadillo: A small breach of propriety or principle.

pedestrian: One who journeys on foot.

Page 58: TABE PERT FCAT Reading and Writing Practice    MmAcVcErReIaCKS

pellucid: Translucent.

penchant: A bias in favor of something.

penurious: Excessively sparing in the use of money.

penury: Indigence.

peregrination: A wandering.

peremptory: Precluding question or appeal.

perfidy: Treachery.

perfunctory: Half-hearted.

peripatetic: Walking about.

perjury: A solemn assertion of a falsity.

permeate: To pervade.

pernicious: Tending to kill or hurt.

persiflage: Banter.

perspicacity: Acuteness or discernment.

perturbation: Mental excitement or confusion.

petrify: To convert into a substance of stony hardness and character.

petulant: Displaying impatience.

phlegmatic: Not easily roused to feeling or action.

physiognomy: The external appearance merely.

pious: Religious.

pique: To excite a slight degree of anger in.

placate: To bring from a state of angry or hostile feeling to one of patience or friendliness.

platitude: A written or spoken statement that is flat, dull, or commonplace.

plea: An argument to obtain some desired action.

plenary: Entire.

plethora: Excess; superabundance.

plumb: A weight suspended by a line to test the verticality of something.

plummet: A piece of lead for making soundings, adjusting walls to the vertical.

poignant: Severely painful or acute to the spirit.

polyglot: Speaking several tongues.

ponderous: Unusually weighty or forcible.

portend: To indicate as being about to happen, especially by previous signs.

portent: Anything that indicates what is to happen.

precarious: Perilous.

Page 59: TABE PERT FCAT Reading and Writing Practice    MmAcVcErReIaCKS

preclude: To prevent.

precocious: Having the mental faculties prematurely developed.

predominate: To be chief in importance, quantity, or degree.

premature: Coming too soon.

presage: To foretell.

prescience: Knowledge of events before they take place.

presumption: That which may be logically assumed to be true until disproved.

preternatural: Extraordinary.

prevalent: Of wide extent or frequent occurrence.

prevaricate: To use ambiguous or evasive language for the purpose of deceiving or diverting attention.

prim: Stiffly proper.

pristine: Primitive.

probity: Virtue or integrity tested and confirmed.

proclivity: A natural inclination.

procrastination: Delay.

prodigal: One wasteful or extravagant, especially in the use of money or property.

prodigious: Immense.

profligacy: Shameless viciousness.

profligate: Recklessly wasteful

profuse: Produced or displayed in overabundance.

prolix: Verbose.

propinquity: Nearness.

propitious: Kindly disposed.

prosaic: Unimaginative.

proscribe: To reject, as a teaching or a practice, with condemnation or denunciation.

protuberant: Bulging.

provident: Anticipating and making ready for future wants or emergencies.

prudence: Caution.

puerile: Childish.

pugnacious: Quarrelsome.

punctilious: Strictly observant of the rules or forms prescribed by law or custom.

pungency: The quality of affecting the sense of smell.

pusillanimous: Without spirit or bravery.

pyre: A heap of combustibles arranged for burning a dead body.

Page 60: TABE PERT FCAT Reading and Writing Practice    MmAcVcErReIaCKS

qualm: A fit of nausea.

quandary: A puzzling predicament.

quibble: An utterly trivial distinction or objection.

quiescence: Being quiet, still, or at rest; inactive

quiescent: Being in a state of repose or inaction.

Quixotic: Chivalrous or romantic to a ridiculous or extravagant degree.

quotidian: Of an everyday character; ordinary.

raconteur: A person skilled in telling stories.

ramify: To divide or subdivide into branches or subdivisions.

rapacious: Sieze by force, avaricious

raucous: Harsh.

reactionary: Pertaining to, of the nature of, causing, or favoring reaction.

rebuff: A peremptory or unexpected rejection of advances or approaches.

recalcitrant: Marked by stubborn resistance.

recant: To withdraw formally one's belief (in something previously believed or maintained).

reciprocity: Equal mutual rights and benefits granted and enjoyed.

recluse: One who lives in retirement or seclusion.

recondite: Incomprehensible to one of ordinary understanding.

recrudescent: Becoming raw or sore again.

recuperate: To recover.

redoubtable: Formidable.

redress: To set right, as a wrong by compensation or the punishment of the wrong-doer.

refractory: Not amenable to control.

regale: To give unusual pleasure.

regicide: The killing of a king or sovereign.

reiterate: To say or do again and again.

relapse: To suffer a return of a disease after partial recovery.

remonstrate: To present a verbal or written protest to those who have power to right or prevent a wrong.

renovate: To restore after deterioration, as a building.

repast: A meal; figuratively, any refreshment.

repel: To force or keep back in a manner, physically or mentally.

repine: To indulge in fretfulness and faultfinding.

reprobate: One abandoned to depravity and sin.

repudiate: To refuse to have anything to do with.

Page 61: TABE PERT FCAT Reading and Writing Practice    MmAcVcErReIaCKS

repulsive: Grossly offensive.

requisite: Necessary.

requite: To repay either good or evil to, as to a person.

rescind: To make void, as an act, by the enacting authority or a superior authority.

resilience: The power of springing back to a former position

resonance: Able to reinforce sound by sympathetic vibrations.

respite: Interval of rest.

restive: Resisting control.

retinue: The group of people who accompany an important person during travels.

revere: To regard with worshipful veneration.

reverent: Humble.

ribald: Indulging in or manifesting coarse indecency or obscenity.

risible: Capable of exciting laughter.

rotund: Round from fullness or plumpness.

ruffian: A lawless or recklessly brutal fellow.

ruminate: To chew over again, as food previously swallowed and regurgitated.

sagacious: Able to discern and distinguish with wise perception.

salacious: Having strong sexual desires.

salient: Standing out prominently.

salubrious: Healthful; promoting health.

salutary: Beneficial.

sanction: To approve authoritatively.

sanguine: Cheerfully confident; optimistic.

sardonic: Scornfully or bitterly sarcastic.

satiate: To satisfy fully the appetite or desire of.

satyr: A very lascivious person.

savor: To perceive by taste or smell.

scabbard: The sheath of a sword or similar bladed weapon.

scintilla: The faintest ray.

scribble: Hasty, careless writing.

sedulous: Persevering in effort or endeavor.

sequence: The order in which a number or persons, things, or events follow one another in space or time.

severance: Separation.

shrewd: Characterized by skill at understanding and profiting by circumstances.

Page 62: TABE PERT FCAT Reading and Writing Practice    MmAcVcErReIaCKS

sinecure: Any position having emoluments with few or no duties.

sinuous: Curving in and out.

skiff: Usually, a small light boat propelled by oars.

sluggard: A person habitually lazy or idle.

solace: Comfort in grief, trouble, or calamity.

solvent: Having sufficient funds to pay all debts.

somniferous: Tending to produce sleep.

somnolent: Sleepy.

sonorous: Resonant.

sophistry: Reasoning sound in appearance only, especially when designedly deceptive.

soporific: Causing sleep; also, something that causes sleep.

sordid: Filthy, morally degraded

specious: Plausible.

spurious: Not genuine.

squalid: Having a dirty, mean, poverty-stricken appearance.

stanch: To stop the flowing of; to check.

stigma: A mark of infamy or token of disgrace attaching to a person as the result of evil-doing.

stingy: Cheap, unwilling to spend money.

stolid: Expressing no power of feeling or perceiving.

submerge: To place or plunge under water.

subterfuge: Evasion.

succinct: Concise.

sumptuous: Rich and costly.

supercilious: Exhibiting haughty and careless contempt.

superfluous: Being more than is needed.

supernumerary: Superfluous.

supersede: To displace.

supine: Lying on the back.

supplicate: To beg.

suppress: To prevent from being disclosed or punished.

surcharge: An additional amount charged.

surfeit: To feed to fullness or to satiety.

susceptibility: A specific capability of feeling or emotion.

sybarite: A luxurious person.

Page 63: TABE PERT FCAT Reading and Writing Practice    MmAcVcErReIaCKS

sycophant: A servile flatterer, especially of those in authority or influence.

synopsis: A syllabus or summary.

taciturn: Disinclined to conversation.

taut: Stretched tight.

temerity: Foolhardy disregard of danger; recklessness.

terse: Pithy.

timorous: Lacking courage.

torpid: Dull; sluggish; inactive.

torrid: Excessively hot.

tortuous: Abounding in irregular bends or turns.

tractable: Easily led or controlled.

transgress: To break a law.

transient: One who or that which is only of temporary existence.

transitory: Existing for a short time only.

travail: Hard or agonizing labor.

travesty: A grotesque imitation.

trenchant: Cutting deeply and quickly.

trepidation: Nervous uncertainty of feeling.

trite: Made commonplace by frequent repetition.

truculence: Ferocity.

truculent: Having the character or the spirit of a savage.

turbid: In a state of turmoil; muddled

turgid: Swollen.

turpitude: Depravity.

tutelage: The act of training or the state of being under instruction.

tyro: One slightly skilled in or acquainted with any trade or profession.

ubiquitous: Being present everywhere.

ulterior: Not so pertinent as something else to the matter spoken of.

umbrage: A sense of injury.

unctuous: Oily.

undermine: To subvert in an underhand way.

undulate: To move like a wave or in waves.

untoward: Causing annoyance or hindrance.

upbraid: To reproach as deserving blame.

Page 64: TABE PERT FCAT Reading and Writing Practice    MmAcVcErReIaCKS

vagary: A sudden desire or action

vainglory: Excessive, pretentious, and demonstrative vanity.

valorous: Courageous.

vapid: Having lost sparkling quality and flavor.

variegated: Having marks or patches of different colors; also, varied.

vehement: Very eager or urgent.

venal: Mercenary, corrupt.

veneer: Outside show or elegance.

venial: That may be pardoned or forgiven, a forgivable sin.

veracious: Habitually disposed to speak the truth.

veracity: Truthfulness.

verbiage: Use of many words without necessity.

verbose: Wordy.

verdant: Green with vegetation.

veritable: Real; true; genuine.

vestige: A visible trace, mark, or impression, of something absent, lost, or gone.

vicissitude: A change, especially a complete change, of condition or circumstances, as of fortune.

vigilance: Alert and intent mental watchfulness in guarding against danger.

vigilant: Being on the alert to discover and ward off danger or insure safety.

virago: Loud talkative women, strong statured women

virtu: Rare, curious, or beautiful quality.

visage: The face, countenance, or look of a person.

vitiate: To contaminate.

vituperate: To overwhelm with wordy abuse.

vivify: To endue with life.

vociferous: Making a loud outcry.

volatile: Changeable.

voluble: Having great fluency in speaking.

wean: To transfer (the young) from dependence on mother's milk to another form of nourishment.

whimsical: Capricious.

winsome: Attractive.

Zeitgeist: The intellectual and moral tendencies that characterize any age or epoch.

READING VOCABULARY

Page 65: TABE PERT FCAT Reading and Writing Practice    MmAcVcErReIaCKS

1. After practice, the girl's softball team stated, “We're famished!” Famished means

A. Fatigued B. Hungry C. Excited D. Ready

2. The newborn baby was enamored with the rattle. Enamored means

A. Fascinated B. Happy C. Unsure what to do D. Aggravated

3. When having a problem, it is best to dissect the situation then act. Dissect means

A. Control B. Discuss C. Ignore D. Analyze

4. The bouncer's countenance discouraged brawls. Countenance means

A. Message B. Presence C. Expression D. Strength

5. The child apprized her father's authority and behaved herself in church. Apprized means

A. Appreciated B. Compromised C. Defied D. Noted

6. The aural component of balance is critical for postural control during ambulation. Aural means related to the

A. Eyes B. Ears C. Nose D. Hands

7. The wound exhibited signs of copious drainage requiring medical intervention. Copious means

A. Minimal B. Clear C. Maximal D. Foul

8. The scientist was able to evoke powerful emotions from her audience. Evoke means

A. Sell B. Calm C. Call forth D. Exaggerate

9. The official exhibited a heedless attitude when dealing with the dignitaries. Heedless mean

A. Thoughtless B. Pleasant C. Friendly D. Bitter

10. The general tried to instill in his troops the hope of victory. Instill means

A. Infuse B. Delay C. Inscribe D. Indict

Page 66: TABE PERT FCAT Reading and Writing Practice    MmAcVcErReIaCKS

11. The winning team of the World Series often has a jovial attitude. Jovial means

A. Merry B. Sad C. Somber D. Laborious

12. A lyre was played in ancient Rome. The lyre is a

A. Stringed instrument in the harp class B. Percussion instrument C. Wind instrument in the wind class D. Rhythmical percussion device

Answer Key 1. B 2. A 3. D 4. C 5. A 6. B 7. C 8. C 9. A 10. A 11. A 12. A 

==========

The Flow of Sentences

1. _____ of the rainbow were _____ against the bright blue sky.

A. Textures, Clear B. Hues, Vivid C. Alabaster, Bright D. Line, Dark E. Hues, Dark

2. The president has a _____ of ______ around him when he makes public appearances.

A. Catalyst, Individuals B. Barrier, Contrast C. Hedge, Protection D. Derrick, Protection E. Derrick, Limits

3. A small selection of terms was found at the back of the textbook. It was a

A. Glossary B. Preface C. Diction D. Kefir E. Prefece

4. The horror movie frightened the children. It was

A. Melancholy B. Dramatic C. Ghastly D. Tragedy E. Comedic

5. The books subject matter was _____ to the _____, and it did not sell.

A. Attractive, Masses B. Limited, People C. Loathsome, Masses D. Colorful, Individual E. Colorful, Massess

6. The kitten was soaked to the _____ from the ____.

Page 67: TABE PERT FCAT Reading and Writing Practice    MmAcVcErReIaCKS

A. Skin, Abyss B. Skin, Craven C. Skin, Storm D. Hide, Abyss E. Hide, Storm

Answer Key 1. B 2. C 3. A 4. C 5. C 6. C 

FIND THE NOUNS

Please select the answer choice that identifies the noun in the sentence.

1. It will take all of your energy and will to be able to walk again.

A. Take B. All C. Yours D. Energy

2. The works of many great poets have been placed on reserve.

A. Many B. Great C. Placed D. Reserve

3. The Brooklyn Bridge was opened in 1883.

A. Bridge B. Was C. Opened D. In

4. Sparta and Athens were enemies during the Peloponnesian War.

A. And B. Were C. During D. War

5. Sharks and lampreys are not true fish because their skeletons are made of cartilage rather than bone.

A. True B. Because C. There D. Bone

6. Joe, have you met your new boss?

A. Have B. Met C. Your D. Boss

7. Sue's parents tried living in the North, but they could not adapt to the cold.

A. North B. But C. Not D. Adapt

8. Mastering basic mathematics is an important goal for younger students.

A. Mastering B. Important C. Younger D. Students

Page 68: TABE PERT FCAT Reading and Writing Practice    MmAcVcErReIaCKS

9. To seize a foreign embassy and its inhabitants is flagrant disregard for diplomatic neutrality.

A. Seize B. Its C. Flagrant D. Neutrality

10. The Trojans' rash decision, to accept the wooden horse, led to their destruction.

A. Their B. Led C. Accept D. Destruction

Answer Key 1. D 2. D 3. A 4. D 5. D 6. D 7. A 8. D 9. D 10. D 

====== COMMON GRAMMAR MISTAKES

Each underlined section corresponds to an answer choice. The first underlined section corresponds to choice A. Please select the answer choice that either contains an error or select choice E which is "No error."

1. Whom did you talk to at the information desk at the airport? No error.

A. A B. B C. C D. D E. E

2. Ellen always got into more trouble than me. No error.

A. A B. B C. C D. D E. E

3. The title of salutatorian goes to whomever has the second highest academic average. No error.

A. A B. B C. C D. D E. E

4. Do you feel good enough to go to the store? No error.

A. A B. B C. C D. D E. E

5. Bolivar, an idol between his contemporaries, has been the inspiration for many modern revolutions. No error.

A. A B. B C. C D. D E. E

6. Birds fly south in the winter threw an instinct not completely understood by scientists. No error.

A. A B. B C. C 

Page 69: TABE PERT FCAT Reading and Writing Practice    MmAcVcErReIaCKS

D. D E. E

7. No animal has yet been discovered that can “see” infrared light with its   eyes. No error.

A. A B. B C. C D. D E. E

8. Lying there in the half-dark of my room, I could see my shelf, with my books- some of them prizes I had won in high school. No error.

A. A B. B C. C D. D E. E

9. The man who sat beside Ben and I was running for the city council. No error.

A. A B. B C. C D. D E. E

10. Whom did you say this package was for? No error.

A. A B. B C. C D. D E. E

11. There isn't scarcely room on the front steps to pose the entire class for a picture. No error.

A. A B. B C. C D. D E. E

12. Haven't none of you seen my dog? No error.

A. A B. B C. C D. D E. E

13. I found the expensive vase broken when I first came in the room. No error.

A. A B. B C. C D. D E. E

14. Mrs. Clement, my English teacher, said that I could of improved my reading comprehension score if I had spent more time reading great literature. No error.

A. A B. B C. C D. D E. E

15. If you sign up as a volunteer for the special olympics, you will find that you receive as much as you give. No error.

A. A B. B C. C D. D E. E

16. “Your themes,” said Ms. Buchanan, will be due in class on September 7; late papers will lose one full grade.” No error.

Page 70: TABE PERT FCAT Reading and Writing Practice    MmAcVcErReIaCKS

A. A B. B C. C D. D E. E

17. What should I do when the computer says, “Sorry, try again?” No error.

A. A B. B C. C D. D E. E

18. “Whose in the office now?” asked Mom. No error.

A. A B. B C. C D. D E. E

19. Parking her car at the depot, Ms. Jones decided to take the bus to town. No error.

A. A B. B C. C D. D E. E

20. In 1936 Adolph Hitler refused to congraduate the great Jesse Owens, winner of four gold medals in the Berlin Olympics. No error.

A. A B. B C. C D. D E. E

21. Preserving   rare and valuable books is one of the challenges facing the Librarian of Congress. No error.

A. A B. B C. C D. D E. E

22. Everyone is excited about graduation because all had worked so hard for it. No error.

A. A B. B C. C D. D E. E

23. Without saying a word, the major gave a nod of ascent. No error.

A. A B. B C. C D. D E. E

24. Just as they were about to go to bed, Jane told her mother, ”Its my turn to wind the clock.” No error.

A. A B. B C. C D. D E. E

Answer Key 1. E 2. D 3. B 4. B 5. A 6. C 7. E 8. E 9. C 10. A 

Page 71: TABE PERT FCAT Reading and Writing Practice    MmAcVcErReIaCKS

11. A 12. B 13. D 14. C 15. A 16. A 17. D 18. A 19. E 20. B 21. D 22. C 23. D 24. C 

==== COMMON USE mistakes part 2

Each underlined section corresponds to an answer choice. The first underlined section corresponds to choice A. Please select the answer choice that either contains an error or select choice E which is "No error."

1. Bill Cosby is a well respected actor and comedian. No error.

A. A B. B C. C D. D E. E

2. Known as Johnny Appleseed, John Chapman distributed   apple seeds and saplings to families bound for thewest. No error.

A. A B. B C. C D. D E. E

3. Dancing to unanimous acclaim in The United States and Europe, she was known for her interpretation of Stranvinsky's Firebird. No error.

A. A B. B C. C D. D E. E

4. Needing a sustained wind for flight, the albatross rarely crosses the equator. No error.

A. A B. B C. C D. D E. E

5. Sparta and Athens, putting aside there own rivalry, fought off the Persians. No error.

A. A B. B C. C D. D E. E

6. Mahalia Jackson, called the greatest potential blue's singer since Bessie Smith, would   sing only religious songs. No error.

A. A B. B C. C D. D E. E

7. Elected to the House of Representatives in 1958, Shirley Chisholm became the first black female member of Congress. No error.

A. A B. B C. C D. D E. E

Page 72: TABE PERT FCAT Reading and Writing Practice    MmAcVcErReIaCKS

8. Put   the ingredients altogether   in a bowl when   you make the cake. No error.

A. A B. B C. C D. D E. E

9. Horatio, Hamlet's loyal friend, wishes to die by his own hand but consents to stay alive so that he can tell Hamlet's story. No error.

A. A B. B C. C D. D E. E

10. We visited Boston Harbor, the site of the Boston Tea Party. No error.

A. A B. B C. C D. D E. E

11. George's mother, Karen, has come with us to the mall. No error.

A. A B. B C. C D. D E. E

12. Marian   Anderson, the first black person employed as a member of the Metropolitan opera, was born February 17, 1902. No error.

A. A B. B C. C D. D E. E

13. The new expenditures will be born by the American taxpayers. No error.

A. A B. B C. C D. D E. E

14. Having joined a volunteer program, Mark helps by reading stories to patients in the children ward. No error.

A. A B. B C. C D. D E. E

15. Mr. French, the superintendent of schools in our district, spoke at the PTA meeting last night. No error.

A. A B. B C. C D. D E. E

16. Specially treated cloths, are best for cleaning eyeglasses lenses. No error.

A. A B. B C. C D. D E. E

17. When he addresses the legislature,   the Governor will have to answer several questions about the budget. No error.

A. A B. B C. C D. D E. E

Page 73: TABE PERT FCAT Reading and Writing Practice    MmAcVcErReIaCKS

18. According to some modern scientists Galileo's observations, which were made possible by the invention of the telescope, was long overdue. No error.

A. A B. B C. C D. D E. E

19. Follow the marked route or you will surely loose your way. No error.

A. A B. B C. C D. D E. E

20. Shakespeare tried to show us the complexity of the human's mind. No error.

A. A B. B C. C D. D E. E

21. The governor announced that the roof of the capital needs repair. No error.

A. A B. B C. C D. D E. E

22. I am reading about Geraldine Ferraro,   the first female vice-presidential candidate of a major party. No error.

A. A B. B C. C D. D E. E

23. After the eggs are weighed, they are separated by weight into five groups. No error.

A. A B. B C. C D. D E. E

24. The President decorated the astronaut, who had orbited the earth. No error.

A. A B. B C. C D. D E. E

Answer Key 1. B 2. D 3. B 4. D 5. A 6. A 7. E 8. B 9. C 10. B 11. B 12. D 13. B 14. D 15. E 16. E 17. B 18. D 19. D 20. D 21. C 22. E 23. D 24. C 

Page 74: TABE PERT FCAT Reading and Writing Practice    MmAcVcErReIaCKS

===

Common Missteaks part 3

Each underlined section corresponds to an answer choice. The first underlined section corresponds to choice A. Please select the answer choice that either contains an error or select choice E which is "No error."

1. Her novel is an American classic about a young girl who she called   Billie Joe. No error.

A. A B. B C. C D. D E. E

2. Hours of driving laid ahead of us   before we could complete the trip. No error.

A. A B. B C. B D. D E. E

3. Both Thoreau and Emerson were abolitionists;   they had spoken out against the evils of slavery. No error.

A. A B. B C. C D. D E. E

4. Yesterday our classroom computer was acting rather strangely. No error.

A. A B. B C. C D. D E. E

5. We decided against medical careers because science had always given Sue and I trouble. No error.

A. A B. B C. C D. D E. E

6. After a whole afternoon of playing basketball, I sleep very sound at night. No error.

A. A B. B C. C D. D E. E

7. In the winter I usually like skiing and to skate. No error.

A. A B. B C. C D. D E. E

8. The judge showed early signs of genius; for example, she began law school when she was only nineteen. No error.

A. A B. B C. C D. D E. E

9. The general greeted his former mess sergeant, whom he had not seen in many years. No error.

A. A B. B 

Page 75: TABE PERT FCAT Reading and Writing Practice    MmAcVcErReIaCKS

C. C D. D E. E

10. Lately, many of the committee's suggestions has been rejected. No error.

A. A B. B C. C D. D E. E

11. The Supreme Court decision, along with discussions of the Justices'   opinions are printed in today's newspaper. No error.

A. A B. B C. C D. D E. E

12. Neither Sue or Carol thinks they is ready to write the final draft. No error.

A. A B. B C. C D. D E. E

13. These notebooks have laid on the desk all week; please put them away. No error.

A. A B. B C. C D. D E. E

14. You had better leave for home quick if you want to avoid the storm. No error.

A. A B. B C. C D. D E. E

15. Monet used short brush strokes to create the allusion of moving water. No error.

A. A B. B C. C D. D E. E

16. Did Mr. Smith infer that our research paper had to be an American author? No error.

A. A B. B C. C D. D E. E

17. A spoonerism is a slip of a tongue in which the beginning sounds of two words are switched. No error.

A. A B. B C. C D. D E. E

18. Being that I was sick, I missed a whole week of classes; luckily I copied all the lecture notes from Linda. No error.

A. A B. B C. C D. D E. E

19. In 1912 Congress set a limit on the total amount of representatives in the House of Representatives. No error.

Page 76: TABE PERT FCAT Reading and Writing Practice    MmAcVcErReIaCKS

A. A B. B C. C D. D E. E

20. In science class we learned about digestion, respiration, circulation; and etc. No error.

A. A B. B C. C D. D E. E

21. Clean air, as well as clean lakes and rivers concern   all the citizens of the United States. No error.

A. A B. B C. C D. D E. E

22. A school handbook is given to everyone who enrolls   in our school. No error.

A. A B. B C. C D. D E. E

23. Yes, records and compact discs have the same sound in my opinion. No error.

A. A B. B C. C D. D E. E

24. Today the talent committee will   audition Joe, Steve, and myself. No error.

A. A B. B C. C D. D E. E

Answer Key 1. C 2. A 3. E 4. E 5. D 6. D 7. C 8. E 9. E 10. C 11. D 12. B 13. A 14. C 15. D 16. A 17. E 18. D 19. B 20. D 21. B 22. E 23. E 24. D 

=========

Verb practice

Page 77: TABE PERT FCAT Reading and Writing Practice    MmAcVcErReIaCKS

Please select the answer choice that identifies the verb in the sentence.

1. The interior temperatures of even the coolest stars are measured in millions of degrees.

A. Coolest B. Of even C. Are measured D. In millions

2. Thomas Edison tried many filaments for his incandescent lamp.

A. Many B. For his C. Filaments D. Tried

3. Jill sets the plates on the table.

A. The B. Plates C. Table D. Sets

4. The child's balloon was slowly rising into the sky.

A. Rising B. Slowly C. Into D. Balloon

5. The shoes were still lying where Ethan had left them.

A. Still B. Were C. Them D. Shoes

6. Several changes in classroom procedures were effected by the new principal.

A. Changes B. In C. By D. Effected

7. The soaked papers were laid in the sunlight.

A. Soaked B. Papers C. Laid D. In the

8. The letter from the teacher implied that the child was not turning in his work.

A. From B. Not C. His D. Turning

9. Luke didn't mean to hurt you during the baseball game.

A. Baseball B. During C. Joe D. Mean

10. Amber used to recite the alphabet in Chinese.

A. The B. Alphabet C. He D. Recite

Answer Key 1. C 2. D 3. D 4. A 

Page 78: TABE PERT FCAT Reading and Writing Practice    MmAcVcErReIaCKS

5. B 6. D 7. C 8. D 9. D 10. D 

WRITING PRACTICE

Each underlined section corresponds to an answer choice. The first underlined section corresponds to choice A. Please select the answer choice that either contains an error or select choice E which is "No error."

1. Are you all ready for the English exam?” asked mom. No error.

A. A B. B C. C D. D E. E

2. According to the fire marshall's statistics, smoking cigarettes in bed are the cause of many tragic fires. No error.

A. A B. B C. C D. D E. E

3. Signs of decay that should be recognized by every citizen  includes  oil spills along the shoreline as well as the absence of wildlife. No error.

A. A B. B C. C D. D E. E

4. Either Philip or Joe will always finish their trigonometry  homework  in class. No error.

A. A B. B C. C D. D E. E

5. Who else beside Jim Smith won a scholarship? No error.

A. A B. B C. C D. D E. E

6. I appreciate you helping me in the kitchen when my mom is at work. No error.

A. A B. B C. C D. D E. E

7. I am certain that the English scholarship winner will be hear. No error.

A. A B. B C. C D. D E. E

8. Talking with my grandfather about World War II effected me more than I thought it would have. No error.

A. A B. B C. C 

Page 79: TABE PERT FCAT Reading and Writing Practice    MmAcVcErReIaCKS

D. D E. E

9. The President is all together opposed to the bill. No error.

A. A B. B C. C D. D E. E

10. Tsetse flies, which carry the dreaded disease called sleeping sickness attacks both human and cattle. No error.

A. A B. B C. C D. D E. E

11. According to Aunt Grace, this kind of a sweater is the warmest for the winter months. No error.

A. A B. B C. C D. D E. E

12. Many reading lists for high-school students include translations from Scandinavian writers such as Lagerlog, Undset, and Ibsen. No error.

A. A B. B C. C D. D E. E

13. Will they ascent to our proposal? No error.

A. A B. B C. C D. D E. E

14. Earlier synthetics, such as Bakelite, have went the way of the dinosaur. No error.

A. A B. B C. C D. D E. E

15. After the attack by the U-boat, the Lusitania sunk in eighteen minutes; over one thousand people drowned. No error.

A. A B. B C. C D. D E. E

16. The teacher discussed the rule that the compliment of thirty degrees is sixty degrees. No error.

A. A B. B C. C D. D E. E

17. As you approach the intersection, drive as cautious as you can. No error.

A. A B. B C. C D. D E. E

18. The queen's counsil could not agree on the correct policy. No error.

Page 80: TABE PERT FCAT Reading and Writing Practice    MmAcVcErReIaCKS

A. A B. B C. C D. D E. E

19. Scury, one of the diseases of modern science has conquered result from a lack of vitamin C. No error.

A. A B. B C. C D. D E. E

20. Some people say that compact discs  offers  a brighter treble and true base than conventional records. No error.

A. A B. B C. C D. D E. E

21. I learned about the Roaring Twenties from my great-grandmother and he. No error.

A. A B. B C. C D. D E. E

22. Mrs. Jones was formally   head of the history department at Center High School. No error.

A. A B. B C. C D. D E. E

23. The British navy, members of which are called “limeys,” were responsible for first using limes to prevent scury during long sea voyages. No error.

A. A B. B C. C D. D E. E

24. Please write and tell my mother and I about your trip to San Francisco. No error.

A. A B. B C. C D. D E. E

Answer Key 1. D 2. D 3. B 4. C 5. C 6. E 7. D 8. B 9. C 10. D 11. B 12. E 13. C 14. C 15. B 16. B 17. B 18. B 19. D 20. B 21. D 22. A 23. D 24. C 

-----------

Page 81: TABE PERT FCAT Reading and Writing Practice    MmAcVcErReIaCKS

Vocabulary ONE

1. The prince abdicated the crown and returned to his castle. Abdicated means

A. Gave up B. Sold C. Reinvested into D. Auctioned

2. The convicted criminal absconded prior to the sentencing phase of the trial. Absconded means

A. Touched the jury B. Reported immediately C. Left after discussion D. Departed suddenly

3. The aural component of balance is critical for postural control during ambulation. Aural means related to the

A. Eye B. Ear C. Nose D. Mouth

4. The old man was benevolent with his fortune. Benevolent means

A. Secretive B. Stingy C. Kind D. Careful

5. The extra dirt was a key buttress to the foundation. Buttress means

A. Limiting factor B. Support C. Overwhelming condition D. Obstacle

6. The cathode of a battery was removed. Cathode means

A. Positive pole B. Negative pole C. Neutral pole D. Opposite pole

7. The doctor was known as a charlatan over the years of his practice. Charlatan means

A. Quack B. Knowledgeable physician C. Procedural physician D. Medical examiner

8. The wound exhibited signs of copious drainage requiring medical intervention. Copious means

A. Minimal B. Clear C. Maximal D. Foul

9. The attorney accused the witness of defaming the defendant. Defaming means

A. Killing B. Badgering C. Suffocating D. Slandering

10. The detective was able to derive the facts of the case. Derive means

A. Desist B. Deter C. Devise D. Deduce

11. The scientist was able to evoke powerful emotions from her audience. Evoke means

Page 82: TABE PERT FCAT Reading and Writing Practice    MmAcVcErReIaCKS

A. Sell B. Calm C. Call forth D. Exaggerate

12. The judge was fallible during deliberation. Fallible means

A. Careful not to err. B. Falsely accused C. Loyal to his supporters D. Capable of mistakes

13. The chemist collected the germane data during the experiment. Germane means

A. Relevant B. Obscure C. Limited D. Usual

14. The desperados held up in a grotto in New Mexico during the escape. Grotto means

A. Large cave B. Small cavern C. Hotel D. Motel

15. The official exhibited a heedless attitude when dealing with the dignitaries. Heedless means

A. Thoughtless B. Pleasant C. Friendly D. Bitter

16. The Sherman tank commander noted innumerable troops moving forward against his position. Innumerable means

A. Limited B. Weary C. Countless D. Harmless

17. The general tried to instill in his troops the hope of victory. Instill means

A. Infuse B. Delay C. Inscribe D. Indict

18. The winning team of the World Series often has a jovial attitude. Jovial means

A. Merry B. Sad C. Somber D. Laborious

19. The plant entered the latent phase of development in the fall. Latent means

A. First B. Growth C. Last D. Dormant

20. The yacht club members were excited about conditions on the loch. Loch means

A. Water B. Lake C. Gulf D. Ocean

Answer Key 1. A 2. D 3. B 4. C 5. B 6. B 7. A 8. C 

Page 83: TABE PERT FCAT Reading and Writing Practice    MmAcVcErReIaCKS

9. D 10. D 11. C 12. D 13. A 14. B 15. A 16. C 17. A 18. A 19. D 20. B 

Vocabulary TWO

1. A lyre was played in ancient Rome. Lyre means

A. Stringed instrument in the harp class B. Percussion instrument C. Wind instrument in the wind class D. Rhythmical percussion device

2. The labyrinth caused confusion to the attacking troops. Labyrinth means

A. Sound B. Noise C. Maze D. Bulwarks

3. The wound was necrotic when examined. Necrotic means

A. Healing B. Dying tissue C. Nauseating D. Infinite

4. The defendant exhibited a peevish appearance. Peevish means

A. Immovable B. Guilty C. Not guilty D. Irritable

5. The band director was an expert a playing the piccolo. Piccolo means

A. Small flute B. Large flute C. Small drum D. Small triangle

6. The renter was remiss about the rent. Remiss means

A. Timely B. Negligent C. Irritable D. Impoverished

7. The old man was known for sapient knowledge. Sapient means

A. Useless B. Possessing wisdom C. Perceptual D. Limited

8. The inventor created several specious ideas to solve the problem. Specious means

A. Inspired B. Insufficient C. Limited D. Falsely plausible

Page 84: TABE PERT FCAT Reading and Writing Practice    MmAcVcErReIaCKS

9. The tolerant attitude of the audience was appreciated. Tolerant means

A. Tireless B. Calm C. Indulgent D. Laborious

10. The verbose language used by the English teacher was tiresome to the class. Verbose means

A. Wordy B. Expressive C. Limited D. Punitive

11. Susan's abhorrence of darkness prevents her from leaving her house at night. Abhorrence means

A. Rationale B. Hatred C. Tremor D. Belief

12. The girl displayed distraught behavior when she found out her puppy was injured. Distraught means.

A. Reckless B. Shifty C. Distressed D. Unreasonable

13. The somber crowd mourned the loss of their leader. Somber means

A. Angry B. Bitter C. Melancholy D. Excited

14. At age 65, the CEO of the company was retiring. He felt he had reached the acme of his profession. Acme means

A. Highest point B. End C. Bottom D. Entrance

15. The genteel southern girl was known for her behavior. Genteel means

A. Refined B. Ambiguous C. Smug D. Loathsome

16. The mother attempted to mollify her son with toys. Mollify means

A. Teach B. Threatening C. Soothe D. Distract

17. Some people accused John of thinking too much. He would sometimes ponder on a subject for months at a time. Ponder means

A. Resolve B. Meditate C. Discuss D. Fret

18. The young artist had an unbridled passion for watercolors. Unbridled means

A. Unrestrained B. Unequaled C. Underachieved D. Distressed

19. The zephyr kept the students cool while they sat outside studying. Zephyr means

A. Cloud B. Tree 

Page 85: TABE PERT FCAT Reading and Writing Practice    MmAcVcErReIaCKS

C. Shade D. Wind

20. The pianist played his rendition of a sonata. Sonata means

A. Instrumental composition B. Piano C. Play D. Vocal score

Answer Key 1. A 2. C 3. B 4. D 5. A 6. B 7. B 8. D 9. C 10. A 11. B 12. C 13. C 14. A 15. A 16. C 17. B 18. A 19. D 20. A 

GENERAL KNOWLEDGE and BASIC EDUCATED PEOPLE’s info

1. Which of the following does not have anything to do with the process of blood flow in the human body?

a. Arteriesb. Ventriclec. Hypothalamusd. Veins

2. The human body uses fats and carbohydrates to sustain itself but it uses ___________ for growth, maintenance, and healing.

a. calciumb. vitamin Cc. proteind. hydrogen peroxide

3. Pick out the reaction or phenomenon which best describes a chemical process.

a. A rusting metal pipeb. Dry icec. Soil erosiond. Helium mixed with oxygen

4. The process by which plants convert carbon dioxide and water into plant sugars and the release of oxygen is called ____________.

a. respirationb. thawingc. photosynthesisd. composting

5. Which one of the creatures shown below is a "scavenger"?

a. horseb. crowc. squirreld. cow

6. Pick the answer from the following options that best explains the concept of "inertia."

a. The tendency of an object in motion to gain speedb. The tendency of an object at rest to remain at restc. The tendency of matter to disintegrated. The tendency of a moving object to change direction

Page 86: TABE PERT FCAT Reading and Writing Practice    MmAcVcErReIaCKS

7. What is the location of the earth during a lunar eclipse?

a. Between the sun and the moonb. Behind the sunc. At its apogeed. At a point 180 degrees from North

answers

General Science Knowledge Practice Questions Answers

1. "C" is the answer that doesn't fit. The hypothalamus is located just above the brain stem and is an early developing part of the brain. Arteries and veins are conduits for blood flow. The left and right ventricles are chambers in the heart.

2. The correct answer is "c" because protein is absolutely necessary for muscle, tissue, and organ growth and healing. Proteins are often referred to as the "building blocks" of the human body.

3. The correct answer is "a." A rusting pipe involves a chemical reaction in which the iron pipe oxidizes or turns into iron oxide. Iron oxide is formed when the oxygen in water combines with iron. In the evaporation of dry ice, the chemical composition does not change and the individual elements of the ice remain the same. Helium is an inert gas and doesn't combine with other elements, so there is no chemical change in this answer either. Soil erosion is not a chemical action; it is a result of the physical action of water and wind on the earth's surface.

4. The correct answer is "c" or photosynthesis. The other choices are either misleading or wrong. Thawing is the opposite of freezing. Composting is a gardening practice where plant material is broken down into humus or fertilizers. Respiration refers to animal activity, not plant activity.

5. The correct answer is "b". Crows can often be seen scavenging or pecking at dead animals on the road or elsewhere. In that regard, they are much like the better known scavenger, the vulture. Squirrels, cows, and horses eat plant materials like grass or acorns but grazing is not thought of as scavenging in the scientific sense of the word.

6. The best answer is "b." Inertia is the tendency of an object to remain at rest unless sufficient force is put upon it to move the object. The other choices are nonsensical. Objects in motion don't gain speed unless additional force is applied. Matter sometimes disintegrates but that has nothing to do with inertia. Choice "d" is a wrong choice because objects in motion tend to keep moving in the same direction unless a directional force is applied.

7. During the lunar eclipse, the shadow of the earth falls on the moon and it appears to darken and disappear. The only way the earth's shadow could fall on the moon is for the earth to be in a straight line between the sun and the moon. All three orbs must be lined up, with the earth between the sun and the moon. Therefore, "a" is the correct choice.

----------

Word Knowledge Practice Questions

Words that mean the same thing are called synonyms. In the following sentences, select the word or group of words that most closely means the same as the underlined word.

1. According to Army policy, every recruit will be accountable for the maintenance of his private living area.

a. applaudedb. responsiblec. rewardedd. punished

2. Before signing the purchase contract, the car buyer decided to scrutinize the payment term agreement.

a. examineb. confusec. comprehendd. access

3. Two soldiers were involved in an animated discussion regarding the correct way to set up a perimeter defense.

a. livelyb. sorrowfulc. briefd. humanitarian

4. A military conflict was avoided because of a diplomatic effort by the much respected ambassador.

Page 87: TABE PERT FCAT Reading and Writing Practice    MmAcVcErReIaCKS

a. belligerentb. tactfulc. submissived. angry

5. The much decorated Drill Sergeant was emulated by other unit soldiers.

a. punishedb. praisedc. fashionedd. imitated

6. The authorities explained the need for compliance with all regulations.

a. defianceb. confidencec. enthusiasmd. obedience

7. The defenders of the citadel were unmatched in courage and tenacity.

a. ferocityb. doggednessc. timidityd. flexibility

8. A mutation is something that

a. has been transformed from its normal state.b. has been silenced.c. has been gained.d. has been corroded.

Word Knowledge Practice Question Answers

1. Choice "b" is the correct answer. "Accountable" most nearly means responsible.

2. Choice "a" is correct here. To scrutinize something is to examine it closely. The word "confuse" can be ruled out by the process of elimination. "Confuse" doesn't fit the context of the sentence. The salesperson may want to confuse the contract but not the "car buyer."

3. An animated conversation is also a lively conversation. Therefore, choice "a" is the correct answer here. Context doesn't help you much in this one; you have to know that "animated" can mean lively or excited. Since you know what a "perimeter defense" is (from watching movies, perhaps), you may eliminate "humanitarian."

4. "Diplomatic" can mean tactful. Using tact in your relationships with others means that you speak and behave in a way that doesn't offend other persons with a different point of view. That "diplomacy" is necessary when an ambassador talks with a world leader. But diplomacy doesn't mean being submissive, so don't be distracted by that wrong answer. The correct choice is "b."

5. To "emulate" someone means to imitate them or to act and behave in a similar way. The reason the Army and the Marines use drill sergeants is so that the recruits can "emulate" their fine example. The correct answer is "d." The words "punished" and "praised" can be ruled out from context. The "unit soldiers" do not often praise or punish the Drill Sergeants. It's usually the other way around.

6. Choice "d" is the one to pick here. "Compliance" most nearly means obedience. If you're compliant with the rules of your school, it means that you have obeyed its rules. "Defiance" can be eliminated because "authorities" don't generally recommend it with regard to their regulations. That would be like the principal of your high school telling you to defy all school rules.

7. The correct answer is "b" doggedness. It means the defenders will stick to the task with unrelenting focus. The word "doggedness" depends on the imagery of a dog holding onto a tug toy and being unwilling to let go. Another close meaning of the word might be "determination."

8. The best choice is "a." Genes often mutate and cause changes in the offspring. Cells can mutate and become other types of cells, even dangerous ones. Choice a" could possibly mislead you because "mutation" contains the prefix "mute," referring to silence or an inability to speak.

==========

Paragraph Comprehension Practice Questions

Questions 1-2The last time the American economy received as much attention as it is now getting was in the 1929 stock market crash and the Great Depression that followed it. There are some similarities in comparing that catastrophic event of long ago to the troubles with the economy today. American and world stock markets have fallen to record lows. Unemployment continues to rise. Many people do not have the money to pay their bills. There has been an increase in the cost of food and other basic living items. However, in today's economy, people have advantages that they did not have in the 1930s when there were no unemployment benefits. Nor were there

Page 88: TABE PERT FCAT Reading and Writing Practice    MmAcVcErReIaCKS

the vast array of social and governmental agencies that offer help to needy families. There was no state Medicaid. There was no Social Security. Certainly, there was no giant bailout of the banking industry as there is today.

1. The main theme of the paragraph is

a. a comparison of the economy in two different eras.b. the Great Depression.c. the cost of food and shelter.d. the great number of social and governmental agencies.

2. In the opinion of the writer of the above paragraph,

a. things are tougher now than in the 1930s.b. food costs are higher than ever before.c. we are in the midst of another Great Depression.d. society is more prepared to deal with economic downturns today than it was during the Great Depression.

Questions 3-5Having a pet is a big responsibility because animals do not often respond the way people do. Therefore, pet owners are often surprised by the behavior of their pets. Dogs are a good example of a common household pet which requires a sense of responsibility on the part of the dog's owner.

A responsible dog owner should be prepared to walk the animal at least twice a day. The larger breed and more active dogs require about forty minutes of exercise each day. Failing to exercise your dog can lead to bad indoor behavior because, like humans, animals get bored. When dogs are bored, they look for things to do, such as chewing, or barking, or pestering family members to play. Ignoring your dog by not exercising it and confusing it with a variety of erratic commands can even make your pet dog very spoiled and possibly aggressive. The best method of turning your dog into a friend you can live with is to hold to a regular schedule of feeding, exercise, play, and quiet time. These rules are the same ones that apply to human children.

3. According to the article above,

a. dogs learn more quickly than adults.b. irresponsible dog owners are lazy individuals.c. a neglected dog is a troublesome dog that causes problems in the household.d. walking a dog eliminates all dog problems.

4. According to the article above,

a. bored dogs can exhibit a variety of unpleasant behaviors, including chewing, excessive barking, and even aggression. b. regular exercise and feeding of your pet is a panacea for all your ills. c. poor dog behavior is caused by friendly dog owners.d. memorizing proper animal commands is key to good dog behavior.

5. The meaning of the word "erratic" in the passage above most closely means

a. inconsistent.b. angry.c. bold.d. congruent.

Questions 6-9The material presented below is an informational document. Read it carefully and choose the best answer.It is clear from most sources that the Indians, at the time of early settler contact, had their villages on both sides of the Upper Delaware River. We are lucky to have a great deal of archeological evidence from local Indian sites due to the hurried excavations during 1967 and 1968. The reason for the hurried excavations was that a plan was afoot to inundate the entire area in the creation of a huge dam project. Many of the dig sites and discoveries during this period can be attributed to the efforts of environmentalists, archeologists, scientists, and just plain folks. Had the dam actually been constructed, the entire natural history museum, and a large part of the Native American heritage of Eastern Pennsylvania, would have been lost forever.

6. This excerpt from a local history of Eastern Pennsylvania is about

a. the abandonment of a dam project.b. the disintegration of the Native American population.c. the building of dams and other water projects.d. the preservation of a historic site.

7. The word "inundate" in the second paragraph most nearly means the same as

a. intubate.b. plow.c. regulate.d. flood.

8. The chief point of the writer of the passage above is that

a. building a dam would provide jobs.b. deciding against building a dam preserved an important historical site. c. building a dam would preserve Indian culture.d. Indians were able to stop a dam project.

Page 89: TABE PERT FCAT Reading and Writing Practice    MmAcVcErReIaCKS

9. According to the passage, the reason for the hurried archeological excavations in 1967 and 1968 was that

a. archeologists needed to dig quickly because after the area was flooded by the dam's waters, historical research would be impossible.b. 1967 and 1968 were the best years for archeology.c. the government ordered the sites to be excavated during those years.d. a team of archeologists petitioned the government to dig during those years. 

==========

Paragraph Comprehension Practice Question Answers

1. The correct answer is "a." The main theme of the selection is a comparison of the economy of the 30s with the economy of today. While other topics are mentioned in the paragraph, they do not represent the main overall theme. The cost of food and shelter, the number of social and governmental agencies, and the Great Depression are all part of the comparison of the two eras.

2. The best answer is "d". To get this one right, however, you must remember what we said about "inference" questions on the Paragraph Comprehension section of the ASVAB. Inference questions are not specifically spelled out. You have to read the passage and interpret the opinions expressed by the writer. The passage mentions that we have unemployment compensation, social security, and Medicaid to support some of the needy. Obviously, these agencies will provide help to some needy people who lived without them in the early 1930s. Therefore, we have more mechanisms in place today to deal with economic downturns.

3. The correct answer is "c." There is a cause and effect aspect to the question. The passages don't say that any one thing is responsible for a dog's poor behavior. The message is that neglect of a dog through lack of exercise and regular training will make any dog a difficult household companion. Choice "b" might be tempting because it's true, but there is no support for that choice in the paragraphs.

4. Choice "a" is the correct one. The paragraph indicates a cause and effect relationship between a bored dog and troubles such as chewing, barking, aggression, or other spoiled behavior. With regard to option "b", it is true that the writer advises regular exercise and feeding, but nowhere does it say that this is a cure for "all your ills." Choices "c" and "d" are irrelevant to a dog and to the score on your ASVAB tests.

5. This is a simple word meaning question. The word "erratic" most closely means "inconsistent" so choice "a" is correct. If you didn't know the meaning of the word, you might have guessed it from context. What kind of commands would confuse the dog? Inconsistent commands confuse soldiers and other people and dogs.

6. The correct answer is "d", the preservation of a historic site. There is mention of archeology and a dam project that would have flooded the area where the archeological site is located. The implications of this are that an important historical site would have been destroyed, along with the knowledge that might come from it. The article does say the dam project was abandoned but this is not the central issue or main theme. By the same token, the Native Americans were driven away but neither is this the central theme.

7. The correct answer is "d." You can pick that up from context too, since a dam would result in the flooding of the area where the dam is to be built. You can use the process of elimination to rule out the word "plow" because it would be ridiculous to plow and farm an area where you are building a dam. "Intubate" is a medical word involving the insertion of a tube ("tube" is within the word itself) and "regulate" doesn't make sense either.

8. The best answer is "b." The main theme of the passage is that the proposed dam would have destroyed the remnants of ancient Indian villages "on both sides of the Delaware River." The author's focus is on the importance of preserving the archeological record of the Indian settlements where the dam would have been built. The other answers are unsupported by the text.

9. The correct answer is "a." The only answer that makes sense is that area flooding would have made historical excavation virtually impossible. The other choices are not supported by any information in the passage. Choice "b" is absurd because there are no particular years that favor archeology.

======

Auto and Shop Knowledge Practice Questions

On the Auto and Shop Knowledge sections of the ASVAB, you will be expected to answer a variety of multiple choice questions concerning auto mechanics and wood shop. Pick the best answer on the questions below:

1. What system of an automobile or truck determines the vehicle's cornering ability and ride stiffness?

a. Steering systemb. Braking systemc. Electrical systemd. Suspension system

2. The purpose of a transfer case is to

a. make a vehicle ride more smoothly.b. make the steering more responsive to driver input.

Page 90: TABE PERT FCAT Reading and Writing Practice    MmAcVcErReIaCKS

c. distribute power to front and rear wheels in a 4 x 4 vehicle.d. shorten the braking distance.

3. The reason a particular quarter inch nut may not fit a particular quarter inch bolt is because

a. they may be of different thread classifications.b. a quarter inch bolt is incompatible with a quarter inch nut of the same size.c. the metal alloys from which the nut and bolt are made may cause the nut to "seize." d. quarter-inch bolts require a nut of a slightly larger size to fit.

4. The "kerf" is

a. a type of wood file.b. the angle of the blade on a circular saw.c. a slot or cut made by the blade of a saw as it cuts into the wood.d. a term of measurement used in vehicle wheel alignment.

5. It would be better to use thick viscosity motor oil in

a. cold climates (makes vehicle startups easier).b. tropical climates (engine heat build-up).c. Eastern United States.d. four-wheel drive vehicles.

6. The part of the motor vehicle electric system which distributes the "spark" to the various combustion cylinders is the

a. battery.b. rotor and distributor assembly.c. injection system.d. ignition coil.

7. A "punch" is used for

a. hammering knots from wooden objects.b. marking metal or wooden objects to prepare for drilling or other activities and for driving small headed nails.c. filing the sharp edges of metal or wooden objects.d. drilling holes.

8. For a better grip on a "stubborn" fastener nut, it is better to use

a. an adjustable wrench.b. an open-end wrench.c. a box-end wrench.d. a pipe wrench.

Auto and Shop Knowledge Practice Question Answers

1. The correct answer is "d," the suspension system. Don't be misled by the possibility of choosing the steering system. While steering does affect directional stability, the main auto system which determines ride comfort, stiffness, softness, and ability to stay on track when cornering is the suspension system.

2. The correct answer is "c." The transfer case shifts power from rear wheels to front wheels in a four-wheeled drive vehicle. The mechanism is positioned between the front and rear drive axles.

3. The correct answer is "a." Threading refers to the grooves around the inside of the nut and on the outside of the bolt. There are two basic thread classifications. Coarse threads have a smaller number of "turns" or grooves and fine threads have a greater number. Coarse threaded bolts require coarse threaded nuts. Fine threaded bolts require fine threaded nuts in order to be compatible. Metal alloys have nothing to do with thread specifications. Answer choice "d" defies common sense, as does choice "b."

4. The correct answer is "c." The term is commonly used in wood working plans and schematics.

5. The best choice answer is "b." While many oil producers make a multiple viscosity motor oil that can be used in both cold and warm climates, you can rule out choice "a" because a higher viscosity or "thick" motor oil tends to become even thicker in cold weather and would make the vehicle even harder to start. Choice "d" is misleading because the motor oil you use would be the same for four or two wheel drive vehicles. Choice "c" is misleading because the Eastern United States has several types of climate.

6. The best answer is "b," the rotor and distributor assembly system. For the engine to run smoothly, the cylinders must fire in a particular order. The rotor and distributor accomplish this by sending the "spark" to the cylinders at closely timed intervals. Should the rotor and distributor assembly fail to do this correctly, the engine combustion would be very rough and cause backfiring and other problems.

7. The correct answer is "b." A punch can be used for putting marks on metal objects so that they can be more easily drilled. A nail punch is often used to drive small trim nails deeply into the wood so that the nails cannot be seen.

8. The word "pipe" in "pipe wrench" should eliminate "d" if common sense doesn't. Adjustable wrenches are sometimes hard to fit onto stubborn nuts and often slip so that's not a good choice. The same applies to open-end wrenches. On the other hand, a box-end wrench entirely surrounds the nut and fits snugly. So long as you've got the right size box-end wrench, it's unlikely to slip. Therefore, "c" is the best answer.

Page 91: TABE PERT FCAT Reading and Writing Practice    MmAcVcErReIaCKS

===

Mechanical Comprehension Practice Questions

1. Why does a heavy object require a larger amount of force than a light object to get it moving?

a. Because heavy objects are most often very large.b. Because you need a lever to lift it.c. Because heavy objects have a greater amount of inertia.d. Because it has no momentum. 

2. According to Newton's First Law of Motion, objects at rest tend to stay at rest unless

a. they are acted upon by an unbalanced force.b. they are positioned at high altitude.c. a fulcrum is used to lift them.d. None of the above

3. Shifting to a smaller gear on a mountain bike will have an effect on the speed of travel. The smaller sized gear will make pedaling easier but it will also

a. increase the speed of travel.b. decrease the speed of travel.c. have no effect on the speed of travel.d. make the bicyclist work harder.

4. Two trucks are unloading 100 forty pound boxes of nails to a pallet 4 feet below the truck bed. The worker on one of the trucks has the use of a hand truck and a ramp. The worker on the other truck is lifting the 100 forty pound boxes with no ramp and no hand truck. Ignoring the effect of friction, the worker with the ramp and hand truck is

a. doing more work than the worker on the other truck.b. doing less work than the worker on the other truck.c. doing the same amount of work as the worker on the other truck.d. doing 100 joules of work.

5. Which of the following examples does not make use of a mechanical advantage?

a. Using a pry bar to pull a nail from a boardb. Moving a load of wood with a wheelbarrowc. Using a screwdriverd. Sliding a heavy box along a concrete floor.

6. The mass of an object is measured in

a. litersb. kilogramsc. joulesd. none of the above.

7. Which of the following examples does not make use of a wedge?

a. Choosing a sand wedge to hit your golf ballb. Splitting firewood with a chisel and sledge hammer c. Chopping wood with an axed. Using a lever to lift a load

8. A block and tackle refers to a device which is used to

a. put under the wheel of a vehicle to prevent it from rolling backward.b. prevent fish from escaping the hook.c. leverage a stationary object.d. hoist an object into the air by means of rope and pulleys.

9. Downshifting an auto or a truck causes

a. a decrease in speed and an increase in torque.b. an increase in speed and a decrease in torque.c. no change in speed and no change in torque.d. None of the above

10. Shifting to a higher gear in a car or truck causes

Page 92: TABE PERT FCAT Reading and Writing Practice    MmAcVcErReIaCKS

a. a decrease in torque and an increase in speed.b. an increase in torque and a decrease in speed.c. an increase in both speed and torque.d. None of the above

Mechanical Comprehension Practice Question Answers

1. The best answer is "c" because the question is geared toward the concept of inertia. A heavy, dense object possesses more inertia and requires a greater amount of force to accelerate it than an object that is less dense. Think of the difference between a basketball and a bowling ball. The bowling ball has more mass and requires greater force to move it along.

2. The correct answer is "a." Newton is known for his Laws of Motion. His first Law of Motion states that an object at rest tends to stay at rest just as an object in motion tends to stay in motion unless acted upon by an unbalanced force. A resting object, therefore, would have to be pushed or accelerated by an external force. Similarly, a moving object tends to keep moving in the same direction unless an opposing force comes to act upon it. Think of a front-end collision as an "unbalanced force" acting upon a moving object.

3. Shifting to a smaller gear will result in a slower speed of travel. Option "b" is the correct answer. The smaller gear will turn a complete revolution more often than will a larger gear. Therefore, a small gear requires many more revolutions to cover a specific distance than does a large gear. A larger rear controlling gear, though harder to "push," would cause the bicycle to pick up speed.

4. It is true that work is measured in joules but this answer is misleading. Work is defined as force x distance so 100 joules of work would be wrong. The correct answer is "c" because the distance moved and the force applied are the same regardless of whether a ramp and hand truck are used. The vertical distance of 4 feet is the same. The amount of force needed to move the 40 pound boxes is the same whether the force is applied straight up and down or on a ramp. The ramp may make the task easier for the worker to accomplish, but it does not change the amount of work to be accomplished.

5. The best answer is "d." While sliding the box along the floor may be easier and less dangerous than lifting it, no mechanical advantage is used. All the other answers use a mechanical advantage. A pry bar is a type of lever that provides a mechanical advantage. A screwdriver uses a mechanical advantage because the handle has a greater diameter than the point of the screwdriver. That is much like a big wheel (handle) geared to a smaller wheel (tip of screwdriver). A wheelbarrow provides more than one mechanical advantage. It uses leverage with a fulcrum; it also uses a wheel and axle arrangement.

6. The correct answer is "b." Kilograms are a unit measuring weight, akin to "pounds" in the American system of measuring weight. Joules is a measurement of work and kinetic energy. Liters are a measure of liquid volume. Substances such as gasoline or milk are often measured in liters.

7. The correct answer is "d." The other examples use the mechanical advantage of a wedge. A chisel splits wood by exerting a force on the wood in two directions. The "sand wedge" golf club applies a force against the ground and another in the direction of the golf ball. Wood axes are wedge shaped so as to accomplish the same task as the chisel, to apply force in two directions within the wood.

8. The correct answer is "d." A block and tackle can be simply constructed with two pulleys and a lifting line such as a rope, a chain, or a belt of some type.

9. The correct answer is "a." The vehicle transmission can be used to convert engine revolutions into torque increases or decreases. Torque increases result in a slowing of velocity or speed. Increasing torque by downshifting slows the vehicle down. Decreasing torque by shifting to a higher gear increases speed.

10. The correct answer is "a." Up shifting results in torque decreases and speed increases. You can feel the power and speed differences when a vehicle goes into "overdrive." Putting a vehicle in "overdrive" lowers the torque output but increases the speed. Overdrive is useful in driving on flat surfaces, but the vehicle may not produce enough torque to power it up a steep hill.

============

Electronics Information Practice Questions

1. Ohm's Law states that

a. E = I x R.b. R = E x I. c. voltage is equal to the current multiplied by the resistance.d. Both "a" and "c"

2. The electrons revolve around the nucleus in a cumulative series of orbits which are called

a. neutrons.b. subatomic particles.c. shells.d. circulating cores.

3. The part of the atom's shell that determines electrical properties is the __________ shell.

Page 93: TABE PERT FCAT Reading and Writing Practice    MmAcVcErReIaCKS

a. insulatorb. nucleicc. valence d. electronic 

4. A semi-conductor is an element or substance which

a. conducts electricity better than a conductor.b. is useful for certain conductive requirements necessary to some electrical technologies.c. completely inhibits the flow of electrons around the outer shell.d. insulates electrical current from contact with other materials. 

5. When applied to electrical conductivity of household current, 60 hertz means that

a. current flows in only one direction.b. current flows in two directions.c. current flows first in one direction and then another.d. 60 voltage cycles take place in one second.

6. The three necessary components of an electrical circuit are

a. an electrical load, conductors, and a circuit for the electricity flow to follow.b. a switch, a resistor, and a path to follow.c. a 60 hertz receptacle, a switch, and a power source.d. a closed circuit, a battery, and radio waves.

7. Doping is a term used in the semiconductor process when

a. impurities are added into the crystal structure of silicon.b. hydrogen atoms are added to the crystal structure of silicon.c. impurities are removed from the crystal structure of silicon.d. semiconductors are used for medical purposes.

8. The property of electricity that pushes and moves it along a circuit is called

a. alternating current.b. amperage.c. resistance.d. voltage. 

Electronics Information Practice Question Answers

1. Choice "d" is the correct choice. Ohm's law is shown in mathematical form in choice "a." Choice "c" says the same thing but expresses it differently. Choice "b" is incorrect.

2. The correct answer is "c." The outer shell gives a substance its electrical properties. The outer shell is called the "valence shell."

3. The correct answer is "c." It is the valence shell that determines a substance's electric properties. Depending on the total number of electrons, the first and second shells are filled first. If a metal or other substance has sufficient electrons to populate the third or valence shell, the resulting configuration will determine whether the element is a conductor, a semi-conductor, or an insulator.

4. The correct answer is "b." Some materials conduct electrical current very easily. Sometimes this is desirable and sometimes it is not. A semi-conductor will allow electrical current to pass through it but will also subject it to some resistance. This restricted flow capacity of the semi-conductor is beneficial when put to appropriate technological uses.

5. The correct answer is "d." Household current is alternating current and flows in two directions at 60 hertz. Each time the current cycles, it goes from zero to 60 hertz in both directions. Within the space of one second, the cycle of current occurs 60 times in household alternating current.

6. The components of a basic circuit are a source of the voltage, the electrical load, a conductor (such as copper wire), and a completed circuit. A switch is unnecessary to current flow, though it certainly helps to be able to open and close the circuit. Current will pass through air but it usually prefers more stable circuitry such as a copper wire; it does not, however, flow upon radio waves. Choice "a" is the correct answer here.

7. The correct answer is "a." Pure silicon is a very stable substance with no free electrons that would allow current flow. "Doping" the silicon by adding impurities like phosphorous, arsenic, or antimony to it causes a free electron to move throughout the crystal. This allows for conductivity.

8. Voltage, in choice "d," is the correct answer. A force is always needed to make electric current flow through a circuit. The force that drives the current through a wire or other material is called voltage. Choice "b" is wrong because amperage is a measure of the electricity flow rate. Just as the name implies, "Resistance" is the process of impeding electrical flow, so option "c" is also wrong. Choice "d" is the correct answer.

Page 94: TABE PERT FCAT Reading and Writing Practice    MmAcVcErReIaCKS

=========

TABE Test Study Guide 1 © . Table of ContentsTABE TEST RESOURCES.................................................................................................................... 4ENGLISH.................................................................................................................................................... 5ENGLISH.................................................................................................................................................... 5STRATEGY 1: FLYING OVER THE PASSAGE ............................................................................................ 5STRATEGY 2: CREATING A TENTATIVE SUMMARY ................................................................................. 6STRATEGY 3: OPENINGS AND ENDINGS ................................................................................................ 7STRATEGY 4: USING KITCHEN LOGIC.................................................................................................... 7STRATEGY 5: GETTING INTO THE AUTHOR’S MIND.............................................................................. 8STRATEGY 6: EMOTIONAL WORDS......................................................................................................... 8STRATEGY 7: FINDING THE KEY WORDS............................................................................................... 9STRATEGY 8: MAKING PROPER INFERENCES....................................................................................... 11STRATEGY 9: APPLYING IDEAS FOR GENERALIZATIONS..................................................................... 12STRATEGY 10: USING CONTEXT CLUES ............................................................................................. 12STRATEGY 11: BREAKING DOWN PASSAGE ORGANIZATION............................................................. 13STRATEGY 12: FIRST WORD ANALYSIS ............................................................................................. 15STRATEGY 13: UNDERSTANDING THE INTIMIDATION......................................................................... 15STRATEGY 14: FINDING YOUR OPTIMAL PACE................................................................................... 16STRATEGY 15: DON’T BE A PERFECTIONIST........................................................................................ 18STRATEGY 16: FACTUALLY CORRECT, BUT ACTUALLY WRONG ......................................................... 19STRATEGY 17: DIFFERENT VIEWPOINTS ............................................................................................. 19STRATEGY 18: EXTRANEOUS INFORMATION ...................................................................................... 20STRATEGY 19: APOSTROPHES .............................................................................................................. 20Possessive Nouns ......................................................................................................................... 20

Page 95: TABE PERT FCAT Reading and Writing Practice    MmAcVcErReIaCKS

Possessive Personal Pronouns vs. Contractions ............................................................... 21STRATEGY 20: COMMA ERRORS.......................................................................................................... 22STRATEGY 21: PROBLEMS WITH REFERENCES ................................................................................... 24STRATEGY 22: PROBLEMS WITH AGREEMENT .................................................................................... 26STRATEGY 23: LACK OF PARALLELISM................................................................................................. 28STRATEGY 24: MISCELLANEOUS PROBLEMS ....................................................................................... 30MATHEMATICS..................................................................................................................................... 34NUMBER TYPES ....................................................................................................................................... 342 © . Integers, Odd and Even Numbers, Prime Numbers, Digits.......................................... 34ADDITION AND MULTIPLICATION OF ODD AND EVEN NUMBERS........................................................ 34PERCENT.................................................................................................................................................. 35Percent less than 100................................................................................................................. 35Percent Greater than 100 ......................................................................................................... 35Percent less than 1...................................................................................................................... 35Percent Increase/Decrease....................................................................................................... 36AVERAGE ................................................................................................................................................. 36WEIGHTED AVERAGE.............................................................................................................................. 37Average Speed .............................................................................................................................. 38PROPERTIES OF SIGNED NUMBERS....................................................................................................... 38FACTORING.............................................................................................................................................. 39PROBABILITY ........................................................................................................................................... 39GEOMETRIC FIGURES ............................................................................................................................. 40GEOMETRIC SKILLS AND CONCEPTS..................................................................................................... 42Properties of Parallel Lines ....................................................................................................... 42Angle Relationships ..................................................................................................................... 43Side Relationships........................................................................................................................ 44

Page 96: TABE PERT FCAT Reading and Writing Practice    MmAcVcErReIaCKS

AREA AND PERIMETER............................................................................................................................ 46Rectangles....................................................................................................................................... 46Circles ............................................................................................................................................... 46Triangles .......................................................................................................................................... 47VOLUME ................................................................................................................................................... 47COORDINATE GEOMETRY ....................................................................................................................... 483 © . TABE Test Resources Free TABE Practice Testshttp://www.testprepreview.com/tabe_practice.htmFinancial Aid Factshttp://www.finaidfacts.orgScholarship Helphttp://www.scholarshiphelp.orgStudy Tips and Informationhttp://www./resource_tips.htm4 © . English The English test measures a test taker’s ability to understand, analyze and evaluate written passages. The passages will contain material that will be from a variety of sources and on a number of different topics. Each of the passages and statements in the English test will be followed by a series of questions covering the content of the passage or statement, in which you will have to answer questions, which will demonstrate how well you understand the passages and are able to draw conclusions about the material. Strategy 1: Flying Over the Passage A topic that is hotly debated among test taking circles is whether or not you should read the reading passages before you read the question. One theory is that you can save time if you read the questions first and then go back and read over the passage. Another theory is that you should read the passage first and then go into the questions. Both theories have their own individual merit and due to the differences in ability and preferences among test takers, one method may work better than another for you. Our recommended theory is the flyover. You want to spend some time on the passage, at a bare minimum so that you have a general idea about what the questions are going to ask and get your mind into the proper mindset for the series of questions. However, you don’t want to waste too much time on reading the passage, because much of the 5 © . detail will be forgotten by the time you get to the questions anyway. Therefore, you should fly over the passage. You should read it very quickly for a high-level overview (hence the flyover) understanding of

Page 97: TABE PERT FCAT Reading and Writing Practice    MmAcVcErReIaCKS

what is contained in the passage. In part, this is a compromise between the theories that gains most of the benefits of each. You won’t waste time on the details and yet will have a general idea of what the passage is about and what to expect. Strategy 2: Creating a Tentative Summary After you’ve finished your flyover of the passage, take a few seconds and compose a tentative mental summary of what you’ve just read. Try to sort out the details you picked up on and arrange them into a loose organizational pattern that describes the passage. Remember that your goal in the flyover is not to check it off of a test-taking list of things to do. You want there to be some purpose behind the flyover and having the definite goal of being able to put together a brief mental summary will allow you to maintain some focus and gain benefit from the flyover – as opposed to just skimming it for the sake of skimming it without actually picking up on anything. As you begin going through the questions and answer choices, if you get good enough at putting together your mental summaries from practice, you should be able to eliminate a number of answer choices that are immediately contrary to your summary. Note, however that if you find yourself without any good answer choices remaining (because you’ve eliminated them all) you obviously had to have eliminated the right answer choice. Don’t hesitate to reopen an answer choice that 6 © . you’ve already “eliminated” from consideration and reconsider it as a possibility. If you think an answer choice contradicts your initial summary, you’re probably right, but are not infallible. Strategy 3: Openings and Endings A main focus of this flyover will be the opening and ending sentences in each paragraph. These are likely to contain the main ideas of the paragraphs and should be mentally tagged for future reference. Try to remember a vague idea of what the different paragraphs are about, because this will save you time when answering questions later. For the most part, make sure you never try to just answer the questions from this first flyover. Always try to go back and confirm the answer, as your memory will play tricks on you and the writers of the test questions may deliberately have planted a trap for you – remember that they don’t exactly have your best interests at heart. Strategy 4: Using Kitchen Logic When a question asks the test taker to identify a main idea, you should first focus on the opening and ending sentences of the passage and each individual paragraph. If you can’t find the main idea from these key sentences, then ask yourself how you would describe the passage to someone who had never read it. Which words and phrases would you use to explain the principle ideas of the passage? This is called “Kitchen Logic” - when you explain something the way you would if you were talking to your friends and family, while sitting 7 © . at your kitchen table. So, when faced with identifying the main idea of a difficult passage, make it easier on yourself by backing away from the passage and thinking about it in terms of using easy “kitchen logic”. Strategy 5: Getting into the Author’s Mind

Page 98: TABE PERT FCAT Reading and Writing Practice    MmAcVcErReIaCKS

A number of questions become much easier when you place yourself into the mind of the author of the passage. Ask yourself a few different questions: “Why did the author write this passage?” “What was the author trying to say?” What angle is the author taking?” “What is the single most important point the author is trying to make?” Put yourself in the shoes of the author and imagine that you wrote the passage and try to identify what you were trying to describe and how you were trying to describe it. If you take on the opinions and ideas expressed by the author as your own, then it becomes easier to answer questions that would be easy for the author to answer. Strategy 6: Emotional Words Each question will be about a different angle of the passage. For questions asking about the author’s emotions, find words in the passage that are adjectives describing emotions. 8 © . So, if a question asks what sort of attitude an author had towards the passage or subject, then look throughout the passage for attitude words that might convey a positive or negative attitude. Are words such as brilliant, excited, delightful used, or are words such as depressive, gloomy, disappointing used? A lot of questions could be answered correctly simply by going through and circling all the adjectives in a passage. Without looking at anything else except for the adjectives in a passage, most questions about attitude or emotion could be answered correctly. Another way of handling these situations is to arrange all of the answer choices in a list going from most negative to most positive. Example: Question: The author’s attitude on this topic is best described as: A. indignation B. eagerness C. impartiality D. fear Now arrange these in order from negative to positive: ( - ) indignation, fear, impartiality, eagerness (+) This will help sort out the different choices and keep you from overlooking an answer choice and making an easy mistake. Strategy 7: Finding the Key Words 9 © . The strategy of finding certain “give-away” words does not only apply to adjectives in questions about emotions or attitude. Many questions about specific details will have key words that hold the “key” to finding the right part of the passage to look in for the answer. Rather than answering based on your memory of the passage, you always want to have support for your answer choice rooted in a specific part of the passage. To gain that support, it follows that you have to identify which part of the passage to look in. While reading back over the entire passage may be the most foolproof method of finding that important part of the passage, it definitely is not the most time economical method of finding that part of the passage.

Page 99: TABE PERT FCAT Reading and Writing Practice    MmAcVcErReIaCKS

A better route is to find key words in the question or answer choices that are likely to stand out in the passage and will enable you to quickly narrow your search down. These key words will be nouns or verbs in the question or answer choices. Once you’ve identified possible key words, then you should scan through the passage quickly looking for either those key words to be repeated in the passage, or their synonyms to appear in the passage. Once you find a particular part of the passage that either has the exact key word repeated or a synonym of the key word, you have probably identified the particular part of the passage that will contain the support or justification that you need to correctly answer the question and will allow you to be confident in your answer choice selection. One warning that should be made here is that often question writers may use the exact same word or wording in their answer choices that are used in the passage, but have done so in such a way as to mislead 10 © . you. So, simply because a particular word or phrase appears in an answer choice and also appears exactly the same in a passage does not make that answer choice correct. Be sure that you reread the answer choice and consider the context that it is in, to ensure that you are not misled by a cheap trick. In conclusion, always try to connect the question to the right words in the passage that will allow you to save time in finding the right part of the passage to look in for the answer and will give you the key to the correct answer choice. Strategy 8: Making Proper Inferences Questions that ask you to make an inference from the passage will require you to use your own personal judgment. Anything directly stated by the author is not an inference. You will need to understand the main idea of the passage in order to make a proper inference about the author’s intent and mindset. The obvious will not be enough to answer an inference question. You must logically deduce what follows from what the author has stated in the passage. You are looking for what can be inferred by the passage, not what is directly stated in the passage. 11 © . Strategy 9: Applying Ideas for Generalizations Generalization questions are similar to inference questions in that you have to go beyond what is directly stated in the passage by the author. It helps to put yourself again in the author’s shoes. If you were the author and believed in what you had just written, how would you feel about another similar situation? What would either strengthen or weaken your argument. How would you apply the information you have just expressed to a completely different situation? Strategy 10: Using Context Clues Context clues are a valuable aide in helping you understand difficult phrases or words in the passage. A number of questions will ask you about the meaning of words as they are used in a given passage. If you already know the definition of the word, or have some familiarity with it, a common mistake is to go with your first impulse and choose the answer that you immediately recognize. However, the

Page 100: TABE PERT FCAT Reading and Writing Practice    MmAcVcErReIaCKS

reason the test writers may have chosen that particular vocabulary word is because it is used in an unusual context. Therefore, return to the passage and find where the word is used and make sure that you understand how it is being used in the passage. Once you’ve made your choice of a good definition go back again to the passage and reread that particular section, but mentally replace the answer choice you’ve chosen for the word being asked about. 12 © . Example: A passage states: “He was notorious for making decisions on the spur of the moment…” Question: Which of the following words, if substituted for the word “notorious” would introduce the LEAST change in the meaning of the sentence? A. evil B. disturbed C. famous D. despised If you knew that the most common definition for “notorious” meant being known in an unfavorable sense, then you might be tempted to choose choice A, “evil.” But once you review back over the passage, choice C, “famous” fits in better into the context of the sentence of passage. Read the sentence again and substitute your chosen answer choice for the word it replaces. This gives you: ““He was famous for making decisions on the spur of the moment…,” which makes sense and is correct. Strategy 11: Breaking Down Passage Organization In trying to understand the author’s perspective, you will sometimes be asked about how the passage is organized. Many times, the 13 © . simplest way to find the answer is to note how the opening sentence in a passage or paragraph relates to the rest of the passage. How does the author’s main idea get developed and broken down into supporting ideas and statements? As you go through the answer choices for these organization problems, quiz yourself on each answer choice. Example: Question: Which of the following best describes the organization of the author’s discussion of this topic? A. He provides an example – Ask yourself, is there an example in the question? Don’t work exclusively from your memory. Make sure you can go back and actually find the example in the passage. B. He makes a comparison – Ask yourself, is there a comparison in the question? Again, go back to the passage and actually find the comparison being made and verify that it exists. C. He makes an acknowledgement – Ask yourself, where is the acknowledgement made and to whom? D. He discusses a theory – Ask yourself, which theory is being discussed? After each of these initial questions, remember that it is not enough

Page 101: TABE PERT FCAT Reading and Writing Practice    MmAcVcErReIaCKS

for them simply to be true, they have to answer the question. Simply because the author provided an example, doesn’t make choice A correct. The example provided may have been to support a comparison that he was making and the comparison may be the main method of organization, which in this case would make answer choice 14 © . B correct. So always read all the answer choices and only choose the one that is the best, not just the first one you read that is factually correct. Strategy 12: First Word Analysis When asked for main ideas that best summarize the passage, an easy strategy is to look at the first words in each answer choice and without looking at the rest of the answer choice, see if you could make a decision based on those first words alone. Example: Question: Which of the following best explains the author’s primary purpose? A. dispute… B. describe… C. condemn… D. convince… If you know that the passage is fairly neutral about the subject, then even if you know nothing else, you can probably eliminate the stronger verbs used in answer choices A, C, and D, leaving you with “describe” or answer choice B as being correct. Strategy 13: Understanding the Intimidation The test writers will generally choose passages that will be completely foreign to most test takers. You can’t expect the passages to be on a topic with which you have any familiarity. If you do happen to come 15 © . across a passage that you are familiar with, consider yourself lucky, but don’t plan on that happening. The passages will also frequently be drawn from longer passages in books, articles, journals, etc. Therefore, the passage that you will face on the test may almost seem out of context and as though it begins in the middle of a thought process. You won’t have a nice title overhead explaining the general topic being covered but will immediately be thrown into the middle of a strange format that you don’t recognize. Also, while the topics chosen may have originally been interesting reading in their original state, after a particular section is pulled and used for the test passage, it will likely be dry and boring. Getting hit by strange reading topics that you don’t recognize, of which you may only have a small part of the original selection, and that are dry and boring can be a bit intimidating if you’re not adequately prepared. Just remember that the passages themselves will contain all the information necessary to answer the questions and you don’t need any prior knowledge of the topic in order to succeed and do well on the test. Strategy 14: Finding your Optimal Pace Everyone reads at a different rate. It will take practice to determine what is the optimal rate at which you can read fast and yet absorb and comprehend the information. This is true for both the flyover that you

Page 102: TABE PERT FCAT Reading and Writing Practice    MmAcVcErReIaCKS

should initially conduct and then the subsequent reading you will have to do as you go through and begin answering the questions. However, 16 © . on the flyover, you are looking for only a surface level knowledge and are not trying to comprehend the minutia of details that will be contained in the passages. You can practice with any form of reading material. Read an article at your normal pace and then after you’re finished, ask yourself some questions about what you just read and see how well you can comprehend. Experiment with reading articles faster and slower and always gauge how well you comprehended what you read at the end. Train your brain to remember the details and absorb the facts. With practice, you will find the pace that you should maintain on the test while going back through passages. It should be a comfortable rate. This is not a speed reading exercise. If you have a good pace, and don’t spend too much time on any question, you should have a sufficient amount of time to read the different sections of the passages at a comfortable rate. The two extremes you want to avoid are the dumbfounded mode, in which you are lip reading every word individually and mouthing each word as though in a stupor, and the overwhelmed mode, where you are panicked and are buzzing back and forth through the passage in a frenzy and not comprehending anything. You must find your own pace that is relaxed and focused, allowing you to have time for every question and give you optimal comprehension. Note that you are looking for optimal comprehension, not maximum comprehension. If you spent hours on each word and memorized the passage, you would have maximum comprehension. That isn’t the goal though, you want to optimize how much you comprehend with 17 © . how much time you spend reading. Practice will allow you to determine that optimal rate. Strategy 15: Don’t be a Perfectionist If you’re a perfectionist, this may be one of the hardest strategies, and yet one of the most important. The test you are taking is timed, and you cannot afford to spend too much time on any one question. If you are working on a problem and you’ve got your answer split between two possible answer choices, and you’re going back through the passage and reading it over and over again in order to decide between the two, you can be in one of the most frustrating situations possible. You feel that if you just spent one more minute on the problem, that you would be able to figure the right answer out and decide between the two. Watch out! You can easily get so absorbed in that problem that you loose track of time, get off track and end up spending the rest of the test playing catch up because of all the wasted time, which may leave you rattled and cause you to miss even more questions that you would have otherwise. Therefore, unless you will only be satisfied with a perfect score and your abilities are in the top .1% strata of test takers, you should not go into the test with the mindset that you’ve got to get every question right. It is far better to accept that you will have to guess on some questions and possibly get them wrong and still have time for every

Page 103: TABE PERT FCAT Reading and Writing Practice    MmAcVcErReIaCKS

question, than to work on every problem until you’re absolutely confident in your answer and then run out of time on the last few problems. 18 © . Strategy 16: Factually Correct, but Actually Wrong A favorite ploy of question writers is to write answer choices that are factually correct on their own, but fail to answer the question, and so are actually wrong. When you are going through the answer choices and one jumps out for being factually correct, watch out. Before you mark it as your answer choice, first make sure that you go back to the question and confirm that the answer choice answers the question being asked. Strategy 17: Different Viewpoints Some passages will express the author’s viewpoint on a topic, along with the viewpoint of other experts or other individuals. This can lead to trouble in answering questions though. If asked for the viewpoint of the author, you might go back to the passage, find where a certain viewpoint is expressed, answer the question based on what you read and move on. For most passages, that would be fine, but when other viewpoints besides the author’s are expressed, you have to discern who is expressing their opinion in the passage. Make sure that if multiple individuals are giving their viewpoint on a topic, that you sort them out for any questions and associate the right viewpoint with the right individual. 19 © . Strategy 18: Extraneous Information Some answer choices will seem to fit in and answer the question being asked. They might even be factually correct. Everything seems to check out, so what could possibly be wrong? Does the answer choice actually match the passage, or is it based on extraneous information not even contained in the passage. Just because an answer choice seems right, don’t assume that you overlooked information while reading the passage. Always try to go back and find the support for the answer choice in the passage. Your mind can easily play tricks on you and make you think that you read something or that you overlooked a phrase. Unless you are behind on time, always go back to the passage and make sure that the answer choice “checks out.” Strategy 19: Apostrophes An apostrophe is used to form a possessive or a contraction. Check for the following common apostrophe errors. The bracketed parenthetical demonstrates the correct use. Possessive Nouns 1. Singular possessive nouns. Use 's to show that a singular noun is possessive [the defendant's motion]. You can apply this rule even when the singular noun already ends in "s" (Charles's 20 © . costume) though many writers add only the apostrophe [Charles' shoes]. 2. Plural possessive nouns not ending in "s." Use 's to show that a plural noun not ending in "s" is possessive [the children's toy].

Page 104: TABE PERT FCAT Reading and Writing Practice    MmAcVcErReIaCKS

3. Plural possessive nouns ending in "s." Add only an apostrophe to make plural nouns possessive [the boys' game]. 4. Nouns that are not possessive. Do not add an apostrophe to a noun that is not possessive [the teachers (not teacher's or teachers') have claimed; the Joneses (not Jones' or Jones's) did not attend]. Possessive Personal Pronouns vs. Contractions Apostrophe errors with possessive personal pronouns are common because possessive personal pronouns indicate possession and we are used to using apostrophes to indicate possession. Also possessive personal pronouns are easily confused with contractions. Here are the basic rules: 5. Possessive personal pronouns. Do not add an apostrophe to a possessive pronoun [the problem is hers (not her's); the corporation must disclose its (not it's) assets.] 6. Contractions. Do use an apostrophe in a contraction (it's time to go; you're the one). 21 © . * Watch especially for errors in using "it's" or "its." Remember that "it's" means "it is" and "its" indicates ownership. Confusing these two words is the most common apostrophe error. Strategy 20: Comma Errors Commas are also major troublemakers. Watch for these situations: 1. Use a comma to separate two independent clauses joined with a coordinating conjunction (and, or, but, nor, so, for). [The child agreed, but the parent objected]. 2. Use a semicolon or a period, not a comma, to separate two independent clauses not joined by a coordinating conjunction [The child agreed; however, the parent objected.]. The following transitional words and phrases are conjunctive adverbs, not conjunctions: accordingly however also moreover consequently on the other hand for example otherwise for instance similarly furthermore therefore hence thus Therefore, do not use a comma to separate two independent clauses on either side of one of these words. 22 © . Incorrect: The city must increase its tax base, however, the citizens must be able to accept the additional tax burden. Correct: The city must increase its tax base; however, the citizens must be able to accept the additional tax burden. 3. Conjunctive adverbs signal the relationship between the point(s) made in the material before their sentence and the material of that sentence. The words and phrases in the list above are examples. When you begin an independent clause with a conjunctive adverb or when you use it in the middle of a sentence, set it off with a comma. Moreover, the defendant has not yet established a proper foundation for this testimony.

Page 105: TABE PERT FCAT Reading and Writing Practice    MmAcVcErReIaCKS

The student, moreover, has not yet turned in an acceptable project to meet his assignment’s requirements. 4. Use commas to set off the year if you also identify the day [The birth of Norma Kelly on June 2, 1974, brought the . . . .], but omit the commas otherwise [The birth of Norma Kelly in June 1974 brought . . . .]. 5. Use commas to separate three or more simple items in a list. If the descriptions of the items are long or complex, use semicolons to separate them. 23 © . 6. Use a comma to separate two independent clauses connected by a coordinating conjunction unless the two independent clauses are short and simple: Correct: The sunlight helped the flowers to grow, but they require frequent watering in order to stay alive. Correct: Yours is timely and mine is late. Strategy 21: Problems With References A referent is a word or phrase that refers to something else (an antecedent). Problems with referents can cause confusion and, sometimes, unintended humor. Problems with references occur primarily (1) when sentences have more than one possible antecedent (often caused by placing the referent too far from the intended antecedent); or (2) when the antecedent is only implied. Here are examples problems with references. More than one possible antecedent The doctors told their patients that they had serious problems. [Who had problems?] To prevent children from sucking their thumbs, some parents soak them in tabasco sauce. [Do the parents soak the children or the thumbs?] The corporate officers had failed to disclose the serious conflicts of interest caused by their ownership of several of T&J's suppliers. The possibility of a bankruptcy was a disaster for them. 24 © . [Was the possibility a disaster for the officers, the conflicts, or the suppliers?] Referring to an antecedent that is only implied: The corporate officers had failed to disclose the serious conflict of interest raised by the possible bankruptcy of T&J's primary supplier. This was a disaster for the officers. [What was a disaster -- the failure to disclose, the conflict, or the possible bankruptcy?] Notice that the confusing reference in this last example is caused by using "this" alone. Using the pronoun "this" or "that" without a noun following immediately is usually inadvisable. Here, for instance, the confusion could be resolved easily by adding the clarifying noun after "this": The corporate officers had failed to disclose the serious conflict of interest raised by the possible bankruptcy of T&J's primary supplier. This failure was a disaster for the officers. The three primary strategies for solving reference problems are: (1) repeating the antecedent (as in the prior example); (2) re-arranging

Page 106: TABE PERT FCAT Reading and Writing Practice    MmAcVcErReIaCKS

the material to place the referent close to the antecedent; or (3) rearranging the material to eliminate the need for the referent. For instance, here is another possible solution to the reference problem above: 25 © . The possibility of a bankruptcy was a disaster for the corporate officers. They had failed to disclose the serious conflicts of interest caused by their ownership of several of T&J's suppliers. Strategy 22: Problems With Agreement Here are the most common errors in agreement: 1. The following indefinite pronouns are singular and take a singular verb: anyone Anyone is welcome. each Each is an expert. either Either supports the argument. everyone Everyone has problems. neither Neither sings in tune. The singular verb is correct even when the indefinite pronoun is followed by a prepositional phrase with a plural noun: Each of these peaceful alternatives was [not "were"] ignored. Either of the twins is [not "are"] available. However, the following indefinite pronouns are either singular or plural, depending on the nouns or pronouns they refer to: all any none 26 © . some For example: All [singers] are permitted . . . . All of the money is counted . . . . None of them are satisfied . . . . None of the royalty was present . . . . 2. "The court" is a singular term, taking a singular pronoun: The court overruled the traditional rule originally announced in December. It [not "They"] held that . . . . 3. This same kind of error can slip in when referring to any institution or business: He said that he is very grateful to St. Catherine's Hospital. They treated him with respect. The problem sometimes occurs because the writer is avoiding the awkwardness of attributing a human action to an "it." For instance, in the St. Catherine's example, the writer is probably avoiding "It treated him with respect." That problem can usually be solved by changing the antecedent to the humans who actually performed the action: He said that he is very grateful to the staff at St. Catherine's Hospital. They treated him with respect. 27 © . 4. Watch for both verb agreement and pronoun agreement when a singular subject is modified by a phrase or clause containing a plural noun: The confidence of several families was [not were] attained.

Page 107: TABE PERT FCAT Reading and Writing Practice    MmAcVcErReIaCKS

Each of the groups agrees [not agree] to resolve the problem peacefully. Every student who had already taken both courses is [not are] excused from this requirement. 5. Use a possessive pronoun before a gerund phrase. A gerund is an "ing" verb that serves as a noun. The gerund can stand alone or can begin a gerund phrase, but either way, the word or phrase functions as a noun: Running is good for you. Coming to work late can result in disciplinary action. Since a gerund or gerund phrase functions as a noun, it takes a possessive pronoun: We'll go to their house for the party instead of their [not them] coming to ours. Strategy 23: Lack of Parallelism 28 © . Where possible, similar ideas should be expressed in a similar (parallel) structure and grammatical form. Parallelism makes for easier reading and clearer meanings. It also improves sentence rhythm and cuts down on verbiage. Parallel structure: Parallelism makes for easier reading and clearer meanings. "easier reading" and "clearer meanings" are parallel. Non-parallel structure: Problems occur when the business conceals relevant documents or by deluging the auditors with irrelevant documents. In the non-parallel example, the writer identifies two situations in which problems occur; however, the two situations are phrased in non-parallel structure. A parallel structure would be: Problems occur when the business conceals relevant documents or when they deluge the auditors with irrelevant documents. Now that the structure is parallel, extra words can go: Problems occur when the business conceals relevant documents or deluge the auditors with irrelevant documents. Parallel structure is especially important in a list. 29 © . When the new commander arrived at the post, he immediately posted a new list of orders: no leaves were to be granted, and no leniency was to be given. Strategy 24: Miscellaneous Problems 1. Use the subjective case for a pronoun that functions as the subject of an understood verb. The subjective case is the form the pronoun takes when used as the subject of the sentence ("I," "we," "they"). For example, consider these two sentences: The corporation's president worked harder than me. The corporation's president worked harder than I. Which is correct? The second version is correct, because "I" is the subject of an understood verb "worked." In other words, the sentence is actually a shortened version of "The corporation's president worked harder than I worked." This is an easy mistake to make because the correct case may sound

Page 108: TABE PERT FCAT Reading and Writing Practice    MmAcVcErReIaCKS

wrong. If so, the best solution is to add the understood verb or to rephrase the sentence completely to avoid the awkwardness. 2. Use the subjective case for a pronoun that functions as the complement of a subject. A pronoun is a subjective complement when it actually equals the subject of the sentence. For example, consider these two sentences: 30 © . The person least anxious after the test was her. The person least anxious after the test was she. Which is correct? The second version is correct, because the pronoun in the predicate, "she," actually equals the subject, "the person least anxious after the test." In other words, the sentence is like an algebra equation: The person least anxious after the test = she. In a sentence like this one, the noun or pronoun in the predicate functions like the subject of the sentence. The sentence should read the same as if you turned it around: She was the person least anxious after the test. Once again, this is an easy mistake to make because the correct case may sound wrong. In spoken English we often hear "It's him," or "It's me." Again, the best solution may be to reverse the sentence or to re-phrase the sentence completely to avoid the awkwardness. 3. Use "try to" and "sure to" rather than "try and" or "sure and." Incorrect: Ms. Thompson wanted to try and finish the project by Friday. Correct: Ms. Thompson wanted to try to finish the project by Friday. Incorrect: Be sure and pick up the baby from daycare. Correct: Be sure to pick up the baby from daycare. 31 © . This rule makes sense if you think about it. The proper function of the word "and" is to connect two different things. Thus, the "and" in the first sentence should mean that Ms. Thompson wanted to do two different things, but she didn't; she only wanted to do one thing – finish the project. The same is true for the third sentence. The "and" seems to tell the reader that the sentence is an instruction to do two different things, but it isn't. The reader is only to do one thing – pick up the baby. 4. "Hopefully" is an adverb that explains how someone does something, as in "She asked hopefully." Often "hopefully" is misused in this or a similar fashion: "Hopefully the storm will pass." This use actually asserts that the storm’s passing will be full of hope. The writer actually means "I hope that the storm will pass." 5. Watch out for one more common error with adverbs: the difference between "I feel bad" and "I feel badly." In the first sentence, the writer is commenting on how she feels, either physically (perhaps she has the flu) or emotionally (perhaps she is sad). The first sentence is the proper use of "bad" as an adjective. However, the second sentence uses "bad" in its form as an adverb. The writer is commenting on her ability to feel; perhaps her fingers are numb. Sometimes writers use the adverbial form "badly" when they

Page 109: TABE PERT FCAT Reading and Writing Practice    MmAcVcErReIaCKS

mean "bad," perhaps believing that "badly" sounds more elevated. 32 © . 6. Watch for errors in using "myself" instead of using "I" or "me." "Myself" is used as a reflective pronoun or as a device for emphasis. Correct: I injured myself. used as a reflective pronoun Correct: I will draft the interrogatory answers myself. used for emphasis Each time "myself" is used, test the use by asking whether "I" or "me" could substitute for "myself." If so, using "myself" is incorrect. Incorrect: Ms. Alpha and myself will meet you for lunch. Incorrect: If you have any questions about this demonstration, contact Mr. Jones or myself. 33 © . Mathematics The Mathematics test measures a test taker’s ability to solve problems representing some of the key concepts in mathematics. Some problems will only test one concept, while others will involve multiple concepts integrated together in a single problem. The problems will have few technical terms, aside from basics, such as area, perimeter, integer, and ratio, which are expected to be common mathematical knowledge. All figures shown will be drawn accurately and lie in a single plane, unless noted otherwise. Number Types Integers, Odd and Even Numbers, Prime Numbers, Digits ƒ Integers…, -4, -3, -2, -1, 0, 1, 2, 3, 4, … ƒ Consecutive Integers: Integers that follow in sequence; for example, 22, 23, 24, 25. Consecutive Integers can be more generally represented by n, n + 1, n + 2, n + 3, … ƒ Odd Numbers…, -9, -7, -5, -3, -1, 1, 3, 5, 7, 9, …ƒ Even Numbers…, -8, -6, -4, -2, 0, 2, 4, 6, 8, … (Note: zero is an even number)ƒ Prime Numbers…, 2, 3, 4, 7, 11, 13, 17, 19, … (Note 1 is not a prime and 2 is the only even prime)ƒ Digits: 0, 1, 2, 3, 4, 5, 6, 7, 8, 9Addition and Multiplication of Odd and Even Numbers 34 © . Addition Multiplication even + even = even even x even = even odd + odd = even even x odd = even even + odd = odd odd x odd = odd Percent Percent means hundredths or number out of 100. For example, 40 percent means 40/100 or .40 or 2/5. Percent less than 100 Problem 1: If the sales tax on a $30 item is $1.80, what is the sales tax rate? Solution: $1.80 = n/100 x $30 n = 6, so 6% is the sale tax rate Percent Greater than 100 Problem 2: What number is 250% of 2?

Page 110: TABE PERT FCAT Reading and Writing Practice    MmAcVcErReIaCKS

Solution: n = 250/100 x 2 n = 5, so 5 is the number Percent less than 1 Problem 3: 3 is 0.2 percent of what number? Solution: 3 = 0.2/100 x n 35 © . n = 1,500, so 1,500 is the number Percent Increase/Decrease Problem 4: If the price of a computer was decreased from $1,000 to $750, by what percent was the price decreased? Solution: The price decrease is $250. The percent decrease is the value of n in the equation 250/1000 = n/100. The value of n is 25, so the price was decreased by 25%. Notes: n% increase means increase/original = n/100; n% decrease means decrease/original = n/100. Average An average is a statistic that is used to summarize data. The most common type of average is the arithmetic mean. The average (arithmetic mean) of a list of n numbers is equal to the sum of the numbers divided by n. For example, the mean of 2, 3, 5, 7, and 13 is equal to 2 + 3 + 5 + 7 + 13 / 5 = 6 When the average of a list of n numbers is given, the sum of the numbers can be found. For example if the average of six numbers is 12, the sum of these six numbers is 12 x 6, or 72. 36 © . The median of a list of numbers is the number in the middle when the numbers are ordered from greatest to least or from least to greatest. For example, the median of 3, 8, 2, 6, and 9 is 6 because when the numbers are ordered, 2, 3, 6, 8, 9, the number in the middle is 6. When there is an even number of values, the median is the same as the mean of the two middle numbers. For example, the median of 6, 8, 9, 13, 14, and 16 is 9 + 13 / 2 = 11 The mode of a list of numbers is the number that occurs most often in the list. For example, 7 is the mode of 2, 7, 5, 8, 7, and 12. The numbers 10, 12, 14, 16, and 18 have no mode and the numbers 2, 4, 2, 8, 2, 4, 7, 4, 9, and 11 have two modes, 2 and 4. Note: The mean, median, and mode can each be considered an average. On the test, the use of the word average refers the arithmetic mean and is indicated by “average (arithmetic mean).” The exception is when a question involves average speed (see problem 2 below). Questions involving the median and mode will have those terms stated as part of the question’s text. Weighted Average Problem 1: In a group of 10 students, 7 are 13 years old and 3 are 17 years old. What is the average (arithmetic mean) age of these 10 students? 37 © . Solution: The solution is not the average of 13 and 17, which is 15. In this case the average is 7(13) + 3(17) / 10 = 91 + 51 / 10 = 14.2 years

Page 111: TABE PERT FCAT Reading and Writing Practice    MmAcVcErReIaCKS

The expression “weighted average” comes from the fact that 13 gets a weight factor of 7, whereas 17 gets a weight factor of 3. Average Speed Problem 2: Jane traveled for 2 hours at a rate of 70 kilometers per hour and for 5 hours at a rate of 60 kilometers per hour. What was her average speed for the 7-hour time period? Solution: In this situation, the average speed is: Total Distance/Total Time The total distance is 2(70) + 5(60) = 440 km. The total time is 7 hours. Thus the average speed was 440/7 = 62 6/7 kilometers per hour. Note: In this example the average speed is not the average of the two separate speeds, which would be 65. Properties of Signed Numbers positive x negative = negative 38 © . negative x negative = positive negative x positive = negative positive x positive = positive Factoring You may need to apply these types of simple factoring: x^2 + 2x = x(x + 2) x^2 – 1 = (x + 1) (x – 1) x^2 + 2x +1 = (x + 1) (x + 1) = (x + 1)^2 x^2 – 3x – 4 = (x – 4)(x + 1) Probability Probability refers to the chance that a specific outcome can occur. It can be found by using the following definition when outcomes are equally likely. Number of ways that a specific outcome can occurTotal number of possible outcomes For example, if a jar contains 13 red marbles and 7 green marbles, the probability that a marble selected from the jar at random will be green is 7 / 7 +13 = 7/20 = or 0.35 If a particular outcome can never occur, its probability is 0. If an outcome is certain to occur, its probability is 1. In general, if p is the 39 © . probability that a specific outcome will occur, values of p fall in the range 0 < p < 1. Probability may be expressed as either a decimal or a fraction. Geometric Figures Figures that accompany problems are intended to provide information useful in solving the problems. They are drawn as accurately as possible EXCEPT when it is stated in a particular problem that the figure is not drawn to scale. In general, even when figure is not drawn to scale, the relative positions of points and angles may be assumed to be in the order shown. Also, line segments that extend through points and appear to lie on the same line may be assumed to be on the same line. The text “Note: Figure not drawn to scale.” is included on the test when degree measures may not be accurately shown and specific lengths may not be drawn proportionally. The following examples

Page 112: TABE PERT FCAT Reading and Writing Practice    MmAcVcErReIaCKS

illustrate the way different figures can be interpreted. Example 1 Since UY and VX are line segments, angels UWV and XWY are vertical angles. Therefore, you can conclude that c° = d°. Even though the figure is drawn to scale, you should NOT make any other assumptions without additional information. For example, you should NOT assume 40 © . that VW = WY or that the angle at vertex Y is a right angle even though they may look that way in the figure. Example 2 A question may refer to a triangle such as XWZ above. Although the note indicates that the figure is not drawn to scale, you may assume that: (1) XWY and YWZ are triangles. (2) Y is between X and Z. (3) X, Y, and Z are points on a line. (4) The length of XY is less than the length of XZ. (5) The measure of angle XWY is less than the measure of angle XWZ. You may not assume the following: (1) The length of XY is less than the length of YZ. (2) The measures of angles WXY and WYX are equal. (3) The measure of angle XWY is greater than the measure of angle WYX. (4) Angle XWZ is a right angle. 41 © . Geometric Skills and Concepts Properties of Parallel Lines 1. If two parallel lines are cut by a third line, the alternate interior angles are equal. a° = b° and d° = c°2. If two parallel lines are cut by a third line, the corresponding angles are equal. a° = b° and d° = c°Note: Words like “alternate interior” or “corresponding” are generally not used on the test, but you do need to know which angles involving parallel lines are equal. 3. If two parallel lines are cut by a third line, the sum of the interior angles on the same side of the third line is 180 degrees. 42 © . a° + b° = 180°, because a° + c° = 180° and b° = c°Angle Relationships 1. The sum of the interior angles of a triangle is 180 degrees. a° = 70° (Because 70° + 40° + a° = 180°.) 2. When two lines intersect, vertical angles are equal. a = b 3. A straight angle measures 180 degrees. a° = 60 (Because a° + 120° = 180°.) 43 © . 4. The sum of the two acute angles in a right triangle is 90 degrees. x = 15 (Because 2x + 4x = 90.)

Page 113: TABE PERT FCAT Reading and Writing Practice    MmAcVcErReIaCKS

5. The sum of the interior angles of a polygon can be found by drawing all diagonals of the polygon from one vertex and multiplying the number of triangles formed by 180 degrees. Since the polygon is divided into 3 triangles, the sum of the angles is 3 x 180° or 540°. Side Relationships 1. Pythagorean Theorem: In any right triangle, a2 + b2 = c2, where c is the length of the longest side and a and b are the lengths of the two shorter sides. a = 5 (By the Pythagorean Theorem, 44 © . a2= 32 + 42a2 = 9 = 16 a2 = 25 a = square root of 25 = 5 2. In any equilateral triangle, all sides are equal and all angles are equal. a = b = 5 (Because the measure of the unmarked angle is 60°, the measure of all angles of the triangle are equal, and therefore, the lengths of all sides of the triangle are equal.) 3. In an isosceles triangle, the angles opposite equal sides are equal. Also the sides opposite equal angles are equal. If A = B, then a° = b°. Also, if a° = b°, then A = B. 4. In any triangle, the longest side is opposite the largest angle ( and the shortest side is opposite the smallest angle.) A>B>C 45 © . 5. Two polygons are similar if the lengths of their corresponding sides are in the same ratio and their corresponding angles are equal. If polygons ABCD and EFGH are similar, and if BC and FG are corresponding sides, then BC = 3 and FG = 2. Therefore, the ratio is 3:2 and since AB = 6, EF = 4 Area and Perimeter Rectangles

Page 114: TABE PERT FCAT Reading and Writing Practice    MmAcVcErReIaCKS

Area of a rectangle = length x width = l x w Perimeter of a rectangle = 2(l + w) = 2l x 2w Area = 5x X 8x = 40x2Perimeter = 2(5x + 8x) = 10x + 16x = 26x Circles 46 © . Area of a circle = Πr2 (where r is the radius) Circumference of a circle = 2Πr = Πd (where d is the diameter) Area = Π22 = 4ΠCircumference = 2Π2 = 4ΠTriangles Area of a triangle = ½ (base X height) Perimenter = Sum of lengths Area = ½ (4 X 3) = 6 Perimeter = 5 + 4 + 3 = 12 Volume Volume of a rectangular solid or cube = length X width X height = l X w X h Volume = 3 X 2 X 4 = 24 47 © . Volume of a cylinder = Πr2h (where r is the radius of the base and h is the height of the cylinder) Volume = Π X 42 X 7 = Π X 16 X 7 = 112ΠCoordinate Geometry In questions that involve the x and y axes, x values to the right of the y axis are positive and x values to the left of the y axis are negative. Also, y values above the x axis are positive and y values below the x axis are negative. In an (x,y) ordered pair, the x value is written first, and the y value is written second. For example, in the ordered pair (1,-2), the x coordinate is 1 and the y coordinate is -2. Slope of a line = rise/run or vertical distance/horizontal distance. 48 © . This line runs through points (1,-2) and (4,4). The slope = (4 – (-2))/(4 – 1) or 6/3 = 2. Any line that slopes upward from left to right has a positive slope. Any line that slopes downward from right to left has a negative slope. 49 © .

===========

Page 115: TABE PERT FCAT Reading and Writing Practice    MmAcVcErReIaCKS

TABE Test Improvement Links:

Basic Algebra Resource 1 - www.sosmath.com/algebra/quadraticeq/bdef/bdef.html

Basic Algebra Resource 2 - www.sosmath.com/algebra/quadraticeq/root/root.html

Basic Algebra Resource 3 - www.mathleague.com/help/algebra/algebra.htm#variables

Advanced Algebra Resource - www.sosmath.com/algebra/quadraticeq/root/root.html

Averages and Rounding Resource - www.math.com/school/subject1/lessons/S1U1L3GL.html#

Basic Operations Resource - www.hyper-ad.com/tutoring/math/algebra/basic_math_operations.html

Commas Resource - grammar.ccc.commnet.edu/grammar/commas.htm

Estimation and Sequences Resource - www.math.com/school/subject1/lessons/S1U1L3GL.html#

Exponents Resource 1 - www.sosmath.com/algebra/logs/log3/log3.html

Exponents Resource 2 - www.jug.net/wt/exponentrules.htm

Fractions and Square Roots Resource 1 - www.sosmath.com/algebra/fraction/frac3/frac3.html

Fractions and Square Roots Resource 2 - www.sosmath.com/algebra/fraction/frac4/frac4.html

Fractions and Square Roots Resource 3 - www.sosmath.com/algebra/fraction/frac8/frac8.html

Fractions and Square Roots Resource 4 - www.math.com/school/subject1/lessons/S1U1L9GL.html

Geometry Resource - library.thinkquest.org/2609/

Graphs Resource - www.sosmath.com/algebra/unitconv/unit2/unit2.html

Measurement Resource 1 - www.sosmath.com/algebra/unitconv/unit3/unit3.html

Measurement Resource 2 - www.sosmath.com/algebra/unitconv/unit4/unit4.html

Nouns Resource 1 - grammar.ccc.commnet.edu/grammar/nouns.htm

Nouns Resource 2 - depts.dyc.edu/learningcenter/owl/nouns.htm

Percents and Ratios - www.sosmath.com/algebra/fraction/frac7/frac7.html

Vocabulary Resource - grammar.ccc.commnet.edu/grammar/vocabulary.htm

Verb Resource 1 - grammar.ccc.commnet.edu/grammar/verbs.htm

Verb Resource 2 - grammar.ccc.commnet.edu/grammar/auxiliary.htm

Confusing Words with Reading - grammar.ccc.commnet.edu/grammar/notorious.htm

Compound Words - grammar.ccc.commnet.edu/grammar/compounds.htm

ParaPro Test Improvement Links:

Basic Algebra Resource 1 - www.sosmath.com/algebra/quadraticeq/bdef/bdef.html

Basic Algebra Resource 2 - www.sosmath.com/algebra/quadraticeq/root/root.html

Basic Algebra Resource 3 - www.mathleague.com/help/algebra/algebra.htm#variables

Page 116: TABE PERT FCAT Reading and Writing Practice    MmAcVcErReIaCKS

Advanced Algebra Resource - www.sosmath.com/algebra/quadraticeq/root/root.html

Averages and Rounding Resource - www.math.com/school/subject1/lessons/S1U1L3GL.html#

Basic Operations Resource - www.hyper-ad.com/tutoring/math/algebra/basic_math_operations.html

Commas Resource - grammar.ccc.commnet.edu/grammar/commas.htm

Estimation and Sequences Resource - www.math.com/school/subject1/lessons/S1U1L3GL.html#

Exponents Resource 1 - www.sosmath.com/algebra/logs/log3/log3.html

Exponents Resource 2 - www.jug.net/wt/exponentrules.htm

Fractions and Square Roots Resource 1 - www.sosmath.com/algebra/fraction/frac3/frac3.html

Fractions and Square Roots Resource 2 - www.sosmath.com/algebra/fraction/frac4/frac4.html

Fractions and Square Roots Resource 3 - www.sosmath.com/algebra/fraction/frac8/frac8.html

Fractions and Square Roots Resource 4 - www.math.com/school/subject1/lessons/S1U1L9GL.html

Geometry Resource - library.thinkquest.org/2609/

Graphs Resource - www.sosmath.com/algebra/unitconv/unit2/unit2.html

Measurement Resource 1 - www.sosmath.com/algebra/unitconv/unit3/unit3.html

Measurement Resource 2 - www.sosmath.com/algebra/unitconv/unit4/unit4.html

Nouns Resource 1 - grammar.ccc.commnet.edu/grammar/nouns.htm

Nouns Resource 2 - depts.dyc.edu/learningcenter/owl/nouns.htm

Percents and Ratios - www.sosmath.com/algebra/fraction/frac7/frac7.html

Vocabulary Resource - grammar.ccc.commnet.edu/grammar/vocabulary.htm

Verb Resource 1 - grammar.ccc.commnet.edu/grammar/verbs.htm

Verb Resource 2 - grammar.ccc.commnet.edu/grammar/auxiliary.htm

Confusing Words with Reading - grammar.ccc.commnet.edu/grammar/notorious.htm

Compound Words - grammar.ccc.commnet.edu/grammar/compounds.htm

SAT Improvement Links:

Basic Algebra Resource 1 - www.sosmath.com/algebra/quadraticeq/bdef/bdef.html

Basic Algebra Resource 2 - www.sosmath.com/algebra/quadraticeq/root/root.html

Basic Algebra Resource 3 - www.mathleague.com/help/algebra/algebra.htm#variables

Advanced Algebra Resource - www.sosmath.com/algebra/quadraticeq/root/root.html

Averages and Rounding Resource - www.math.com/school/subject1/lessons/S1U1L3GL.html#

Basic Operations Resource - www.hyper-ad.com/tutoring/math/algebra/basic_math_operations.html

Commas Resource - grammar.ccc.commnet.edu/grammar/commas.htm

Estimation and Sequences Resource - www.math.com/school/subject1/lessons/S1U1L3GL.html#

Page 117: TABE PERT FCAT Reading and Writing Practice    MmAcVcErReIaCKS

Exponents Resource 1 - www.sosmath.com/algebra/logs/log3/log3.html

Exponents Resource 2 - www.jug.net/wt/exponentrules.htm

Fractions and Square Roots Resource 1 - www.sosmath.com/algebra/fraction/frac3/frac3.html

Fractions and Square Roots Resource 2 - www.sosmath.com/algebra/fraction/frac4/frac4.html

Fractions and Square Roots Resource 3 - www.sosmath.com/algebra/fraction/frac8/frac8.html

Fractions and Square Roots Resource 4 - www.math.com/school/subject1/lessons/S1U1L9GL.html

Geometry Resource - library.thinkquest.org/2609/

Graphs Resource - www.sosmath.com/algebra/unitconv/unit2/unit2.html

Measurement Resource 1 - www.sosmath.com/algebra/unitconv/unit3/unit3.html

Measurement Resource 2 - www.sosmath.com/algebra/unitconv/unit4/unit4.html

Nouns Resource 1 - grammar.ccc.commnet.edu/grammar/nouns.htm

Nouns Resource 2 - depts.dyc.edu/learningcenter/owl/nouns.htm

Percents and Ratios - www.sosmath.com/algebra/fraction/frac7/frac7.html

Vocabulary Resource - grammar.ccc.commnet.edu/grammar/vocabulary.htm

Verb Resource 1 - grammar.ccc.commnet.edu/grammar/verbs.htm

Verb Resource 2 - grammar.ccc.commnet.edu/grammar/auxiliary.htm

Resources for Writing an Essay - grammar.ccc.commnet.edu/grammar/composition/brainstorm_freewrite.htm

Five Paragraph Essay - grammar.ccc.commnet.edu/grammar/five_par.htm

Evaluative Essay - grammar.ccc.commnet.edu/grammar/composition/review.htm

Argumentative Essay - grammar.ccc.commnet.edu/grammar/composition/argument.htm

Overcoming Writer's Block - grammar.ccc.commnet.edu/grammar/composition/brainstorm_block.htm

Essay Plague Words - grammar.ccc.commnet.edu/grammar/plague.htm

Topic Sentences - grammar.ccc.commnet.edu/grammar/paragraphs.htm#topic_sentences

Confusing Words with Reading - grammar.ccc.commnet.edu/grammar/notorious.htm

Compound Words - grammar.ccc.commnet.edu/grammar/compounds.htm

FCAT Test Improvement Links:

Basic Algebra Resource 1 - www.sosmath.com/algebra/quadraticeq/bdef/bdef.html

Basic Algebra Resource 2 - www.sosmath.com/algebra/quadraticeq/root/root.html

Basic Algebra Resource 3 - www.mathleague.com/help/algebra/algebra.htm#variables

Advanced Algebra Resource - www.sosmath.com/algebra/quadraticeq/root/root.html

Averages and Rounding Resource - www.math.com/school/subject1/lessons/S1U1L3GL.html#

Basic Operations Resource - www.hyper-ad.com/tutoring/math/algebra/basic_math_operations.html

Page 118: TABE PERT FCAT Reading and Writing Practice    MmAcVcErReIaCKS

Commas Resource - grammar.ccc.commnet.edu/grammar/commas.htm

Estimation and Sequences Resource - www.math.com/school/subject1/lessons/S1U1L3GL.html#

Exponents Resource 1 - www.sosmath.com/algebra/logs/log3/log3.html

Exponents Resource 2 - www.jug.net/wt/exponentrules.htm

Fractions and Square Roots Resource 1 - www.sosmath.com/algebra/fraction/frac3/frac3.html

Fractions and Square Roots Resource 2 - www.sosmath.com/algebra/fraction/frac4/frac4.html

Fractions and Square Roots Resource 3 - www.sosmath.com/algebra/fraction/frac8/frac8.html

Fractions and Square Roots Resource 4 - www.math.com/school/subject1/lessons/S1U1L9GL.html

Geometry Resource - library.thinkquest.org/2609/

Graphs Resource - www.sosmath.com/algebra/unitconv/unit2/unit2.html

Measurement Resource 1 - www.sosmath.com/algebra/unitconv/unit3/unit3.html

Measurement Resource 2 - www.sosmath.com/algebra/unitconv/unit4/unit4.html

Nouns Resource 1 - grammar.ccc.commnet.edu/grammar/nouns.htm

Nouns Resource 2 - depts.dyc.edu/learningcenter/owl/nouns.htm

Percents and Ratios - www.sosmath.com/algebra/fraction/frac7/frac7.html

Verb Resource 1 - grammar.ccc.commnet.edu/grammar/verbs.htm

Verb Resource 2 - grammar.ccc.commnet.edu/grammar/auxiliary.htm

Confusing Words with Reading - grammar.ccc.commnet.edu/grammar/notorious.htm

Resources for Writing an Essay - grammar.ccc.commnet.edu/grammar/composition/brainstorm_freewrite.htm

Five Paragraph Essay - grammar.ccc.commnet.edu/grammar/five_par.htm

Evaluative Essay - grammar.ccc.commnet.edu/grammar/composition/review.htm

Argumentative Essay - grammar.ccc.commnet.edu/grammar/composition/argument.htm

Overcoming Writer's Block - grammar.ccc.commnet.edu/grammar/composition/brainstorm_block.htm

Essay Plague Words - grammar.ccc.commnet.edu/grammar/plague.htm

ASVAB Test Improvement Links:

Basic Algebra Resource 1 - www.sosmath.com/algebra/quadraticeq/bdef/bdef.html

Basic Algebra Resource 2 - www.sosmath.com/algebra/quadraticeq/root/root.html

Basic Algebra Resource 3 - www.mathleague.com/help/algebra/algebra.htm#variables

Advanced Algebra Resource - www.sosmath.com/algebra/quadraticeq/root/root.html

Averages and Rounding Resource - www.math.com/school/subject1/lessons/S1U1L3GL.html#

Basic Operations Resource - www.hyper-ad.com/tutoring/math/algebra/basic_math_operations.html

Commas Resource - grammar.ccc.commnet.edu/grammar/commas.htm

Page 119: TABE PERT FCAT Reading and Writing Practice    MmAcVcErReIaCKS

Estimation and Sequences Resource - www.math.com/school/subject1/lessons/S1U1L3GL.html#

Exponents Resource 1 - www.sosmath.com/algebra/logs/log3/log3.html

Exponents Resource 2 - www.jug.net/wt/exponentrules.htm

Fractions and Square Roots Resource 1 - www.sosmath.com/algebra/fraction/frac3/frac3.html

Fractions and Square Roots Resource 2 - www.sosmath.com/algebra/fraction/frac4/frac4.html

Fractions and Square Roots Resource 3 - www.sosmath.com/algebra/fraction/frac8/frac8.html

Fractions and Square Roots Resource 4 - www.math.com/school/subject1/lessons/S1U1L9GL.html

Geometry Resource - library.thinkquest.org/2609/

Graphs Resource - www.sosmath.com/algebra/unitconv/unit2/unit2.html

Measurement Resource 1 - www.sosmath.com/algebra/unitconv/unit3/unit3.html

Measurement Resource 2 - www.sosmath.com/algebra/unitconv/unit4/unit4.html

Nouns Resource 1 - grammar.ccc.commnet.edu/grammar/nouns.htm

Nouns Resource 2 - depts.dyc.edu/learningcenter/owl/nouns.htm

Percents and Ratios - www.sosmath.com/algebra/fraction/frac7/frac7.html

Vocabulary Resource - grammar.ccc.commnet.edu/grammar/vocabulary.htm

Verb Resource 1 - grammar.ccc.commnet.edu/grammar/verbs.htm

Verb Resource 2 - grammar.ccc.commnet.edu/grammar/auxiliary.htm

Topic Sentences - grammar.ccc.commnet.edu/grammar/paragraphs.htm#topic_sentences

Confusing Words with Reading - grammar.ccc.commnet.edu/grammar/notorious.htm

Compound Words - grammar.ccc.commnet.edu/grammar/compounds.htm